You are on page 1of 236

BỘ GIÁO DỤC VÀ ĐÀO TẠO

VỤ GIÁO DỤC TRUNG HỌC CHƯƠNG TRÌNH PHÁT TRIỂN


GIÁO DỤC TRUNG HỌC

TÀI LIỆU TẬP HUẤN


PHÁT TRIỂN CHUYÊN MÔN GIÁO VIÊN
TRƯỜNG THPT CHUYÊN

MÔN TOÁN
(Tài liệu lưu hành nội bộ)

Hà Nội, tháng 7 năm 2012


Chủ trì biên soạn:
Vụ Giáo dục Trung học
Chương trình phát triển giáo dục trung học

NHÓM TÁC GIẢ BIÊN SOẠN TÀI LIỆU:

1. GS.TSKH Hà Huy Khoái


2. GS.TSKH Nguyễn Văn Mậu
3. GS.TSKH Đặng Hùng Thắng
4. PGS.TSKH Vũ Đình Hòa
5. PGS.TS Nguyễn Vũ Lương
6. TS Phạm Văn Quốc
7. TS Lê Anh Vinh
8. TS Trịnh Đào Chiến
Mục lục

Lời nói đầu 4

Hà Huy Khoái
Một số bài toán Số học - Tổ hợp 5

Nguyễn Văn Mậu


Lớp các phương trình hàm Cauchy, d’Alembert và dạng toán liên quan 22

Đặng Hùng Thắng


Một số lớp phương trình Diophant cơ bản 66

Vũ Đình Hoà
Bài toán tô màu đồ thị 105

Nguyễn Vũ Lương
Một cách tiếp cận tới bài toán tổ hợp 134

Trịnh Đào Chiến


Một số dạng bất phương trình hàm 188

Phạm Văn Quốc


Phương trình Pell 207

Lê Anh Vinh
Bất biến và nửa bất biến 220

3
Lời nói đầu
Hoạt động bồi dưỡng theo các bộ môn, phân theo các cụm, khu vực theo
địa hình và đặc thù văn hoá, ... đã trở thành sinh hoạt chuyên môn truyền
thống và ngày càng đi vào nề nếp trong hệ thống các trường trung học chuyên
và năng khiếu bậc phổ thông. Nhờ đó, các đơn vị, các trường THPT chuyên
chủ động xây dựng chương trình hành động và lựa chọn cách thức triển khai.

Đặc biệt chương trình tập huấn phát triển năng lực chuyên môn giáo viên
trường THPT chuyên gắn với các hoạt động bồi dưỡng chuyên môn nghiệp
vụ và năng lực tổ chức các hoạt động xã hội cho giáo viên đang giảng dạy ở
các đội tuyển các trường THPT Chuyên.

Ban tổ chức đã xây dựng nội dung, chương trình kế hoạch cho các hoạt
động bồi dưỡng chuyên môn nghiệp vụ; liên hệ mời các giáo sư, các nhà khoa
học có kinh nghiệm và tâm huyết trực tiếp giảng bài và tổ chức các semina
khoa học.

Sản phẩm của chương trình tập huấn là đã xây dựng một tập san (tài liệu
tập huấn) trong đó lưu giữ các nội dung gồm các bài viết, bài giảng, các đề
thi Olympic đề xuất kèm theo lời giải và giới thiệu những xu hướng mới cập
nhật với olympic khu vực và quốc tế.

Vì thời gian rất gấp gáp, nên các khâu chế bản và nội dung cuốn tài liệu
tập huấn này chắc chắn còn nhiều khiếm khuyết. Mong nhận được sự góp ý
của các thầy, cô và các đồng nghiệp.

Ban Tổ chức

4
Một số bài toán Số học - Tổ hợp

Hà Huy Khoái
Viện Toán học

Bài giảng này nhằm mục tiêu giới thiệu một số bài toán có thể gọi là thuộc
loại "số học - tổ hợp". Thực ra không có một "định nghĩa" nào cho loại bài
toán đó, nên ở đây chỉ giới hạn ở việc đưa ra một số ví dụ về loại bài toán
thường gặp trong những kỳ thi học sinh giỏi, mà việc giải chúng đòi hỏi những
phương pháp của số học và tổ hợp. Để tiện theo giõi, chũng tôi tạm chia bài
giảng thành bốn phần: Tỷ số vàng, Các dãy nhị phân, Tính chia hết và Trò
chơi.
Khi trình bày các lời giải, trong chừng mực có thể, chúng tôi cố gắng mô
tả quá trình hình thành nên lời giải đó, hơn là đưa ra một lời giải ngắn gọn.

§1. Tỷ số vàng.
Chúng ta đều biết "tỷ số vàng" sau đây thường xuất hiện trong khoa học,
nghệ thuật và đời sống √
1+ 5
.
2
Tỷ số vàng đó cũng thường bắt gặp trong lời giải của những bài toán số học
- tổ hợp. Trước tiên ta xét ví dụ sau:
Bài toán 1. Giả sử γ, δ là những số vô tỷ dương, thỏa mãn
1 1
+ = 1.
γ δ

5
Chứng minh rằng nếu đặt an = [nγ], bn = [nδ] thì mỗi số nguyên dương xuất
hiện đúng một lần trong một trong hai dãy an , bn .
Phân tích - Lời giải
Rõ ràng yêu cầu của bài toán tương đương với việc chứng minh rằng, các
số trong mỗi đoạn hữu hạn tùy ý [1, 2, · · · , N ] có mặt ở một trong hai dãy, và
xuất hiện đúng một lần. Như vậy vấn đề chỉ còn là đếm xem trong N − 1 số
nguyên dương nhỏ hơn N , có bao nhiêu số thuộc một trong hai dãy nói trên.
N
Xét mọi số nguyên dương n thỏa mãn [nγ] < N , tức là n < γ. Như vậy,
các số n thỏa mãn là n = 1, 2, · · · , [ Nγ ]. Tương tự, các số m sao cho [mδ] < N
là m = 1, 2, · · · , [ Nδ ].
Như vậy, trong các số nguyên dương nhỏ hơn N , số các số thuộc một trong
hai dãy an , bn là [ Nγ ] + [ Nδ ]. Do γ; δ là các số vô tỷ nên [ Nγ ]; [ Nδ ] 6∈ Z. Từ đó ta
có:

N N N
−1<[ ]< ,
γ γ γ
N N N
−1<[ ]< ,
δ δ δ
suy ra
1 1 N N 1 1
N ( + ) − 2 = N − 2 < [ ] + [ ] < N ( + ) = N.
γ δ γ δ γ δ
Do đó
N N
[ ] + [ ] = N − 1.
γ δ
Như vậy trong N − 1 số nhỏ hơn N có đúng N − 1 số thuộc một trong hai dãy
đang xét, đ.p.c.m.

Bài toán trên đây là một "thành phần" của rất nhiều bài toán tổ hợp.
Trong bài giảng này, chúng ta sẽ xem xét hai bài thuộc loại đó.
Bài toán 2.

6
Tìm các dãy tăng các số nguyên dương {an }; {bn } thỏa mãn những tính
chất sau:
1/ a1 = 1.
2/ Với mọi n ≥ 1, bn = an + n.
3/ an là số nguyên dương nhỏ nhất không thuộc tập hợp {a1 , a2 , · · · , an−1 ;
b1 , b2 , · · · , bn−1 }.
Rõ ràng ba điều kiện nói trên xác định một cách duy nhất các dãy
{an }, {bn }. Hơn nữa, đối với hai dãy tăng, việc thỏa mãn các điều kiện 1/
, 2/, 3/ tương đương với việc thỏa mãn các điều kiện 1/, 2/ , và 3’/ như sau:
3’/ Mỗi số nguyên dương đều thuộc một và chỉ một trong hai dãy đang
xét.
Do tính xác định duy nhất của các dãy thỏa mãn 1/, 2/, 3’/, ta chỉ cần
chứng minh sự tồn tại, bằng cách chỉ ra ví dụ cụ thể. Bài toán 1 cho ta cách
tìm hai dãy thỏa mãn điều kiện 3’/: đó chính là các dãy an = [nγ], bn = [nδ],
trong đó γ, δ là những số vô tỷ dương, thỏa mãn
1 1
+ = 1.
γ δ

Vấn đề chỉ là tìm γ để các điều kiện 1/ và 2/ được thỏa mãn. Để ý rằng

n = n + [nγ] − [nγ] = [n + nγ] − [nγ] = [(1 + γ)n] − [nγ].

Như vậy chỉ cần chọn γ vô tỷ, thỏa mãn:


1 1
+ = 1.
γ γ+1

1+ 5
Nghiệm của phương trình trên là "tỷ số vàng" 2 .
Các dãy an , bn cần tìm là:
√ √
1+ 5 1+3 5
an = [ n]; bn = [ n].
2 2

7
Bài toán 1 và Bài toán 2 lại có thể làm "thành phần" cho bài toán phức
tạp hơn sau đây:
Bài toán 3
Lập dãy số theo cách sau: lấy x1 = 1, với i ≥ 2 số xi nhận được từ xi−1
bằng cách đổi (trong cách viết số xi−1 ) số 1 thành 01, số 0 thành 1. Làm như
vậy, ta nhận được dãy số 1, 01, 101, 01101,...Trong dãy này, gọi an là vị trí
của chữ số 1 thứ n, bn là vị trí của chữ số 0 thứ n (như vậy a1 = 1, a2 = 3, a3 =
4, b1 = 2, b2 = 5, · · · ).
Tìm công thức xác định an , bn .
Phân tích - Lời giải
Trước tiên ta cần tìm một công thức xác định mối liên hệ giữa an và bn .
Gọi kn là số chữ số 0 đứng trước chữ số 1 thứ n. Theo định nghĩa hai dãy
đang xét ta có
an = n + kn .

Theo bài ra, chữ số 0 thứ n được "sinh ra" từ chữ số 1 thứ n. Mặt khác, chữ
số 1 biến thành hai chữ số 01, chữ số 0 biến thành một chữ số 1. Trước chữ số
1 thứ n có kn chữ số 0, và "biến thành kn chữ số; còn n chữ số 1 "biến thành"
2n chữ số. Từ đó suy ra:
bn = kn + 2n.

Từ hai công thức trên đây, ta có

bn = an + n.

Vì an và bn đều là các "số thứ tự" nên hai dãy là dãy tăng, đồng thời mỗi
một số nguyên dương xuất hiện đúng một lần trong một trong hai dãy.
Các Bài toán 1 và Bài toán 2 cho ta đáp số:
√ √
1+ 5 1+3 5
an = [ n]; bn = [ n].
2 2

8
Trong phần các bài toán về trò chơi, ta sẽ gặp lại "tỷ số vàng".

§2 Các dãy nhị phân


Trong rất nhiều bài toán tổ hợp, đặc biệt là các bài toán "đếm", ta thường
gặp những tình huống mà tại đó có hai khả năng xẩy ra: được tô bởi hai màu;
đường đi chỉ được phép sang phải hoặc đi lên; học sinh nam hay nữ, số chẵn
hoặc lẻ; ...Về thực chất, những bài toán như vậy luôn luôn có thể đưa về cùng
một dạng phát biểu, trong đó thông thường nhất là dùng các dãy nhị phân
(các dãy gồm hai chữ số 0 và 1).
Để hiểu rõ hơn điều đó, ta xét bài toán sau đây.
Bài toán 4.
Sau giờ học, các em học sinh xếp hàng để nhận xe đạp ở nhà gửi xe. Giá
tiền gửi mỗi xe là 1000 đồng. Giả sử có k em học sinh có tờ 1000 đồng, m em
có tờ 2000 đồng. Hỏi có bao nhiêu cách xếp hàng lấy xe sao cho không em
nào phải chờ để lấy tiền trả lại? (Với giả thiết người giữ xe không có đồng
tiền lẻ nào).
Phân tích - Lời giải
Đây là một bài toán thuộc loại "hai khả năng": mỗi em học sinh hoặc có
tờ 1000 đồng, hoặc có tờ 2000 đồng. Như vậy, để dễ thấy bản chất bài toán,
ta có thể lập tương ứng mỗi hàng học sinh với một dãy gồm hai chữ số 0, 1.
Giả sử ứng với mỗi học sinh có tờ 1000 đồng trong hàng, ta viết số 0; ứng với
học sinh có tờ 2000 đồng, ta viết số 1. Như vậy, mỗi hàng học sinh tương ứng
một dãy gồm k chữ số 0, m chữ số 1. Để tồn tại cách xếp mà không có em
nào phải chờ lấy tiền trả lại, điều kiện cần là k ≥ m.
Cũng tương tự như trong nhiều bài toán tổ hợp khác, khi việc đếm số phần
tử thỏa mãn bài ra là khó, ta đếm "phần bù" của nó, tức là những phần tử
không thỏa mãn bài ra. Như vậy, ở đây ta sẽ xét xem có bao nhiêu hàng mà

9
có học sinh nào đó đến lượt mình phải chờ lấy tiển trả lại. Theo cách tương
ứng của chúng ta, một hàng như vậy sẽ tương ứng một - một với một dãy
gồm k số 0, m số 1, trong đó tồn tại vị trí 2s + 1 sao cho tại đó có số 1, đồng
thời ở 2s vị trí đầu, số số 0 và số số 1 là như nhau. Ta gọi một hàng như vậy
là "hàng xấu".
Nếu bạn là người giữ xe thì bạn sẽ làm thế nào trong tình huống đó?
Hiển nhiên là gọi thêm một bạn nào đó có tờ 1000 đồng lên đứng trước hàng!
Phương pháp "thực tiễn" này gợi cho ta lập tương ững mỗi hàng xấu gồm k
chữ số 0, m chữ số 1 với một hàng gồm k + 1 chữ số 0, m chữ số 1, với chữ số
0 đứng đầu. Đồng thời, trong 2s + 2 vị trí đầu tiên, số chữ số 0 và số chữ số
1 là như nhau.
Nếu ta đổi số 0 thành số 1, số 1 thành số 0 ở 2s + 2 vị trí đầu, một hàng
như trên sẽ tương ứng với một hàng gồm k + 1 chữ số 0, m chữ số 1, nhưng
với số 1 đứng đầu tiên. Lại bỏ đi số 1 đầu tiên này, ta được một hàng gồm
k + 1 chữ số 0, m − 1 chữ số 1.
Như vậy, mỗi hàng xấu gồm k chữ số 0, m chữ số 1 sẽ được tương ứng với
một hàng gồm k + 1 chữ số 0, m − 1 chữ số 1 theo cách trên đậy. Dễ thấy rằng,
tương ứng như trên là một - một. Thật vậy, xét một hàng tùy ý gồm k + 1
chữ số 0, m − 1 chữ số 1. Ta thêm số 1 vào đầu hàng, để được hàng gồm k + 1
chữ số 0, m chữ số 1. Do điều kiện k ≥ m nên trong hàng này phải tồn tại vị
trí (2s + 2) mà từ đó trở lên, số số 0 bằng số số 1. Ta đổi số 0 thành số 1 và
ngược lại ở các vị trí từ đó trở lên, để được hàng gồm k + 1 chữ số 0, m chữ
số 1, với số 0 đứng đầu. Bỏ số 0 đầu tiên này, ta nhận được một hàng xấu.
Từ những tương ứng trên suy ra rằng, số các hàng xấu gồm k chữ số 0, m
chữ số 1 bằng số các hàng (tùy ý) với k + 1 chữ số 0, m − 1 chữ số 1, tức là
k+1
bằng Cm+k .
Như vậy, số cách xếp hàng sao cho không học sinh nào phải chờ lấy tiền

10
trả lại là:
k k+1
Cm+k − Cm+k .

Bài toán 5.
Có 2k học sinh chiều cao khác nhau đôi một. Người ta muốn xếp họ thành
hai hàng ngang, sao cho trong mỗi hàng, chiều cao của học sinh giảm dần,
và ở mỗi vị trí, em đứng hàng trước cao hơn em đứng hàng sau. Hỏi có bao
nhiêu cách xếp hàng thỏa mãn yêu cầu?
Phân tích - Lời giải
Thoạt nhìn thì đây là bài toán khác hẳn với bài toán trên. Tuy nhiên, nếu
ta đã nắm vững nguyên tắc "hai khả năng" thì thấy cả hai bài chỉ cùng một
loại. Thật vậy, vấn đề ở đây chỉ là xếp học sinh vào một trong hai hàng.
Với quan niệm như trên, ta cho mỗi em đứng hàng trước cầm một tấm
biển ghi số "0", mỗi em hàng sau cầm tấm biểm ghi số "1". Sau đó gọi tất cả
2k em xếp lại thành một hàng dọc theo thứ tự chiều cao giảm dần. Làm như
vậy, ta được một dãy gồm k chữ số 0, k chữ số 1.
Dễ thấy rằng, một cách xếp hàng thỏa mãn bài ra chính là một cách xếp
hàng sao cho tại mỗi vị trí 2s + 1, số số 0 từ vị trí đầu tiên đến đó phải ≥ s + 1.
Theo Bài toán 4, số cách xếp hàng hỏa mãn bài ra là:

k k+1 (2k)!
C2k − C2k = .
k!(k + 1)!

§3. Tính chia hết


Ta bắt đầu với bài toán đơn giản sau.
Bài toán 6.
Tìm tất cả các tập hợp A = {a1 , a2 , · · · , an ; n > 2012} gồm những số nguyên
và có tính chất sau: 2012 ∈ A, đồng thời mỗi tập con tùy ý gồm 2012 số thuộc

11
A đều có thể chia thành 4 nhóm có số phần tử bằng nhau và tổng các phần
tử trong mỗi nhóm bằng nhau.
Phân tích - Lời giải.
Điều kiện của bài toán cho ta thấy rằng, tổng của 2012 số tùy ý thuộc A
là một số chia hết cho 4. Thay từ tập hợp 2012 phần tử một phần tử bất kỳ
bởi một phần tử khác tùy ý không thuộc tập đã chọn, tính chất tổng chia hết
cho 4 không hề thay đổi. Như vậy có thể thấy ngay rằng, mọi phần tử thuộc
A đều đồng dư nhau modulo 4.

Dĩ nhiên ta nẩy ra ngay dự đóan rằng, các phần tử của A đều bằng nhau.
Để có thể kiểm chứng dự đoán này, ta xét phần tử nhỏ nhất của A : ā. Khi
đó tập hợp B = {a1 − ā, a2 − ā, · · · , an − ā} rõ ràng cũng thỏa mãn những tính
chất đã nêu trong bài toán như đối với tập hợp A. Do đó, mọi phần tử của
B cũng đồng dư nhau modulo 4,và vì B chứa phần tử bàng 0 nên suy ra mọi
phần tử của B đều chia hết cho 4.
Từ đây, suy luận "quy nạp lùi" quen thuộc cho ta lời giải: tập hợp nhận
b
được từ B bằng cách thay mọi phần tử b ∈ B bởi 4 cũng có tính chất nêu
trong bài ra; và do đó các phần tử của tập hợp này cũng chia hết cho 4. Tiếp
tục quá trình,dễ suy ra mọi phần tử của B đều bằng 0. Như vậy, mọi số thuộc
A đều bằng 2012.

Bài toán 7
Với mỗi số nguyên dương d, gọi f (d) là số nguyên dương nhỏ nhất có đúng
d ước số dương. Chứng minh rằng với mọi k ≥ 0, f (2k+1 ) chia hết cho f (2k ).

Phân tích- Lời giải. Để giải bài này, rõ ràng ta cần biết những số nào
sẽ có 2k ước dương, sau đó mới tìm số nhỏ nhất trong những số như vậy. Lẽ
tự nhiên là ta nghĩ ngay đến việc phân tích một số nguyên dương n ra thừa
số nguyên tố. Vì cần có công thức cho n tổng quát (khi chưa có thông tin gì

12
về các ước nguyên tố của nó), ta viết

n = Πp pα(p) ,

trong đó α(p) là các số nguyên không âm, đồng thời chỉ có hữu hạn số khác
0. Nếu gọi d(n) là số ước dương của n thì

d(n) = Πp (α(p) + 1).

Như vậy, d(n) là một lũy thừa của 2 khi và chỉ khi với mọi p, tồn tại b(p) ≥ 0
sao cho
α(p) = 2b(p) − 1.

Để có thể nhìn rõ hơn"cấu trúc" của n, ta viết số mũ thành tổng (và có n


dưới dạng tích)

α(p) = 2b(p) − 1 = 1 + 2 + 22 + · · · + 2b(p)−1 ,

b(p)−1 2i
n = Πp Πi=0 p .

Khi đó d(n) = 2k , với k = Σp b(p). Như vậy các số n cần tìm là tích của những
r
số nào đó có dạng p2 , với p nguyên tố và r nguyên không âm. Từ phân tích
r
trên đây ta thấy rằng nếu số p2 có mặt trong biểu diễn n dưới dạng tích, thì
m
những số p2 với m < r cũng có mặt trong tích đó. Nghĩa là nếu một số nào
đó có mặt trong biểu diễn n thì mọi ước số của nó cũng đều tham gia trong
r
biểu diễn. Do đó ta có kết luận sau đây: nếu gọi S là tập hợp các số dạng p2 ,
với p nguyên tố và r nguyên không âm, thì d(n) là một lũy thừa của 2 khi và
chỉ khi n là tích các phần tử thuộc một tập con hữu hạn T của S có tính chất
sau: với mọi t ∈ T , s ∈ S , mà s|t thì s ∈ T. Hơn nữa, nếu d(n) = 2k thì tập
hợp T gồm k phần tử.

13
Dễ thấy rằng, với mọi k nguyên dương, tập hợp Tk gồm k phần tử nhỏ
nhất của S thỏa mãn tính chất trên, suy ra f (2k ) chính là tích các phần tử
thuộc Tk . Từ đó suy ra ngay kết luận của bài toán.
Nhận xét: có thể thấy S = {2, 3, 4, 5, 7, 9, 11, 13, 16, 17, · · · }, suy ra f (2) =
2; f (4) = 2.3 = 6; f (8) = 2.3.4 = 24; f (16) = 2.3.4.5 = 120; f (32) = 2.3.4.5.7 =
840, ...

Biểu diễn số
Có rất nhiều bài toán liên quan đến việc biểu diễn số nguyên (dương) dưới
một dạng nào đó. Nhìn chung, lời giải thường xuất phát từ việc xem xét kỹ
những trường hợp riêng rẽ, đặc biệt là khi các số đang xét tương đối nhỏ.
Ta xét ví dụ sau đây.
Bài toán 8.
Tìm số k nguyên dương lớn nhất có tính chất sau: tập hợp các số nguyên
dương phân hoạch được thành k tập con A1 , A2 , · · · , Ak sao cho với mọi n ≥ 15
, với mọi i ∈ {1, 2, · · · , k} tồn tại hai phần tử khác nhau của Ai có tổng bằng
n.
Phân tích - Lời giải
Ta bắt đầu với trường hợp k = 2. Rõ ràng có thể phân hoạch tập hợp các
số nguyên dương thành hai tập con: A1 = {2n, n ≥ 3} ∪ {1, 2}; A2 = {2n − 1, n ≥
3} ∪ {3, 4} có tính chất: mọi số nguyên dương ≥ 7 biểu diễn được dạng tổng
hai số thuộc A1 và tổng hai số thuộc A2 . Khi k = 3, dĩ nhiên việc phân hoạch
thành "chẵn, lẻ" như trên được thay bởi phân hoạch theo modulo 3, và cũng
như trước, cần thêm vào mỗi lớp đồng dư modulo 3 một số số đầu tiên để
bảo đảm mỗi tập đều biểu diễn được mọi số lớn hơn hoặc bằng 15 dưới dạng
tổng hai số. Có thể chọn phân hoạch sau đây:

A1 = {1, 2, 3} ∪ {3m; m ≥ 4},

14
A2 = {4, 5, 6} ∪ {3m − 1; m ≥ 4},

A3 = {7, 8, 9} ∪ {3m − 2; m ≥ 4}.

Dễ thử lại rằng, phân hoạch trên đây thỏa mãn bài ra. Hơn nữa, có thể nhận
thấy rằng, điều kiện biểu diễn được mọi số ≥ 15 đối với phân hoạch trên đây
đã rất "chặt", chẳng hạn số 14 không thể biểu diễn dạng tổng hai số thuộc
A2 hoặc hai số thuộc A3 . Từ đó có thể dự đoán rằng, k = 3 là giá trị lớn nhất
có thể để tồn tại phân hoạch thỏa mãn bài ra.
Ta sẽ chứng minh dự đoán trên, nghĩa là với k ≥ 4 không thể phân hoạch
tập các số tự nhiên thành k tập hợp con thỏa mãn bài ra.
Rõ ràng nếu với k ≥ 4 nào đó mà tồn tại phân hoạch thỏa mãn, thì phân
hoạch như vậy cũng tồn tại với k = 4: chỉ cần lấy phân hoạch A1 , A2 , A3 , A4 ∪
A5 ∪ · · · ∪ Ak ta được phân hoạch gồm 4 tập hợp thỏa mãn bài ra. Như vậy chỉ
cần chứng minh không thể tồn tại phân hoạch gồm 4 tập hợp con thỏa mãn
bài ra.
Giả sử tồn tại một phân hoạch như vậy: A1 , A2 , A3 , A4 . Như ta đã thấy
trong các ví dụ khi k = 2, 3, các tập hợp Ai phải chứa những số nào đó trong
những số tự nhiên đầu tiên. Xét 10 số nhỏ nhất mà mỗi một tập hợp Ai đều
phải biểu diễn được: 15, 16, ..., 24. Mỗi số trong 10 số này đều là tổng của
hai số nào đó thuộc tập hợp B = {1, 2, · · · , 23}. Như vậy, mỗi tập hợp Ai cần
chứa ít nhất 5 số thuộc B . Do bốn tập Ai rời nhau mà B chỉ có 23 phần tử
nên phải tồn tại tập Aj nào đó chứa đúng 5 số thuộc B, giả sử đó là các số
{x1 , x2 , x3 , x4 , x5 }. Năm số này biểu diễn được đúng 10 số trong các số từ 15
đến 24, tức là 10 số đó chính là 10 tổng có thể {xk + xl , k 6= l; 1 ≤ k,
eq5}. Từ đó suy ra:

15 + 16 + · · · + 24 = 4(x1 + x2 + x3 + x4 + x5 ),

15
vì mỗi số xi tham gia trong đúng 4 cặp số. Đẳng thức trên đây cho ta mâu
thuẫn vì tổng ở vế trái là 195, trong khi vế phải chia hết cho 4.

Bài toán 9.
Với mỗi số nguyên dương m, ký hiệu C(m) là số nguyên dương k lớn nhất
để tồn tại tập hợp S gồm m số nguyên sao cho mỗi số nguyên từ 1 đến k đều
thuộc S , hoặc là tổng của hai số thuộc S (không nhất thiết khác nhau).
Ví dụ : C(3) = 8; S = {1, 3, 4}.
Chứng minh bất đẳng thức sau đây:
m(m + 6) m(m + 3)
≤ C(m) ≤ .
4 2
Phân tích - Lời giải
Điều kiện bài toán gợi cho ta thấy cần phải tính số phần tử của tập hợp
S ∪ (S + S), trong đó
S + S = {x + y|x, y ∈ S}.

Nếu với mỗi tập hợp A, ký hiệu qua |A| số phần tử của nó, thì ta có bất đẳng
thức hiển nhiên sau:

|S ∪ (S + S)| ≤ |S| + |S + S|.

Mỗi phần tử của S + S nhận được bằng cách lấy tổng một cặp số (bằng nhau
hoặc khác nhau) của S , suy ra bất đẳng thức sau:

2 1
|S + S| ≤ |S| + C|S| = |S|(|S| + 1).
2
Trong hai bất đẳng thức cần chứng minh, bất đẳng thức bên phải tương
đối dễ thấy. Số C(m) là số nguyên dương k lớn nhất sao cho tồn tại tập S
gồm m số nguyên dương thỏa mãn

{1, 2, · · · , k} ⊂ S ∪ (S + S).

16
Từ chứng minh trên ta có
1
k ≤ |S ∪ (S + S)| ≤ |S|(|S| + 3).
2

Từ đó suy ra
m(m + 3)
C(m) ≤ .
2
Ta chứng minh bất đẳng thức bên trái. Trước tiên, tập hợp S cần tìm chứa
một số số tự nhiên liên tiếp nào đó 1, 2, · · · , t; t < m. Với những số này, tập
hợp S ∪ (S + S) chứa các số 1, 2, · · · , 2t. Như vậy, để biểu diễn được những số
tiếp theo, cần thêm số 2t + 1. Các số 1, 2, · · · , t, 2t + 1 cho phép biểu diễn các
số tự nhiên cho đến số 3t + 1, vì thế muốn biểu diễn các số lớn hơn, ta cần
thêm số 3t + 2. Lý luận trên đây chỉ ra rằng tập hợp S gồm m phần tử cần
tìm có dạng

{1, 2, · · · , t} ∪ {(k + 1)t + k; k = 1, 2, · · · , m − t.}

Khi đó dễ thấy rằng

{1, 2, · · · , m + (m + 1)t − t2 } ⊂ S ∪ (S + S).

Ta cần tìm giá trị t < m sao cho số các số biểu diễn được là lớn nhất, tức là
tìm t sao cho m+)m + 1)t − t2 đạt cực đại. Dễ chứng mỉnh rằng đại lượng này
m+1 m(m + 6)
đạt cực đại khi t = [ ], và giá trị cực đại là [ ].
2 4
Từ đó ta có bất đẳng thức cần chứng minh.

§4. Trò chơi


Bài toán 10.
Trên bàn có n > 1 que diêm. Có hai người lần lượt lấy diêm. Mỗi người
đến lượt mình được lấy một số que diêm tùy ý trong những que còn lại trên
bàn, nhưng không vượt quá số que diêm mà người đi trước vừa lấy, và người

17
đi đầu tiên không lấy quá n − 1 que. Người nào lấy que diêm cuối cùng được
xem là chiến thắng.
Tìm các số n sao cho người đi trước có chiến lược thắng.
Phân tích -Lời giải.
Dễ thấy rằng nếu n lẻ thì người đi trước luôn luôn thắng, bằng cách ở
nước đi đầu tiên, người đó chỉ lấy một que diêm, do đó ở những nước đi tiếp
theo, mỗi người chỉ được lấy một que diêm.
Xét trường hợp n chẵn. Rõ ràng người nào lấy một số lẻ que diêm đầu
tiên sẽ thua, vì để lại cho người đi nước tiếp theo một số lẻ que diêm: trở
về trường hợp trên. Do đó, người chiến thắng phải luôn lấy một số chẵn que
diêm. Như vậy, có thể hình dung các que diêm được gắn thành từng cặp, và
mỗi người đến lượt sẽ lấy một số cặp nào đó.
1/ Nếu chỉ có một cặp (n = 2): người đi trước thua, vì chỉ được lấy một
que.
2/ Nếu số cặp lẻ và > 1 (n ≡ 2 mod 4): ta trở về trường hợp n lẻ (vì các
que diêm đã được gắn thành cặp), và người đi trước thắng.
3/ Nếu số cặp chẵn (n ≡ 0 mod 4): mỗi người muốn thắng thì luôn phải
lấy một số chẵn cặp (nếu ngược lại thì ta trở về trường hợp 2/). Khi đó có thể
hình dung các que diêm được gắn thành từng nhóm 4 que. Tương tự trường
hợp 1/ và 2/ ta thấy nếu số nhóm là một (n = 4) thì người đi trước thua; nếu
n > 4 và số nhóm lẻ (n ≡ 4 mod 8) thì người đi trước thắng. Nếu số nhóm là
chẵn (n ≡ 0 mod 8), ta lại gắn các que diêm thành từng nhóm 8 que,...
Như vậy, người đi trước có chiến lược thắng khi và chỉ khi n không phải
là một lũy thừa của 2 (n 6= 2k .)
Trên đây là một bài về trò chơi, mà lời giải đơn giản dựa vào việc xét
modulo 2k . Trong phần còn lại của mục này, chúng ta sẽ xem xét một bài
toán trò chơi rất tổng quát. Sở dĩ chúng tôi đưa ra ví dụ này vì lời giải của nó

18
hội tụ đầy đủ những vấn đề thường gặp trong bài toán về trò chơi: tập hợp
có chiến lược thắng, đồ thị, số học. Hơn nữa, chúng ta lại một lần nữa tìm
thấy sự tham gia của "tỷ số vàng"

Bài toán 11.


Cho hai đống đá, một đống có a hòn đá, đống kia có b hòn. Hai người
chơi, mỗi người đến lượt mình được lấy một số tùy ý hòn đá từ một trong
hai đống, hoặc lấy từ hai đống số hòn đá như nhau. Người lấy được hòn cuối
cùng là người chiến thắng. Tìm tất cả các cặp (a, b) sao cho người đi sau có
chiến lược thắng. (Ví dụ: (1,2) là cặp có tính chất đó).
Phân tích - Lời giải.
Việc cho phép lấy hoặc một số đá tùy ý từ một trong hai đống, hoặc lấy số
đá như nhau từ mỗi đống gợi cho ta hình ảnh: hoặc giảm theo chiều ngang,
theo chiều dọc, hoặc theo đường chéo. Để mô tả tình hình đó, tốt nhất là
dùng một đồ thị có hướng.
Xét đồ thị mà các đỉnh là các điểm trên mặt phẳng với tọa độ nguyên
không âm, các cạnh là những đoạn nối hai điểm, nằm trên những đường song
song các trục tọa độ và trên những đường song song với đường thẳng y = x.
Các cạnh có hướng từ phải sang trái, từ trên xuống dưới. Với bài toán có hai
đống đá với số đá là a, b, ta có một đồ thị có hướng mà điểm xuất phát là
điểm có tọa độ (a, b). Trong quá trình chơi, mỗi người đến lượt mình sẽ đi
theo một trong ba hướng (phải-trái, trên-dưới hoặc giảm theo đường chéo)
đến một đỉnh mới của đồ thị. Người chiến thắng là người đến điểm (0,0) đầu
tiên. Như vậy, một cặp (a, b) thỏa mãn bài ra ứng với một vị trí thắng theo
định nghĩa sau:
Định nghĩa Ta gọi điểm (a, b) là một vị trí thắng nếu người chơi nào đến
được vị trí đó thì họ sẽ có chiến lược thắng, không phụ thuộc bước tiếp theo

19
của dối thủ.
Như vậy, bài toán đặt ra là tìm tập hợp các vị trí thắng. Trước tiên ta làm
quen một khái niệm trong trò chơi.
Định nghĩa. Giả sử R là một tập hợp những vị trí nào đó. Khi đó ta nói
R ổn định trong nếu mỗi bước (theo quy tắc của trò chơi) xuất phát từ một
điểm thuộc R sẽ có đích đến là một điểm không thuộc R. Tập hợp R gọi là
ổn định ngoài nếu từ vị trí tùy ý không thuộc R, tồn tại một bước đi mà đích
đến thuộc R.
Tập hợp R vừa ổn định trong, vừa ổn định ngoài được gọi là một lời giải.
Rõ ràng (a, b) là một vị trí thắng nếu và chỉ nếu (a, b) thuộc một tập lời
giải nào đó chứa điểm (0,0). Do đó, bài toán đòi hỏi tìm các tập lời giải chứa
điểm (0,0).
Bổ đề Tồn tại không quá một lời giải chứa điểm (0, 0).
Chứng minh. Giả sử R, S là hai lời giải khác nhau chứa điểm (0,0). Lấy
điểm s1 ∈ S; s1 6∈ R. Từ s1 , tồn tại bước đi đến r1 ∈ R, và r1 6∈ S. Khi đó
lại tồn tại bước đi từ r1 đến s2 ∈ S, s2 6∈ R. Lặp lại quá trình, ta được dãy
s1 , r1 , s2 , r2 , · · · , kết thúc tại (0,0), mà mỗi vị trí trong dãy chỉ thuộc một trong
hai tập hợp R; S.. Suy ra điểm (0,0) chỉ thuộc một trong hai tập hợp R; S :
mâu thuẫn.
Như vậy, ta đã chứng minh được tính duy nhất của tập hợp lời giải. Vấn
đề còn lại là chỉ ra một tập hợp có tính chất đòi hỏi. Quan sát trên đồ thị
dẫn ta đến Bổ đề sau:
Bổ đề. Giả sử R là một tập hợp vị trí có các tính chất sau:
1/ (0, 0) ∈ R.
2/ Nếu (a, b) ∈ R thì (b, a) ∈ R.
3/ Với mọi a ∈ N∗ tồn tại duy nhất b ∈ N∗ sao cho (a, b) ∈ R.
4/ Với mọi d ∈ N∗ tồn tại duy nhất (a, b) ∈ R sao cho a − b = d.

20
5/ Nếu (a, b), (k, l) ∈ R, a < b, k < l, b − a < l − k thì a < k, b < l.
Khi đó R là một tập hợp lời giải.
Chứng minh. Từ điều kiện 3/ suy ra rằng mọi số tự nhiên là tọa độ của
một cặp đối xứng (tính chất 2/) các vị trí thuộc R. Ta chứng minh tính ổn
định trong và ổn định ngoài của R.
a/ Tính ổn định trong. Giả sử (a, b) ∈ R. Theo tính chất 3/ nếu ta giảm a
hoặc b thì sẽ đến một vị trí không thuộc R. Nếu giảm a và b với cùng một đại
lượng, thì theo tính chất 4/, ta cũng đến một vị trí ngoài R.
b/ Tính ổn định ngoài. Giả sử (a, b) 6∈ R. Nếu a = b, ta có ngay một bước
đi đến điểm (0, 0) ∈ R. Giả sử a 6= b. Theo tính chất 3/ tồn tại c sao cho
(a, c) ∈ R. Theo tính chất 4/, tồn tại k, l sao cho l − k = b − a, (k, l) ∈ R. Khi
đó nếu c < b thì tồn tại bước đi từ (a, b) đến (a, c) (giảm b). Nếu c > b thì
c − a > b − a = l − k nên theo tính chất 5/ , c > l; a > k : khi đó ta giảm đồng
thời a, b một đại lượng bằng a − k = b − l, và đi đến vị trí (k, l) ∈ R..
Như vậy để giải bài toán chỉ cần tìm một tập hợp vị trí thỏa mãn tính
chất 1/- 5/. Tập hợp như thế ta đã từng gặp trong Bài toán 2.
Thật vậy, xuất phát từ điểm (0, 0), ta xây dựng dãy an , bn , n ≥ 1 bằng
quy nạp như sau: giả sử đã có (a1 , b1 ); (a2 , b2 ), · · · , (an , bn ). Khi đó lấy an+1 là
số nguyên dương nhỏ nhất chưa xuất hiện trong hai dãy an , bn trước đó, đồng
thời lấy bn+1 = an+1 + (n + 1).
Dễ chứng minh rằng, tập hợp R = {(0, 0), (an , bn ), n = 1, 2, · · · là tập hợp
thỏa mãn các điều kiện 1/-5/, và do đó là tập hợp các vị trí thắng.
Vậy tập hợp cần tìm là
√ √ √ √
1+ 5 3+ 5 3+ 5 1+ 5
{(0, 0); ( n; n); ( n; n), n = 1, 2, 3, · · · }.
2 2 2 2
Ví dụ: (0, 0); (1, 2); (3, 5); (4, 7); (6, 10); ...
(2, 1); (5, 3); (7, 4) ; (10, 6)...

21
Lớp các phương trình hàm Cauchy, d’Alembert

và dạng toán liên quan

Nguyễn Văn Mậu


Trường Đại học KHTN, ĐHQGHN

Trong lý thuyết các phương trình hàm thì hai dạng toán cơ bản nhất là
phương trình hàm Cauchy và phương trình hàm d’Alembert. Chúng đóng vai
trò nòng cốt (về phương pháp luận và phương pháp giải) để giải quyết lớp các
khác nhau về xác định hàm số trong đại số và trong lượng giác tương ứng.
Hầu hết, các đề toán về phương trình hàm trong các kỳ thi Olympic quốc tế
và khu vực thường tập trung khai thác các tính chất giải tích của hàm số như
tính giải tích, tính khả vi, liên tục, khả tích, giới nội, đo được,... Khi không
viện trợ đến một trong các tính chất giải tích đó thì việc giải các phương trình
hàm thường chỉ thực hiện được trên tập rời rạc (Z, N, Q,...), còn trên tập R
thì gặp muôn vàn khó khăn, chưa có hướng giải quyết tổng thể. Vì thế, việc
xem xét các phương trình hàm cộng tính Cauchy trên R trong lớp hàm không
liên tục sẽ gắn với tính phi tuyến (mạnh) và rất khó hình dung ra dáng điệu
đồ thị của chúng trên mặt phẳng.
Bài viết này nhằm giới thiệu và phân tích toàn cảnh bức tranh phức hợp
đó.

22
1 Hàm cộng tính và song cộng tính
Trong mục này sẽ giới thiệu tổng quan về lớp phương trình hàm Cauchy
(xem [1]-[8]). Có lẽ A.M. Legendre là người đầu tiên đã tìm được nghiệm của
phương trình hàm Cauchy (hàm cộng tính)

f (x + y) = f (x) + f (y), ∀x, y ∈ R.

Trong cuốn sách của mình, Kuczma (1985) đã trình bày rất chi tiết các tính
chất của lớp hàm cộng tính. Tiếp theo, lớp hàm này được đề cập nhiều trong
các cuốn sách của Acze’l (1966, 1987), Acze’l và Dhombres (1989), và Smital
(1988).
Dựa vào nghiệm của phương trình hàm Cauchy (hàm cộng tính), lớp
nghiệm tổng quát của nhiều phương trình hàm (hai hay nhiều biến) thông
qua lớp các hàm cộng tính, lũy thừa, logarit hay mũ... có thể biểu diễn thông
qua chúng.

1.1 Về lớp hàm cộng tính liên tục

Trong mục này, ta định nghĩa hàm cộng tính và khảo sát dáng điệu của
chúng dựa theo các giả thiết về tính trơn khác nhau chẳng hạn như tính đo
được, tính liên tục, tính khả vi, tính đơn điệu, tính lồi, lõm, ...

Định nghĩa 1.1. Một hàm f : R → R được gọi là một hàm cộng tính khi và
chỉ nó thỏa phương trình hàm Cauchy

f (x + y) = f (x) + f (y), ∀x, y ∈ R. (1)

Phương trình hàm (1) được xét đầu tiên bởi A.M. Legendre (1791) và
C.F. Gauss (1809) nhưng A.L. Cauchy (1821) mới là người đầu tiên tìm thấy

23
nghiệm tổng quát của nó. Phương trình (1) có một vị trí đặc biệt trong toán
học. Nó được gặp ở hầu hết tất cả các ngành học của toán như là sự khởi đầu
của các phép tính đối với hàm số.

Định nghĩa 1.2. Một hàm f : R → R được gọi là tuyến tính khi và chỉ khi
nó có dạng
f (x) = ax ∀x ∈ R,

với a là hằng số tùy ý.

Đồ thị của một hàm tuyến tính f (x) = ax là một đường thẳng (không
thẳng đứng) đi qua gốc tọa độ và do đó nó được gọi là tuyến tính. Câu hỏi
từ rất lâu đã được đặt ra là ngoài hàm tuyến tính trên thì còn có hàm cộng
tính nào khác nữa không?
Ta dễ dàng chỉ ra rằng chỉ có các hàm cộng tính liên tục mới là tuyến tính.
Điều này đã được Cauchy khẳng định vào năm (1821).

Định lý 1.1. Cho f : R → R là một hàm cộng tính liên tục. Thế thì f là
tuyến tính, nghĩa là f (x) = ax ở đây a là một hằng số tùy ý.

Để ý rằng, tính liên tục của f suy ra nó cũng là hàm khả tích. Chính tính
khả tích của f đã làm cho hàm cộng tính f trở thành tuyến tính. Do đó mọi
hàm cộng tính khả tích cũng là tuyến tính.

Định nghĩa 1.3. Hàm f : R → R được gọi là khả tích địa phương khi và chỉ
khi nó là khả tích trên mọi đoạn hữu hạn.

Ta có kết luận rằng mọi ánh xạ cộng tính khả tích địa phương cũng là
tuyến tính. Ta nêu một chứng minh ngắn của Shapiro (1973). Giả thiết f
là hàm cộng tính khả tích địa phương. Từ đó f (x + y) = f (x) + f (y) đúng

24
∀x, y ∈ R. Từ điều này và sử dụng tính khả tích địa phương của f , ta có

Ry Ry
yf (x) = f (x)dz = [f (x + z) − f (z)] dz
0 0
x+y
R Ry x+y
R Rx Ry
= f (u)du − f (z)dz = f (u)du − f (u)du − f (u)du.
x 0 0 0 0

Vế phải của phương trình trên là bất biến khi hoán đổi x và y , nghĩa là
f (x)
yf (x) = xf (y), ∀x ∈ R, nên với x 6= 0, ta thu được x = a, với a là hằng số
tùy ý.
Từ đây suy ra f (x) = ax, ∀x ∈ R \ {0}. Vì f là cộng tính, ta biết rằng
f (0) = 0. Kết hợp hai điều trên, ta kết luận f là hàm tuyến tính trong R.

Mặc dù chứng minh trên là rất vắn tắt và được suy ra bằng cách chỉ vận
dụng các tính toán thông thường, nhưng nó chưa làm sáng tỏ vấn đề liên
quan giữa tính cộng tính và tính tuyến tính.
Tiếp theo, ta sẽ trình bày một chứng minh khác giúp ta hiểu dáng điệu
của hàm cộng tính nhiều hơn. Trước tiên, ta bắt đầu với định nghĩa sau đây.

Định nghĩa 1.4. Hàm f : R → R được gọi là thuần nhất hữu tỉ khi và chỉ
khi
f (rx) = rf (x), ∀x ∈ R, r ∈ Q. (2)

Định lý dưới đây chứng tỏ rằng hàm cộng tính là thuần nhất hữu tỉ.

Định lý 1.2. Mọi hàm f : R → R cộng tính là thuần nhất hữu tỉ. Hơn nữa,
f là tuyến tính trên tập số hữu tỉ Q.

Chứng minh. Cho x = 0 = y ở (1) ta thấy rằng f (0) = 0. Thế y = −x ở (1)


và sử dụng f (0) = 0, ta thấy rằng f là một hàm lẻ trong R, nghĩa là

f (−x) = −f (x), ∀x ∈ R. (3)

25
Tiếp theo, ta sẽ chỉ ra rằng một hàm cộng tính là thuần nhất hữu tỉ. Với x
tùy ý,
f (2x) = f (x + x) = f (x) + f (x) = 2f (x).

Từ đó
f (3x) = f (2x + x) = f (2x) + f (x) = 2f (x) + f (x) = 3f (x);

vì vậy, trường hợp tổng quát (sử dụng phép quy nạp)

f (nx) = nf (x) (4)

với mọi n nguyên dương. Nếu n nguyên âm, thì −n là một số nguyên dương
và bởi (4) và (3) , ta có

f (nx) = f (−(−n)x) = nf (x).

Như vậy, ta đã chứng tỏ f (nx) = nf (x) với mọi số nguyên n và mọi x ∈ R.


k
Tiếp theo, với r là một số hữu tỉ tùy ý, ta có r = , với k là một số nguyên và
l
l là một số tự nhiên. Ngoài ra, kx = l(rx). Sử dụng tính nguyên thuần nhất
của f , ta có được
kf (x) = f (kx) = f (l(rx)) = lf (rx)

nghĩa là f (rx) = kl f (x) = rf (x).


Do đó, f là thuần nhất hữu tỉ. Ngoài ra, cho x = 1 trong phương trình
trên và đặt a = f (1), ta thấy rằng

f (r) = ar, ∀r ∈ Q.

Từ đó, f là tuyến tính trên tập các số hữu tỉ.

Bây giờ ta trình bày cách chứng minh thứ hai của Định lí 1.2. Giả sử f là
hàm cộng tính và liên tục trên tập số thực. Với số thực tùy ý x thì luôn tồn

26
tại một dãy {rn } các số hữu tỉ với rn → x. Do f là cộng tính, theo Định lí 1.2,
f là tuyến tính trên tập số hữu tỉ. Nghĩa là

f (rn ) = arn , ∀n ∈ N.

Sử dụng tính liên tục của f, ta có


   
f (x) = f lim rn = lim f rn = lim arn = ax.
n→∞ n→∞ n→∞

Dễ thấy rằng

Định lý 1.3. Hàm cộng tính liên tục tại một điểm, thì nó liên tục tại mọi
điểm.

1.2 Lớp hàm cộng tính không liên tục

Ta đã chứng tỏ rằng lớp hàm cộng tính liên tục là tuyến tính. Thậm chí
nếu ta làm yếu điều kiện liên tục thành liên tục tại một điểm, lớp hàm cộng
tính vẫn là tuyến tính. Từ lâu, sự tồn tại của lớp hàm cộng tính gián đoạn
vẫn còn là một vấn đề mở. Các nhà toán học không chứng minh được rằng
mọi hàm cộng tính là liên tục hay chỉ ra một phản ví dụ về một hàm cộng
tính gián đoạn. Năm 1905, nhà toán học Đức G. Hamel, người đầu tiên đã
thành công trong việc chứng minh rằng tồn tại các hàm cộng tính gián đoạn.

Bây giờ ta bắt đầu tìm hiểu về lớp hàm cộng tính phi tuyến (gián đoạn).
Trước tiên, ta chỉ ra rằng lớp hàm cộng tính phi tuyến phô diễn một dáng
điệu rất kì lạ.

Định nghĩa 1.5. Ta gọi đồ thị của một hàm f : R → R là tập

G = {(x, y)|x ∈ R, y = f (x)} .

27
Dễ dàng thấy rằng đồ thị của hàm f : R → R là tập con của không gian
R2 .

Định lý 1.4. Đồ thị của mọi hàm cộng tính phi tuyến f : R → R là trù mật
khắp nơi trong không gian R2 .

Chứng minh. Đồ thị G của hàm f được cho bởi

G = {(x, y)|x ∈ R, y = f (x)} .

Chọn một số khác không x1 trong R. Từ f là một ánh xạ cộng tính phi tuyến,
f (x1 ) f (x2 )
với mọi hằng số m, luôn tồn tại số thực x2 6= 0 sao cho 6= , theo
x1 x2
f (x1 )
cách khác viết m = và giả sử x1 = x, ta sẽ có f (x) = mx với mọi x 6= 0,
x1
và từ f (0) = 0 điều này kéo theo f là tuyến tính trái với giả thiết của ta rằng
f là phi tuyến. Suy ra
x1 f (x1 )
6= 0,
x2 f (x2 )

vì vậy các véctơ X1 = (x1 , f (x1 )) và X2 = (x2 , f (x2 )) là độc lập tuyến tính và
vì vậy chúng trải rộng toàn bộ không gian R2 . Điều này có nghĩa rằng với
mọi véctơ X = (x, f (x)) tồn tại các số thực r1 và r2 sao cho X = r1 X1 + r2 X2 .
Nếu ta cố định các số hữu tỉ ρ1 , ρ2 thì bằng phép chọn x1 , x2 thích hợp, ta có
ρ1 X1 + ρ2 X2 là tập đóng trong không gian véctơ X do tập hợp số hữu tỉ Q là
trù mật trong tập số thực R và do đó Q2 là trù mật trong R2 . Vậy thì

ρ1 X1 + ρ2 X2 = ρ1 (x1 , f (x1 )) + ρ2 (x2 , f (x2 ))


= (ρ1 x1 + ρ2 x2 , ρ1 f (x1 ) + ρ2 f (x2 ))
= (ρ1 x1 + ρ2 x2 , f (ρ1 x1 + ρ2 x2 )).

Vì vậy, tập

b = {(x, y)|x = ρ1 x1 + rho2 x2 , y = f (ρ1 x1 + rho2 x2 ), ρ1 , ρ2 ∈ Q}


G

28
là trù mật khắp nơi trong R2 . Từ đó G
b ⊂ G, đồ thị G của hàm cộng tính phi
tuyến f cũng trù mật trong R2 .

Đồ thị của một hàm liên tục cộng tính là một đường thẳng đi qua gốc tọa
độ. Đồ thị của một hàm cộng tính phi tuyến là trù mật trong không gian R2 .
Tiếp theo, ta làm quen với khái niệm cơ sỏ Hamel để xây dựng một hàm
cộng tính gián đoạn. Xét tập hợp
√ √
S = {x ∈ R|s = u + v 2 + w 3, u, v, w ∈ Q, }
√ √
tập mà các phần tử là tổ hợp tuyến tính hữu tỉ của 1, 2, 3. Hơn nữa tập
sinh bởi tổ hợp hữu tỉ này là duy nhất. Nghĩa là nếu một phần tử s ∈ S có
hai tổ hợp tuyến tính hữu tỉ khác nhau, chẳng hạn,
√ √ √ √
s = u + v 2 + w 3 = u0 + v 0 2 + w 0 3

thì u = u0 , v = v 0 và w = w0 . Để chứng tỏ điều này, ta chú ý rằng giả thiết này


kéo theo
√ √
(u − u0 ) + (v − v 0 ) 2 + (w + w0 ) 3 = 0.

Đặt a = (u − u0 ), b = (v − v 0 ) và c = w − w0 , ta thấy rằng biểu thức trên quy về


√ √
a + b 2 + c 3 = 0.

Tiếp theo, ta chỉ ra rằng a = 0 = b = c. Biểu thức trên cho


√ √
b 2 + c 3 = −a

và bình phương hai vế, ta có



2bc 6 = a2 − 2b2 − 3c2 .

Từ đây suy ra b hoặc c là 0; Trường hợp khác, ta chia hai vế cho 2bc và có
√ a2 − 2b2 − 3c2
6=
2bc

29
√ √
cho ta sự mâu thuẫn rằng 6 là một số vô tỉ. Nếu b = 0, thì ta có a + c 3 = 0;

điều này kéo theo c = 0 (nếu ngược lại thì 3 = − ac , là một số hữu tỉ trái với

thực tế rằng 3 là một số vô tỉ). Tương tự nếu c = 0, ta được b = 0. Như vậy
cả b và c đều bằng không. Từ đó lập tức có được a = 0. Nếu đặt
 √ √
B = 1, 2, 3

thì mỗi phần tử của S là một tổ hợp tuyến tính duy nhất của các phần tử
của B . Tập B được gọi là một cơ sở Hamel đối với tập S . Về mặt hình thức,
một cơ sở Hamel cũng được định nghĩa tương tự.

Định nghĩa 1.6. Giả sử S là tập các số thực và B là một tập con của S .
Thế thì B được gọi là một cơ sở Hamel của S nếu mọi phần tử của S là một
tổ hợp tuyến tính hữu tỉ (hữu hạn) duy nhất của B .

Nếu S là tập các số thực, thì sử dụng tiên đề chọn ( hoặc bằng phép quy
nạp siêu hạn) có thể chỉ ra rằng một cơ sở Hamel B đối với R tồn tại.

Nhận xét rằng, có một mối liên hệ chặt chẽ giữa các hàm cộng tính và cơ
sở Hamel. Để diễn tả một hàm cộng tính thì chỉ cần cho các giá trị trên một
cơ sở Hamel là đủ, và các giá trị đó có thể phân bố tùy ý. Điều này là nội
dung của hai định lí tiếp theo.

Định lý 1.5. Giả sử B là một cơ sở Hamel đối với R. Nếu hai hàm cộng tính
có giá trị trùng nhau tại mỗi phần tử của B , thì chúng bằng nhau.

Chứng minh. Giả sử f1 và f2 là hai hàm cộng tính có giá trị trùng nhau
tại mỗi phần tử của B . Thế thì f1 − f2 là cộng tính. Ta kí hiệu f = f1 − f2 .
Giả sử x là số thực tùy ý. Thế thì tồn tại các số b1 , b2 , . . . bn trong B và các số
hữu tỉ r1 , r2 , . . . rn sao cho

x = r1 b1 + r2 b2 + · · · + rn bn .

30
Từ đó

f1 (x) − f2 (x) = f (x) = f (r1 b1 + r2 b2 + · · · + rn bn )


= f (r1 b1 ) + f (r2 b2 ) + · · · + f (rn bn ) = r1 f (b1 ) + r2 f (b2 ) + · · · + rn f (bn )
= r1 [f1 (b1 ) − f2 (b1 )] + r2 [f1 (b2 ) − f2 (b2 )] + · · · + rn [f1 (bn ) − f2 (bn )] = 0

Như vậy, ta có f1 = f2 .

Định lý 1.6. Giả sử B là một cơ sở Hamel đối với R. Giả sử g : B → R là


một hàm tùy ý xác định trên B. Thế thì tồn tại một hàm cộng tính f : R → R
sao cho f (b) = g(b) với mọi b ∈ B.

Chứng minh. Ứng với mỗi số thực x đều có thể tìm được b1 , b2 , . . . , bn trong
B và các số hữu tỉ r1 , r2 , . . . rn sao cho

x = r1 b1 + r2 b2 + · · · + rn bn .

Việc xác định f (x) chính là biểu thức

r1 g(b1 ) + r2 g(b2 ) + · · · + rn g(bn ).

Biểu thức này xác định f (x) với mọi x một cách duy nhất, đối với mỗi x. Việc
chọn b1 , b2 , . . . , bn , r1 , r2 , . . . rn là duy nhất, không kể đến thứ tự các số bi và ri
được chọn. Đối với mỗi b ∈ B , ta có f (b) = g(b) bởi cách xác định của f . Tiếp
theo, ta sẽ chỉ ra rằng f là cộng tính trên số thực. Giả sử x và y là hai số thực
tùy ý. Thế thì
x = r1 a1 + r2 a2 + · · · + rn an

y = s 1 b1 + s 2 b2 + · · · + s m bm ,

với r1 , r2 , . . . rn , s1 , s2 , . . . sm là các số hữu tỉ và a1 , a2 , . . . , an , b1 , b2 , . . . , bm là các


phần tư của cơ sở Hamel B. Hai tập {a1 , a2 , . . . , an } và {b1 , b2 , . . . , bm } có thể

31
có nhừng phần tử chung. Giả sử hợp của hai tập đó là {c1 , c2 , . . . , cl }. Thế thì
l ≤ m + n, và
x = u1 c1 + u2 c2 + · · · + ul cl

y = v1 c1 + v2 c2 + · · · + vl cl ,

ở đây u1 , u2 , . . . ul , v1 , v2 , . . . vl là các số hữu tỉ, và không đồng thời bằng không.


Xét
x + y = (u1 + v1 )c1 + (u2 + v2 )c2 + · · · + (ul + vl )cl


f (x + y) = f ((u1 + v1 )c1 + (u2 + v2 )c2 + · · · + (ul + vl )cl )
= (u1 + v1 )g(c1 ) + (u2 + v2 )g(c2 ) + · · · + (ul + vl )g(cl )
= [u1 g(c1 ) + u2 g(c2 ) + · · · + ul g(cl )]
+ [v1 g(c1 v2 g(c2 ) + · · · + vl g(cl )] = f (x) + f (y)

Do đó f là cộng tính trên tập các số thực R. Đây là điều cần chứng minh.

Tiếp theo ta xây dựng một hàm cộng tính phi tuyến. Giả sử B là cơ sở
Hamel đối với tập các số thực R. Giả sử b ∈ B là phần tử tùy ý của B . Đặt
(
0 nếu x ∈ B \ {b}
g(x) =
1 nếu x = b.

Theo định lí 1.6 thì tồn tại một hàm cộng tính f : R → R với f (x) = g(x) đối
với mỗi x ∈ B . Chú ý rằng f ở đây có thể không tuyến tính đối với x ∈ B và
x 6= b, ta có
f (x) f (b)
0= 6= .
x b
Do đó f là một hàm cộng tính phi tuyến.

Chú ý rằng, hiện tại vẫn không có ví dụ cụ thể nào về một cơ sở Hamel,
ta chỉ biết rằng nó tồn tại. Đồ thị của một hàm cộng tính gián đoạn là không
dễ để vẽ khi biết tập {f (x)|x ∈ R} là trù mật trong R.

32
1.3 Về một tiêu chuẩn nhận dạng tính tuyến tính

Ta đã biết rằng, đồ thị của một hàm cộng tính phi tuyến f là trù mất
trong mặt phẳng. Nghĩa là, mọi hình tròn đều chứa một điểm (x, y) sao cho
y = f (x). Ta cũng thấy rằng một hàm cộng tính f trở thành tuyến tính khi
nó liên tục. Có thể làm yếu điều kiện liên tục trên R thành liên tục tại một
điểm mà vẫn bảo toàn tính tuyến tính của f .
Tiếp theo, ta trình bày một số điều kiện tựa chính quy để một hàm cộng
tính trở thành tuyến tính.

Định lý 1.7. Hàm cộng tính bị chặn một phía hoặc đơn điệu là tuyến tính.

Chứng minh. Giả sử f không tuyến tính. Thế thì đồ thị của f là trù mật
trong mặt phẳng. Vì f bị chặn trên, đối với hàng số M hàm cộng tính f thoả
mãn f (x) ≤ M, x ∈ R. Do đó nó không thể trù mật trên mặt phẳng, điều này
là mâu thuẫn. Phần còn lại của định lí dễ kiểm chứng bằng phương pháp đơn
giản.

Định nghĩa 1.7. Hàm f được gọi là nhân tính khi và chỉ khi f (xy) = f (x)f (y)
với mọi x và y .

Định lý 1.8. Một hàm f vừa cộng tính và nhân tính thì nó là tuyến tính.

2 Hàm cộng tính với cặp biến thực và biến phức

Trong mục này, đầu tiên ta trình bày một số tính chất liên quan đến hàm
cộng tính trên mặt phẳng R2 và sau đó tìm hiểu lớp hàm cộng tính nhận giá
trị phức trên mặt phẳng phức.

33
Định lý 2.1. Nếu f : R2 → R là cộng tính trên mặt phẳng R2 thì luôn tồn
tại các hàm cộng tính A1 , A2 : R → R sao cho

f (x1 , x2 ) = A1 (x1 ) + A2 (x2 ), ∀x1 , x2 ∈ R. (5)

Chứng minh. Giả sử x = (x1 , x2 ) và y = (y1 , y2 ) là hai điểm tùy ý trong mặt
phẳng. Tính cộng tính của f cho ta f (x + y) = f (x) + f (y), nghĩa là

f (x1 + y1 , x2 + y2 ) = f (x1 , x2 ) + f (y1 , y2 ).

Đặt A1 (x1 ) = f (x1 , 0) và A2 (x2 ) = f (0, x2 ) và kiểm tra các hàm A1 , A2 là cộng
tính. Điều này suy ra từ

A1 (x1 + y1 ) = f (x1 + y1 , 0) = f (x1 + y1 , 0 + 0)


= f (x1 , 0) + f (y1 , 0) = A1 (x1 ) + A1 (y1 ).

Từ đó A1 là cộng tính trên R. Tương tự, A2 là cộng tính trên R. Tiếp theo,
chú ý rằng (x1 , x2 ) = (x1 , 0) + (x2 , 0) nên

f (x1 , x2 ) = f (x1 , 0) + f (0, x2 ) = A1 (x1 ) + A2 (x2 ).

Định lí sau đây suy từ kết quả trên.

Định lý 2.2. Nếu f : R2 → R là một hàm cộng tính liên tục trên mặt phẳng
R2 thì tồn tại các hằng số c1 , c2 sao cho

f (x1 , x2 ) = c1 x1 + c2 x2 , ∀x1 , x2 ∈ R. (6)

Kết quả này có thể được làm mạnh hơn bằng việc làm yếu giả thiết về
tính liên tục của f : R2 → R.

Bổ đề 2.1. Nếu một hàm cộng tính f : R2 → R là liên tục theo từng biến thì
nó là liên tục điểm.

34
Chứng minh. Vì hàm f : R2 → R là cộng tính, nên ta có f (x, y) = A1 (x) +
A2 (y) với mọi x, y ∈ R. Vì f tuyến tính theo từng biến, nên A1 , A2 là liên tục.
Do đó
lim A1 (x) = A + 1(x0 ) và lim A2 (y) = A2 (y0 ).
x→x0 y→y0

Để chỉ ra f là liên tục điểm, ta tính


lim f (x, y) = lim [A1 (x) + A2 (y)]
(x,y)→(x0 ,y0 ) (x,y)→(x0 ,y0 )
= lim A1 (x) + lim A2 (y)
x→x0 y→y0
= A1 (x0 ) + A2 (y0 ) = f (x0 , y0 ).

Từ đây chỉ ra rằng f là liên tục điểm. Định lí được chứng minh. Theo kết quả

này, có thể thay tính tuyến tính ở các hàm cộng tính giá trị thực trên mặt
phẳng bằng giả thiết tính liên tục theo từng biến. Ngoài ra có thể mở rộng
Định lí 2.1 đối với các hàm cộng tính trên Rn .

Định lý 2.3. Nếu f : Rn → R là một hàm cộng tính liên tục trên Rn , thì tồn
tại các hằng số c1 , c2 , . . . , cn sao cho

f (x1 , x2 , . . . , xn ) = c1 x1 + c2 x2 + · · · + cn xn ∀x1 , x2 , . . . , xn ∈ R. (7)

Trong phần còn lại của mục này, ta khảo sát lớp hàm cộng tính giá trị
phức trên mặt phẳng phức. Một hàm f : C → C có thể được viết

f (z) = f1 (z) + if2 (z), (8)

với f1 : C → R và f2 : C → R được cho bởi

f1 (z) = Re (z), và f2 (z) = Im f (z). (9)

Nếu f là cộng tính, thì theo (8) và (9) ta có

f1 (z1 + z2 ) = Re f (z1 + z2 ) = Re [f (z1 ) + f (z2 )]


= Re f (z1 ) + Re f (z2 ) = f1 (z1 ) + f1 (z2 ),

35

f2 (z1 + z2 ) = Im f (z1 + z2 ) = Im [f (z1 ) + f (z2 )]
= Im f (z1 ) + Im f (z2 ) = f2 (z1 ) + f2 (z2 ).

Định lý 2.4. Giả sử f : C → C là cộng tính, khi đó tồn tại hàm cộng tính
fkj : R → R (k, j = 1, 2) sao cho

f (z) = f11 (Re z) + f12 (Im z) + if21 (Re z) + if22 (Im z)

Định lí tiếp theo liên quan đến dạng của hàm cộng tính liên tục giá trị
phức trên mặt phẳng phức.

Định lý 2.5. Giả sử f : C → C là hàm cộng tính liên tục, khi đó tồn tại các
hằng số phức c1 , c2 sao cho

f (z) = c1 z + c2 z (10)

ở đây z kí hiệu số phức liên hợp của z .

Chú ý rằng không giống như các hàm cộng tính liên tục nhận giá trị thực
trên số thực, các hàm cộng tính liên tục nhận giá trị phức trên mặt phẳng
phức là không tuyến tính. Tính tuyến tính có thể được phục hồi nếu có thêm
một giả thiết về tính trơn mạnh hơn giống như tính giải tích thay thế cho
tính liên tục.

Định lý 2.6. Nếu f : C → C là một hàm cộng tính giải tích thì tồn tại hằng
số phức c sao cho
f (z) = cz,

hay f là tuyến tính.

Chứng minh. Vì f là giải tích nên f khả vi. Lấy vi phân

f (z1 + z2 ) = f (z1 ) + f (z2 ) (11)

36
theo biến z1 , ta thu được f 0 (z1 + z2 ) = f 0 (z1 ), ∀z1 , z2 ∈ C. Từ đó, chọn z1 = 0
và z2 = z , ta thu được f 0 (z) = c, ở đây c = f 0 (0) là một hằng số phức. Từ trên,
ta có f (z) = cz + b, với b là một hằng số phức. Thay dạng này của f (z) vào
(11), ta thu được b = 0.

3 Hàm song cộng tính


Trong mục này ta khảo sát lớp các hàm song cộng tính. Ta bắt đầu với
định nghĩa dưới đây về hàm song cộng tính.

Định nghĩa 3.1. Một hàm f : R2 → R được gọi là song cộng tính khi và chỉ
khi nó là tuyến tính với mỗi biến, nghĩa là

f (x + y, z) = f (x, z) + f (y, z) (12)

f (x, y + z) = f (x, y) + f (x, z) (13)

với mọi x, y, z ∈ R.

Nếu m là một hằng số và ta định nghĩa f bởi f (x, y) = mxy, x, y ∈ R, thì


f là song cộng tính.

Câu hỏi tự nhiên nảy sinh là liệu có còn các hàm song cộng tính khác hay
không?

Định lý 3.1. Mọi ánh xạ song tuyến tính liên tục f : R2 → R đều có dạng
f (x, y) = mxy, với mọi x, y ∈ R và m là hằng số thực tùy ý.

Chứng minh. Giả sử f : R2 → R là một ánh xạ song tuyến tính liên tục.
Do đó f thỏa mãn điều kiện

f (x + y, z) = f (x, z) + f (y, z) (14)

37
với mọi x, y, z ∈ R.. Cho x = 0, y = 0 ở phương trình trên, ta có

f (0, z) = 0 (15)

với mọi z ∈ R. Cố định z, và đặt ϕ(x) = f (x, z), ta thấy rằng phương trình
(14) quy về
ϕ(x + y) = ϕ(x) + ϕ(y). (16)

Do f liên tục nên ϕ cũng liên tục và do đó theo (16), ϕ tuyến tính, nghĩa là
ϕ(x) = kx và
f (x, y) = k(y)x (17)

ở đây k : R → R là một hàm tùy ý. Vì f cộng tính theo biến thứ hai, ta có

f (x, y + z) = f (x + y) + f (x, z). (18)

Thay (17) vào (18), ta thu được xk(y + z) = xk(y) + xk(z), với mọi x, y, z ∈ R.
Nếu x 6= 0, thì phương trình trên dẫn đến

k(y + z) = k(y) + k(z). (19)

Lại sử dụng tiếp tính cộng tính của f, ta thấy rằng k cũng cộng tính và do
đó nó tuyến tính. Do vậy k(x) = my với m là hằng số tùy ý. Kết hợp với (17)
ta được
f (x, y) = mxy, ∀x, y ∈ R, x 6= 0. (20)

Nếu x = 0, thì từ (13) , ta thấy rằng f (0, y) = 0 và do đó (20) đúng với


∀x, y ∈ R.

Ở định lí tiếp theo, ta trình bày một biểu diễn tổng quát đối với hàm song
cộng tính trong các số hạng của cơ sở Hamel.

38
Định lý 3.2. Mọi ánh xạ song cộng tính f : R2 → R đều có thể biểu diễn
như n X
m
X
f (x, y) = αkj rk sj , (21)
k=1 j=1

ở đây
n
X m
X
x= rk bk , y = αkj sj bj ,
k=1 j=1

rk , sj là hữu tỉ, bj là các phần tử của một cơ sở Hamel B và αkj tùy ý phụ
thuộc vào rk và bj .

Chứng minh. Cho B là một cơ sở Hamel đối với tập số thực R. Thế thì,
mỗi số thực x đều có thể biểu diễn dưới dạng
n
X
x= r k bk (22)
k=1

với bk ∈ R và với hệ số hữu tỉ rk . Tương tự, đối với bất kỳ số thực y khác, ta
cũng có biểu diễn
n
X
y= s j bj (23)
k=1

với bj ∈ R và với hệ số hữu tỉ sj . Vì f là song cộng tính

f (x1 + x2 , y) = f (x1 , y) + f (x2 , y) (24)

f (x, y1 + y2 ) = f (x, y1 ) + f (x, y2 ) (25)

với x1 , x2 , y1 , y2 ∈ R. Từ (24) và (25), sử dụng phép quy nạp, ta có


n
X  n
X
f xk , y = f (xk , y) (26)
k=1 k=1
n
 X  n
X
f x yk = f (x, yk ). (27)
k=1 k=1

39
Cho x1 = x2 = · · · = xn = xvà y1 = y2 = · · · = yn = y , trong (26) và (27), ta có

f (nx, y) = nf (x, y) = f (x, ny). (28)

m
Từ (28) với t = nx (hay nt = mx), ta thu được

nf (t, y) = f (nt, y) = f (mx, y) = f (x, y)

m
hay f (t, y) = n f (x, y), nghĩa là

m m
f( , x, y) = f (x, y). (29)
n n

Do f là song cộng tính, ta thấy rằng

f (x, 0) = 0 = f (0, y), ∀x, y ∈ R. (30)

Tiếp theo thay x2 = −x1 = x ở (24) và sử dụng (30) ta thu được

f (−x, y) = −f (x, y) (31)

Từ (31) và (29) ta kết luận rằng (28) là đúng với mọi số hữu tỉ. Lí luận giống
như trên với biến thứ hai và vì vậy ta có với mọi số hữu tỉ r và mọi số thực
x, y
f (rx, y) = rf (x, y) = f (x, ry). (32)

Từ đây, do (24), (25), (26) và (32), ta thu được


n
P m
P  n
P  m
P 
f (x, y) = f rk bk , s j bj = rk f bk , s j bj
k=1 j=1 k=1 j=1
n P
P m n P
P m
= rk sj f (bk , bj ) = rk sj αkj ,
k=1 j=1 k=1 j=1

ở đây αkj = f (bk , bj ).

40
4 Biểu diễn một số lớp hàm hai biến với phép
phản xạ

Ta sẽ mô tả một số lớp hàm hai biến với với đối số biến đổi dạng phản xạ.
Cụ thể là ta xét hàm hai biến đối xứng qua điểm (p, q).

Định nghĩa 4.1. Cho tập Ω := P × Q ⊂ R × R và điểm (p, q) là tâm đối xứng
của nó. Hàm f (x, y) xác định trên Ω được gọi là chẵn-chẵn (hoặc chẵn theo
cặp biến) đối với điểm (p, q) khi và chỉ khi

f (2p − x, y) = f (x, y) và f (x, 2q − y) = f (x, y), ∀(x, y) ∈ Ω.

Định nghĩa 4.2. Cho tập Ω := P × Q ⊂ R × R và điểm (p, q) là tâm đối xứng
của nó. Hàm f (x, y) xác định trên Ω được gọi là chẵn-lẻ đối với điểm (p, q) khi
và chỉ khi

f (2p − x, y) = f (x, y) và f (x, 2q − y) = −f (x, y), ∀(x, y) ∈ Ω.

Nhận xét 4.1. Tương tự, ta có các định nghĩa cho lớp hàm lẻ-chẵn, lẻ-lẻ.

Trường hợp riêng, ta có

Định nghĩa 4.3. Hàm f (x, y) xác định trong R × R được gọi là chẵn-chẵn
khi và chỉ khi

f (−x, y) = f (x, y) và f (x, −y) = f (x, y), ∀x, y ∈ R.

Câu hỏi tự nhiên nảy sinh là: Có thể mô tả hàm hai biến f (x, y) khi biết
nó là chẵn-chẵn theo điểm (p, q), tức là

f (2p − x, y) = f (x, y) và f (x, 2q − y) = f (x, y), ∀(x, y) ∈ Ω (33)

41
hay không? Nhận xét rằng

f (2p − x, 2q − y) = f (x, 2q − y) = f (x, y), ∀x, y ∈ Ω,

nên
1
f (x, y) = [f (x, y) + f (x, 2q − y) + f (2p − x, y) + f (2p − x, 2q − y)]. (34)
4

Ta chứng minh rằng hàm f (x, y) là chẵn-chẵn khi và chỉ khi tồn tại g(x, y) xác
định trong R × R sao cho
1
f (x, y) = [g(x, y) + g(x, 2q − y) + g(2p − x, y) + g(2p − x, 2q − y)]. (35)
4

Thật vậy, nếu f (x, y) có dạng (35) thì dễ kiểm chứng rằng điều kiện (33) là
thỏa mãn và nếuf (x, y) là chẵn thì nó có dạng (34) và vì thế có dạng (35) với
g = f.

Hệ quả 4.1. Hàm hai biến f (x, y) là chẵn-chẵn, tức là

f (−x, y) = f (x, y) và f (x, −y) = f (x, y), ∀(x, y) ∈ R. (36)

khi và chỉ khi


1
f (x, y) = [g(x, y) + g(x, −y) + g(−x, y) + g(−x, −y)]. (37)
4

trong đó g(x, y) là hàm tùy ý xác định trong R × R.

Tương tự, ta cũng có bài toán sau.

Bài toán 4.1. Hãy mô tả hàm hai biến f (x, y) khi nó là chẵn-lẻ theo điểm
(p, q), tức là

f (2p − x, y) = f (x, y) và f (x, 2q − y) = −f (x, y), ∀(x, y) ∈ Ω. (38)

42
Giải. Nhận xét rằng

f (2p − x, 2q − y) = f (x, 2q − y) = −f (x, y), ∀x, y ∈ Ω,

nên
1
f (x, y) = [f (x, y) + f (2p − x, y) − f (x, 2q − y) − f (2p − x, 2q − y)]. (39)
4
Ta chứng minh rằng hàm f (x, y) là chẵn-lẻ khi và chỉ khi tồn tại g(x, y) xác
định trong R × R sao cho
1
f (x, y) = [g(x, y) + g(2p − x, y) − g(x, 2q − y) − g(2p − x, 2q − y)]. (40)
4
Thật vậy, nếu f (x, y) có dạng (40) thì dễ kiểm chứng rằng điều kiện (38) là
thỏa mãn và nếuf (x, y) là chẵn-lẻ thì nó có dạng (39) và vì thế có dạng (40)
với g = f.

Tương tự, ta phát biểu các kết quả

Định lý 4.1. Mọi hàm hai biến f (x, y) là chẵn-chẵn, tức là

f (−x, y) = f (x, y) và f (x, −y) = f (x, y), ∀(x, y) ∈ R. (41)

khi và chỉ khi


1
f (x, y) = [g(x, y) + g(x, −y) + g(−x, y) + g(−x, −y)]. (42)
4
trong đó g(x, y) là hàm tùy ý xác định trong R × R.

Định lý 4.2. Mọi hàm hai biến f (x, y) là chẵn lẻ đối với điểm (p, q), tức là

f (2p − x, y) = f (x, y) và f (x, 2q − y) = −f (x, y), ∀(x, y) ∈ Ω. (43)

đều có dạng
1
f (x, y) = [g(x, y) + g(2p − x, y) − g(x, 2q − y) − g(2p − x, 2q − y)]. (44)
4

43
Định lý 4.3. Mọi hàm hai biến f (x, y) là lẻ-chẵn đối với điểm (p, q), tức là

f (2p − x, y) = −f (x, y) và f (x, 2q − y) = f (x, y), ∀(x, y) ∈ Ω. (45)

đều có dạng
1
f (x, y) = [g(x, y) − g(2p − x, y) + g(x, 2q − y) − g(2p − x, 2q − y)]. (46)
4
Định lý 4.4. Mọi hàm hai biến f (x, y) là lẻ-lẻ đối với điểm (p, q), tức là

f (2p − x, y) = −f (x, y) và f (x, 2q − y) = −f (x, y), ∀(x, y) ∈ Ω. (47)

đều có dạng
1
f (x, y) = [g(x, y) − g(2p − x, y) − g(x, 2q − y) + g(2p − x, 2q − y)]. (48)
4
Bây giờ ta xét trường hợp riêng đối với hàm f (x, y) xác định trong tập
 1
Ω0 = (0, 1) × (0, 1) và là chẵn-chẵn đối với điểm 0, .
2
Khi đó định lý 4.1 có dạng.
1 1
Hệ quả 4.2. Mọi hàm hai biến f (x, y) là chẵn-chẵn đối với điểm , , tức
2 2

f (1 − x, y) = f (x, y) và f (x, 1 − y) = f (x, y), ∀x, y ∈ (0, 1) (49)

đều có dạng
1
f (x, y) = [g(x, y) + g(1 − x, y) + g(x, 1 − y) + g(1 − x, 1 − y)]. (50)
4
1 1
Hệ quả 4.3. Hàm hai biến f (x, y) là lẻ-lẻ đối với điểm , , tức là
2 2
f (1 − x, y) = −f (x, y) và f (x, 1 − y) = −f (x, y), ∀x, y ∈ (0, 1) (51)

khi và chỉ khi nó có dạng


1
f (x, y) = [g(x, y) − g(1 − x, y) − g(x, 1 − y) + g(1 − x, 1 − y)]. (52)
4

44
5 Phương trình d’Alembert trong lớp hàm số liên
tục
Sử dụng các công thức biến đổi

cos(x + y) + cos(x − y) = 2 cos x cos y, ∀x, y ∈ R,

cosh(x + y) + cosh(x − y) = 2 cosh x cosh y, ∀x, y ∈ R,

ta sẽ có hai bài toán phương trình hàm tương ứng sau:

Bài toán 5.1. Tìm các hàm f (x) xác định, liên tục trên R và thỏa mãn các
điều kiện 
f (x + y) + f (x − y) = 2f (x)f (y), ∀x, y ∈ R,
(53)
f (0) = 1, ∃x0 ∈ R sao cho |f (x0 )| < 1.

Giải. Vì f (0) = 1 và f (x) liên tục trên R nên

∃ε > 0 sao cho f (x) > 0, ∀x ∈ (−ε, ε). (54)


x 
Khi đó theo (54) với n0 ∈ N đủ lớn thì f n00 > 0.
x  2
0
Nhận xét rằng f n < 1, ∀n ∈ N.
2
Từ (53) suy ra

f (2x1 ) = 2[f (x1 )]2 − 1 = 2 cos2 α − 1 = cos 2α.

Từ đó, bằng quy nạp suy ra f (mx1 ) = cos mα, ∀m ∈ N+ .


Mặt khác, đổi vai trò của x và y trong (53), ta có f (x−y) = f (y−x), ∀x, y ∈
R, do đó f (x) là hàm chẵn trên R nên

f (mx1 ) = cos nα, ∀m ∈ Z. (55)

45
x1
Cho x = y = vào (53) ta nhận được
2
h  x i2 1 + f (x1 ) 1 + cos α α
 2
1
f = = = cos ,
2 2 2 2
 
x1
do vậy f 2 = cos α2 .
Do vậy, bằng quy nạp ta chứng minh
x  α 1
f n
= cos , ∀n ∈ N. (56)
2 2n

Từ (55) và (56) suy ra


 mx  mα
1
f = cos , ∀n ∈ N+ , ∀m ∈ Z. (57)
2n 2n

Vì f (x) và cos x là các hàm chẵn liên tục trên R nên từ (57) suy ra
α
 
f (x1 t) = cos αt ⇔ f (x) = cos ax, với a = , ∀x ∈ R .
x1

Thử lại ta thấy f (x) = cos ax (a 6= 0) thỏa mãn các điều kiện bài toán.

Bài toán 5.2. Tìm các hàm f (x) xác định, liên tục trên R và thỏa mãn các
điều kiện 
f (x + y) + f (x − y) = 2f (x)f (y), ∀x, y ∈ R,
(58)
f (0) = 1, ∃x0 ∈ R sao cho f (x0 ) > 1.

Đáp số. f (x) = cosh ax, (a 6= 0).

Nhận xét 5.1. Dễ kiểm tra thấy hàm f ≡ 0 và f ≡ 1 cũng thỏa mãn điều
kiện

f (x + y) + f (x − y) = 2f (x)f (y), ∀x, y ∈ R.

Từ đó, ta có định lý về nghiệm của bài toán phương trình hàm d’Alembert.

46
Định lý 5.1 (Định lý nghiệm của phương trình hàm d’Alembert). Giả sử
f : R → R , liên tục và thỏa mãn điều kiện

f (x + y) + f (x − y) = 2f (x)f (y), ∀x, y ∈ R.

Khi đó f là một trong các hàm sau:

f (x) = 0, ∀x ∈ R,

f (x) = 1, ∀x ∈ R,

f (x) = cosh(αx),

f (x) = cos(βx),

trong đó α, β là các hằng số thực khác 0.

Bài toán 5.3. Cho a ∈ R, (α 6= 0) tìm các hàm f (x) liên tục trên R và thỏa
mãn điều kiện

f (x − y + a) − f (x + y + a) = 2f (x)f (y), ∀x, y ∈ R. (59)

Giải. Dễ kiểm tra thấy f ≡ 0 thỏa mãn các yêu cầu của bài toán.
Xét f 6≡ 0 khi đó, thay y bằng −y vào (59), ta có

f (x + y + a) − f (x − y + a) = 2f (x)f (y). (60)

Từ (59) và (60) ta có

f (x)f (y) = −f (x)f (y), x, y ∈ R, suy ra f (y) = −f (y), ∀y ∈ R do f 6≡ 0.

Suy ra f (x) là hàm lẻ.


Đổi vai trò của x và y trong (59), ta được

f (y − x + a) − f (x + y + a) = 2f (x)f (y) x, y ∈ R. (61)

47
Từ (59) và (61), ta có

f (x − y + a) = f (y − x + a) = f (−(x − y) + a) = −f (x − y − a) vì f là hàm lẻ.

Suy ra
f (x − y + a) = −f (x − y − a). (62)

Cho y = 0 thay vào (62) ta có

f (x + 4a) == f (x), ∀x ∈ R.

Ta thay x bằng x + a và y bằng y + a vào (59) ta được

f (x − y + a) + f (x + y + a) = 2f (x + a)f (y + a), ∀x, y ∈ R.

Đặt g(x) = f (x + a), suy ra g(x + y) + g(x − y) = 2g(x)g(y).


Từ tính liên tục của hàm f (x) trên R suy ra hàm g(x) liên tục trên R.
Từ g(x + y) + g(x − y) = 2g(x)g(y), ∀x, y ∈ R ta có g(x) là một trong bốn
hàm sau

g ≡ 0, g ≡ 1, g(x) = cosh(kx), g(x) = cos(kx), ∀x ∈ R.

Trường hợp 1: Nếu g(x) = 0, ∀x ∈ R, suy ra f (x) = 0, ∀x ∈ R không thỏa


mãn (trái với giải thiết f 6≡ 0).
Trường hợp 2: Nếu g(x) = 1, ∀x ∈ R, suy ra f (x) = 1, ∀x ∈ R không thỏa
mãn (59) nên không là nghiệm của bài toán.
Trường hợp 3: Nếu g(x) = cosh(kx), ∀x ∈ R, suy ra f (x) = cosh[k(x − a)]
không thỏa mãn vì cosh[k(x − a)] không là hàm tuần hoàn.
Trường hợp 4: Nếu g(x) = cos(kx), ∀x ∈ R suy ra f (x) = cos(k(x − a)).
Mà f (x + 4a) = f (x) nên ta có

f (x + 4a) = cos(k(x + 4a − a)) = f (x) = cos(k(x − a)),

⇒ cos(k(x + 3a)) = cos(k(x − a)), ( chọn x = x + a).

48
π
Suy ra cos(k(x + 4a)) = cos(kx) hay k = .
π  2a
Suy ra f (x) = cos (x − a) , ∀x ∈ R.
2a π 
Thử lại ta thấy hàm f (x) = cos (x − a) , x ∈ R thỏa mãn yêu cầu của
2a
bài toán.
π 
Kết luận: f ≡ 0 ; f (x) = cos (x − a) .
2a

Nhận xét 5.2. Từ kết quả và cách giải của bài toán 5.3 thì ta có các bài
toán khi cho a các giá trị cụ thể khác nhau.

6 Phương trình d’Alembert trong lớp các hàm số


không liên tục.

Từ các công thức Euler

eix + e−ix ex + e−x


cos x = ; cosh x = ;
2 2
eix − e−ix ex − e−x
sin x = ; sinh x = ,
2 2

ta thấy nghiệm của bài toán phương trình hàm d’Alembert có thể biểu diễn
thông qua hàm mũ. Tuy nhiên trong các bài toán trên ta phải sử dụng giả
thiết rằng hàm cần tìm liên tục trên R. Câu hỏi đặt ra là nếu bỏ giả thiết
về tính liên tục của nghiệm trên R thì có tồn tại nghiệm và cách xác định
nghiệm đó không?
Để trả lời câu hỏi đó, ta xây dựng một hàm mới để có thể biểu diễn nghiệm
của bài toán: Tìm hàm f (x) xác định trên R thỏa mãn điều kiện

f (x + y) + f (x − y) = 2f (x)f (y), ∀x, y ∈ R.

49
Định nghĩa 6.1. Hàm E : R → C được gọi là hàm dạng mũ (exponential)
nếu E thỏa mãn phương trình

E(x + y) = E(x)E(y), ∀x, y ∈ R.

Nếu E 6≡ 0 và là hàm liên tục thì E(x) = eλx ở đây 0 6= λ ∈ R.


Nếu E(x) 6= 0, ∀x ∈ R ta định nghĩa

E ∗ (x) = E(x)−1 , ∀x ∈ R.

Tiếp theo, ta khảo sát các tính chất sơ cấp của hàm E(x).

Tính chất 6.1. Nếu E : R → C là hàm dạng mũ và E(0) = 0 thì E(x) ≡ 0.


Chứng minh. Vì E : R → R là hàm dạng mũ nên

E(x + y) = E(x)E(y), ∀x, y ∈ R.

Cho y = 0, ta có E(x) = E(x)E(0), ∀x ∈ R.


Theo giả thiết E(0) = 0 suy ra E(x) = 0, ∀x ∈ R.

Tính chất 6.2. Nếu E : R → R là hàm dạng mũ và E 6≡ 0 thì E(0) = 1.

Tính chất 6.3. Nếu E : R → C là hàm dạng mũ và tồn tại x0 ∈ R sao cho
E(x0 ) = 0 thì E(x) = 0, ∀x ∈ R.

Tính chất 6.4. Nếu E : R → C là hàm dạng mũ và E 6≡ 0 thì

E ∗ (−x) = E(x).

50
Tính chất 6.5. Nếu E : R → C là hàm dạng mũ và E 6≡ 0 thì

E ∗ (x + y) = E ∗ (x)E ∗ (y), ∀x, y ∈ R.

Tính chất 6.6. Mọi hàm f : R → C thỏa mãn điều kiện

f (x + y) + f (x − y) = 2f (x)f (y), ∀x, y ∈ R (DE)

đều là hàm chẵn trên R.

Định lý 6.1. Mọi hàm f : R → C thỏa mãn

f (x + y) + f (x − y) = 2f (x)f (y), ∀x, y ∈ R (DE)

là các hàm
E(x) + E ∗ (x)
f ≡ 0 hoặc f (x) = ,
2
trong đó E : R −→ C∗ là hàm dạng mũ định nghĩa ở trên.

Chứng minh. Dễ thấy f ≡ 0 thỏa mãn (DE).


Xét f 6≡ 0.
Cho x = 0 = y thay vào (DE) ta được

f (0)[1 − f (0)] = 0 ⇒ f (0) = 0 hoặc f (0) = 1.

Nếu f (0) = 0, đặt u = x + y, v = x − y khi đó (DE) trở thành


u + v  u − v 
f (u) + f (v) = 2f f , ∀u, v ∈ R.
2 2
u + v 
Cho u = v, suy ra 2f (u) = 2f f (0) = 0, ∀u ∈ R (trái giả thiết f 6≡ 0),
2
suy ra f (0) = 1.

51
Cho y = x thay vào (DE), ta có:
f (2x) + f (0) = 2[f (x)]2 hay f (2x) = 2f (x)2 − 1 1

Thay x bằng x + y và y bằng x − y vào phương trình (DE), ta có

f (x + y + x − y) + f (x + y − x + y) = 2f (x + y)f (x − y)

⇒ f (2x) + f (2y) = 2f (x + y)f (x − y), ∀x, y ∈ R.

Ta có:
h i2 h i2
f (x + y) − f (x − y) = f (x + y) + f (x − y) − 4f (x + y)f (x − y)
h i2
= 2f (x)f (y) − 4f (x + y)f (x − y)
h i
2 2
= 4f (x) f (y) − 2 f (2x) + f (2y)
h i
2 2 2 2
= 4f (x) f (y) − 2 f (x) − 1 + f (y) − 1
h ih i
= 4 f (x)2 − 1 f (y)2 − 1

rh ih i
⇒ f (x + y) − f (x − y) = ±2 f (x)2 − 1 f (y)2 − 1 , ∀x, y ∈ R.

Cộng hai vế phương trình trên với phương trình (DE) ta có


p
f (x + y) = f (x)f (y) ± [f (x)2 − 1][f (y)2 − 1]

suy ra [f (x + y) − f (x)f (y)]2 = [f (x)2 − 1][f (y)2 − 1]. (63)

Ta xét hai trường hợp.


- f (x) ∈ {−1; 1}, ∀x ∈ R và
- f (x) 6∈ {−1; 1}, với một vài giá trị x ∈ R.
Trường hợp 1: Nếu f (x) ∈ {−1; 1}, ∀x ∈ R. Kết hợp với (63) ta có

f (x + y) = f (x)f (y), ∀x, y ∈ R,


1
Để đơn giản trong ký hiệu từ đây ta hiểu f (x)2 = [f (x)]2 .

52
ở đây f (x) nhận một trong hai giá trị 1 hoặc −1 nên
f (x) + f ∗ (x)
f (x) = f ∗ (x) và f (x) = , ∀x ∈ R.
2
E(x) + E ∗ (x)
Khi đó f (x) = , ∀x ∈ R với E(x) ∈ {−1; 1}.
2
Trường hợp 2: f (x) 6∈ {−1; 1}, với một vài giá trị x ∈ R.
Do đó tồn tại x0 ∈ R sao cho f (x0 )2 − 1 6= 0.
Đặt α = f (x0 ) thì α2 − 1 6= 0. Đặt β 2 = α2 − 1. Ta đặt
1
E(x) :=f (x) + [f (x + x0 ) − f (x)f (x0 )]
β
1 
= f (x + x0 ) + (β − α)f (x) , ∀x ∈ R.
β

Ta thấy biểu thức E(x) là một hàm số xác định trên R và


1
[E(x) − f (x)]2 = [f (x + x0 ) − f (x)f (x0 )]2
β2
1
= 2 [f (x)2 − 1][f (x0 )2 − 1]theo (63)
β
α2 − 1
= [f (x)2 − 1]vì (β 2 = α2 − 1)
β2
=f (x)2 − 1.

Suy ra

E(x)2 − 2E(x)f (x) + f (x)2 = f (x)2 − 1

⇒ E(x)2 − 2E(x)f (x) + 1 = 0, ∀x ∈ R.

Nếu E(x) = 0 kéo theo 1 = 0, vô lý, nên E(x) 6= 0 và


E(x)2 + 1 E(x) + E ∗ (x)
f (x) = = , ∀x ∈ R.
2E(x) 2

Ta sẽ chứng minh E(x) thỏa mãn tính chất

E(x + y) = E(x)E(y), ∀x, y ∈ R.

53
Thật vậy, ta có

2[f (x0 + x)f (y) + f (x0 + y)f (x)]

=f (x0 + x + y) + f (x0 + x − y) + f (x0 + y + x) + f (x0 + y − x) theo (DE)

=2f (x0 + x + y) + f (x0 + x − y) + f (x0 + y − x)

=2f (x0 + x + y) + 2f (x0 )f (x − y)

=2{f (x0 + x + y) + f (x0 )[2f (x)f (y) − f (x + y)]}


 
=2{f (x0 + x + y) + α 2f (x)f (y) − f (x + y) }.

Mặt khác ta có

2f (x0 + x)f (x0 + y)

=f (x0 + x + x0 + y) + f (x0 + x − x0 − y) theo (DE)

=f (x0 + (x0 + x + y)) + f (x − y)

=[2f (x0 )f (x0 + x + y) − f (x0 + x + y − x0 )] + [2f (x)f (y) − f (x + y)] theo (DE)

=2f (x0 )f (x0 + x + y) − f (x + y) + 2f (x)f (y) − f (x + y)

=2[f (x)f (y) + αf (x0 + x + y) − f (x + y)].

54
Ta lại có
1
E(x)E(y) = [f (x + x0 ) + (β − α)f (x)][f (y + x0 ) + (β − α)f (y)]
β2
1
= {f (x + x0 )f (y + x0 ) + (β − α)[f (x)f (x0 + y) + f (y)f (x0 + x)
β2
+ f (y)f (x0 + x)] + (β − α)2 f (x)f (y)}
1
= 2 {f (x)f (y) + αf (x0 + x + y) − f (x + y)
β
+ (β − α)[f (x0 + x + y) + 2αf (x)f (y) − αf (x + y)]

+ (β − α)2 f (x)f (y)}


1
= 2 {[(β − α)2 + 2α(β − α) + 1]f (x)f (y) + βf (x0 + x + y)
β
− [1 + (β − α)α]f (x + y)}
1
= 2 {(β 2 − α2 + 1)f (x)f (y) + βf (x0 + x + y) − (βα − β 2 )f (x)f (y)}
β
1
= 2 [βf (x0 + x + y) + β(β − α)f (x + y)] = E(x + y).
β

Suy ra E(x) là hàm mũ. Thử lại ta có:


E(x) + E ∗ (x) E(y) + E ∗ (y)
f (x + y) + f (x − y) = 2 = 2f (x)f (y)
2 2
E(x) + E ∗ (x)
Vậy: f (x) = , trong đó E(x) là hàm mũ.
2

Bài toán 6.1. Tìm các hàm f : R → C và g : R → C thỏa mãn điều kiện

f (x − y) = f (x)f (y) + g(x)g(y), ∀x, y ∈ R. (64)

Giải. Đổi vai trò của x và y cho nhau trong (64) và kết hợp (64) và (??), ta
được
f (x − y) = f (y − x), ∀x, y ∈ R. (65)

55
Cho y = 0 vào (65), ta được

f (x) = f (−x), ∀x ∈ R,

suy ra f (x) là hàm số chẵn trên R.


Thay x bằng −x và y bằng −y vào (64), ta thu được

f (y − x) = f (−x)f (−y) + g(−x)g(−y),

mặt khác do f là hàm chẵn nên

f (x − y) = f (x)f (y) + g(−x)g(−y). (66)

Kết hợp (66) và (64) ta thấy

g(x)g(y) = g(−x)g(−y), ∀x, y ∈ R. (67)

Xét hai trường hợp


Trường hợp1: g(x) là hàm hằng, khi đó g(x) = c, ∀x ∈ R thì (64) trở thành

f (x − y) = f (x)f (y) + c2 , ∀x, y ∈ R.

Thay y bằng −y và sử dụng tính chất hàm f (x) là hàm chẵn, ta được

f (x + y) = f (x)f (y) + c2 = f (x − y), ∀x, y ∈ R,

suy ra f là hàm hằng.


Đặt f (x) = b,√ ∀x ∈ R, khi đó (64) trở thành b = b2 + c2 hay b2 − b + c2 = 0.
1± 1 − 4c2
Suy ra b = .
2
Vậy nghiệm của bài toán trong trường hợp này là:

1 − 4c2

f (x) = 1 ±
,
2 ∀x ∈ R, c = const ∈ C
g(x) = c.

56
Trường hợp 2: g(x) không là hàm hằng, ta chọn y0 sao cho g(y0 ) 6= 0, từ (67)
thay y = y0 ta được
g(x) = cg(−x), ∀x ∈ R, (68)

ở đây c là hằng số khác 0. Suy ra

g(x) = c2 g(x), ∀x ∈ R,

mặt khác do g không là hàm hằng nên c2 = 1 hay c = ±1.


Giả sử c = 1, thay vào (68) ta được

g(x) = g(−x), ∀x ∈ R,

suy ra g(x) là hàm chẵn.


Thay y bằng −y vào (64), ta có

f (x + y) = f (x)f (−y) + g(x)g(−y)

= f (x)f (y) + g(x)g(y) = f (x − y).

Suy ra f (u) = f (v), ∀u, v ∈ R, hay f là hàm hằng.


Đặt f (u) = m, ∀u ∈ R, m là một hằng số khi đó (64) trở thành

m = m2 + g(x)g(y), ∀x, y ∈ R,

suy ra g(x) là hàm hằng (trái giả thiết g(x) không là hàm hằng) điều đó chứng
tỏ rằng c = −1 hay g(x) là hàm lẻ.
Thay y bằng −y vào (64), ta có

f (x + y) = f (x)f (−y) + g(x)g(−y)

= f (x)f (y) − g(x)g(y) (vì f là hàm chẵn, g là hàm lẻ),

suy ra
f (x + y) = f (x)f (y) − g(x)g(y). (69)

57
Cộng hai vế của (64) và (69), ta được

f (x + y) + f (x − y) = 2f (x)f (y), ∀x, y ∈ R.

Hay hàm f thỏa mãn điều kiện của phương trình (DE), suy ra
E(x) + E ∗ (x)
f (x) = , E(x) là hàm mũ.
2
Thay hàm f vào (64), ta nhận được
E(x − y) + E ∗ (x − y) E(x) + E ∗ (x) E(y) + E ∗ (y)
= + g(x)g(y)
2 2 2
hay
E(x) − E ∗ (x) E(y) − E ∗ (y)
g(x)g(y) = − , ∀x, y ∈ R,
2 2
E(x) − E ∗ (x)
mà hàm g(x) không là hàm hằng, suy ra g(x) = i .
2
Suy ra nghiệm của bài toán trong trường hợp này là:
E(x) + E ∗ (x) E(x) − E ∗ (x)
f (x) = , g(x) = i ,
2 2
ở đây E(x) là hàm mũ.
Kết luận:

f (x) = 1 ± 1 − 4c2 ,

2 ∀x ∈ R, c = const ∈ C;
g(x) = c,


f (x) = E(x) + E (x) ,


2 E(x) là hàm mũ.
g(x) = i E(x) − E (x) ,
2
Chú ý 6.1. Nếu cho f : R → C , g : R → C ( g(x) không là hàm hằng, f 6≡ 0)
và thỏa mãn điều kiện

f (x − y) = f (x)f (y) + g(x)g(y), ∀x, y ∈ R,

thì ta chứng minh được : g(x)2 + f (x)2 = 1, ∀x ∈ R.

58
Bài toán 6.2. Tìm các hàm f, g : R → R liên tục trên R và thỏa mãn điều
kiện
f (x − y) = f (x)f (y) + g(x)g(y), ∀x, y ∈ R. (70)
p
Giải. Nếu f (x) là hàm hằng thì dễ thấy f (x) = c và g(x) = c(1 − c), ở đây

c = const là nghiệm của bài toán,


p
f (x) = c và g(x) = − c(1 − c) ở đây c = const cũng là nghiệm của bài
toán.
Nếu f (x) không phải là hàm hằng thì đổi vai trò của x và y cho nhau trong
(70), ta thu được
f (y − x) = f (y)f (x) + g(y)g(x). (71)

Từ (70) và (71) ta được

f (x − y) = f (y − x), ∀x, y ∈ R. (72)

Cho y = 0 thay vào (72) ta được

f (x) = f (−x), ∀x ∈ R,

suy ra f (x) là hàm số chẵn trên R.


Thay x bằng −x và y bằng −y vào (70) ta có

f (y − x) = f (−x)f (−y) + g(−x)g(−y).

Do f (x) là hàm chẵn nên thay x bằng −x và y bằng −y vào (70) ta có

f (−x − y) = f (−x)f (−y) + g(−x)g(−y) = f (x)f (y) + g(−x)g(−y). (73)

Kết hợp (73) và (70) ta có:

g(x)g(y) = g(−x)g(−y), ∀x, y ∈ R. (74)

59
Giả sử g(x) là hàm hằng, khi đó g(x) = c, ∀x ∈ R thì (70) trở thành

f (x − y) = f (x)f (y) + c2 , ∀x, y ∈ R.

Thay y bằng −y và sử dụng tính chất hàm f (x) là hàm chẵn, ta có

f (x + y) = f (x)f (−y) + c2 = f (x)f (y) + c2 = f (x − y), ∀x, y ∈ R,

suy ra f (x) là hàm hằng (trái giả thiết) từ đó chứng tỏ g(x) không là hàm
hằng.
Chọn y0 sao cho g(y0 ) 6= 0, từ (74) thay y = y0 ta có:

g(x) = cg(−x), ∀x ∈ R, (75)

ở đây c là hằng số khác 0. Từ đó suy ra:

g(x) = c2 g(x), ∀x ∈ R.

Mặt khác do g(x) không là hàm hằng nên c2 = 1 ⇒ c = ±1.


Giả sử c = 1, từ (75) suy ra

g(x) = g(−x), ∀x ∈ R. (76)

Suy ra g(x) là hàm chẵn. Thay y bằng −y vào (70), ta có

f (x + y) = f (x)f (−y) + g(x)g(−y)

= f (x)f (y) + g(x)g(y) = f (x − y),

suy ra f (u) = f (v), ∀u, v ∈ R.


Suy ra f (u) = m, ∀u ∈ R, m là một hằng số hay f (x) là hàm hằng (trái
giả thiết) điều đó chứng tỏ rằng c = −1 hay g(x) là hàm lẻ.
Thay y bằng −y vào (70), ta có

f (x + y) = f (x)f (−y) + g(x)g(−y)

= f (x)f (y) − g(x)g(y) (vì f (x) là hàm chẵn, g(x) là hàm lẻ),

60
suy ra,
f (x + y) = f (x)f (y) − g(x)g(y). (77)

Cộng hai vế của (70) và (77) , ta được

f (x + y) + f (x − y) = 2f (x)f (y), ∀x, y ∈ R.

Suy ra f (x) là hàm liên tục nên thỏa mãn điều kiện của phương trình d’Alembert
và f 6≡ c thì f (x) là một trong hai hàm sau

f (x) = cos(αx), f (x) = cosh(βx), (α 6= 0, β 6= 0).

Và theo chú ý 6.1 ta có f (x)2 + g(x)2 = 1, ∀x ∈ R.


Nếu f (x) = cos(αx) thì

g(x)2 =1 − cos2 (αx) ⇒ g(x) = ± sin(αx).

Nếu f (x) = cosh(βx) khi đó |f (x)| = | cosh(βx)| ≥ 1 mà theo chú ý 6.1 thì
ta có |f (x)| ≤ 1, ∀x ∈ R, từ đó suy ra f (x) = cosh(βx) không là nghiệm của
bài toán.
Kết luận:
  
f = c f = c f (x) = cos(αx)
p ; ; , (α 6= 0, α ∈ R).
 g= c(1 − c) g = −pc(1 − c) g(x) = ± sin(αx)

Bài toán 6.3. Tìm các hàm f, g : R → C thỏa mãn

f (x + y) = f (x)g(y) + f (y)g(x), ∀x, y ∈ R. (78)

Giải. Dễ kểm tra thấy f ≡ 0 và g(x) là hàm số tùy ý là nghiệm của (78).
Xét f 6≡ 0, ta có:

f (x + y + z) =f (x)g(y + z) + f (y + z)g(x)

=f (x)g(y + z) + f (y)g(z)g(x) + f (z)g(y)g(x) (79)

61

f (x + y + z) =f (x + y)g(z) + f (z)g(x + y)

=f (x)g(y)g(z) + f (y)g(x)g(z) + f (z)g(x + y) (80)

Từ (79)và (80), ta có:

f (x)[g(y + z) − g(y)g(z)] = f (z)[g(x + y) − g(x)g(y)]. (81)

Vì f 6≡ 0 khi đó tồn tại z0 6= 0 sao cho f (z0 ) 6= 0.


Thay z = z0 vào (81) suy ra,

g(x + y) − g(x)g(y) = f (x)k(y), (82)


g(y + z0 ) + g(y)g(z0 )
trong đó k(y) = .
f (z0 )
Đổi vai trò của x và y cho nhau trong (82), ta có

g(x + y) − g(x)g(y) = f (y)k(x) (83)

Từ (82) và (83), ta có

f (x)k(y) = f (y)k(x), ∀x, y ∈ R. (84)

Thay x = z0 vào (84) ta được,


k(z0 )
f (z0 )k(y) = f (y)k(z0 ) ⇒ k(y) = f (y), ∀y ∈ R.
f (z0 )

Suy ra
k(x) = α2 f (x), ∀x ∈ R (85)

ở đây α là một hằng số nên từ (85) và (83) ta có:

g(x + y) = g(x)g(y) + α2 f (x)f (y), ∀x, y ∈ R. (86)

62
Trường hợp 1: Nếu α = 0 thì từ (86), ta có

g(x + y) = g(x)g(y), ∀x, y ∈ R

suy ra g(x) = E(x) ở đây E(x) là hàm mũ.


Mặt khác, từ f 6≡ 0 ta có E(x) 6= 0, ∀x ∈ R.
Thay g(x) = E(x) vào (78) ta có,

f (x + y) = f (x)E(y) + f (y)E(x)
f (x + y) f (x) f (y)
⇔ = + . (87)
E(x + y) E(x) E(y)

f (x)
Đặt A(x) = khi đó theo (87) ta có
E(x)

A(x + y) = A(x) + A(y)

suy ra A : R 7−→ C là hàm cộng tính.


Trong trường hợp này thì nghiệm của bài toán là:

f (x) = A(x)E(x), g(x) = E(x).

Trường hợp 2: Nếu α 6= 0, ta nhân hai vế của (78) với α và cộng và trừ kết
quả với (86), ta được

αf (x + y) + g(x + y) = αf (x)g(y) + αf (y)g(x) + g(x)g(y) + α2 f (x)f (y)

⇔g(x + y) + αf (x + y) = [g(x) + αf (x)][g(y) + αf (y)] (88)


g(x + y) − αf (x + y) = [g(x) − αf (x)][g(y) − αf (y)]. (89)

E1 (x) = g(x) + αf (x),
Đặt thì (88) và (89) được viết lại là
E2 (x) = g(x) − αf (x),

63
E1 (x + y) = E1 (x).E1 (y) và E2 (x + y) = E2 (x).E2 (y) suy ra E1 (x), E2 (x) là
hàm mũ. Ta có,

f (x) = E1 (x) − E2 (x)


 
g(x) + αf (x) = E1 (x)
⇔ 2α
g(x) − αf (x) = E2 (x) g(x) = E1 (x) + E2 (x)
2

Kết luận: Nghiệm của bài toán là các cặp hàm



  E (x) − E2 (x)
f ≡ 0 f (x) = A(x)E(x) f (x) = 1

, , 2α , (α 6= 0)
g(x) là hàm tùy ý g(x) = E(x) g(x) = E1 (x) + E2 (x)

2

ở đây E, E1 , E2 : R → C là hàm mũ, A : R → C là hàm cộng tính.

Chú ý 6.2. Nếu bổ sung thêm điều kiện hàm f (x) là hàm chẵn và g(x) là
hàm lẻ thì nghiệm của bài toán 2.6 là

E1 (x) − E1∗ (x) E1 (x) + E1∗ (x)


f (x) = và g(x) =
2α 2

Chứng minh. Thật vậy từ nghiệm của bài toán 2.6 suy ra

E1∗ (x) = E1 (−x) = g(−x) + αf (−x) = g(x) − αf (x) = E2 (x)

E1 (x)−E1∗ (x) E1 (x)+E1∗ (x)


suy ra f (x) = 2α và g(x) = 2 .

Tài liệu

[1] T. Acze’l, Lectures on functional equations and their applications, Aca-


demic Press, New York/San Francisco/London, 1966.

[2] Christopher G.,Functinal equations and how to solve them, Springer,2000.

64
[3] Z. Daroczy and A. Jarai, On the measurable solution of a functional
equation of the information theory, Acta Math. Acad Sci. Hungaricae,
vol.34, 105-116, 1979.

[4] M. Kuczma, B. Choczewski, R. Ger, Interative hàm al Equa-


tions, Cambridge University Press, Cambridge/New York/Port
Chester/Melbourne/Sydney, 1990.

[5] P.K. Sahoo, T. Riedel, Mean Value Theorems and Functional Equations,
World Scientific, Singapore/New Jersey/London/HongKong, 1998.

[6] D. Przeworska - Rolewicz, Algebraic analysis, PWN - Polish Scientific


Publishers and D. Reided Publishing Company, Warszawa - Dordrecht,
1988.

[7] D. Przeworska - Rolewicz and S. Rolewicz, Equations in linear space,


Monografie Matematyezne 47, PWN - Polish Scientific Publishers,
Warszawa, 1968.

[8] Ng. V. Mau, Boundary value problems and controllability of linear sys-
tems with right invertible operators, Dissertationes Math., CCCXVI,
Warszawa, 1992.

[9] Nguyễn Văn Mậu, Phương trình hàm, NXBGD, 1997.

[10] Ng. V. Mau, Algebraic Elements and boundary value problems in linear
spaces, VNU Pub. Hours, 2005.

[11] Nguyễn Văn Mậu, 1997, Phương trình hàm, NXB Giáo Dục

[12] Nguyễn Văn Mậu, 2006, Các bài toán nội suy và áp dụng, NXB GD.

65
Một số lớp phương trình Diophant cơ bản

Đặng Hùng Thắng


Trường Đại học KHTN, ĐHQGHN

1 Mở đầu

Phương trình Điophant là phương trình đại số có dạng

P (x1 , x2 , . . . , xn ) = 0

ở đó P (x1 , x2 , . . . , xn ) là một đa thức hệ số nguyên và ta chỉ yêu cầu tìm các


nghiệm nguyên (hoặc nguyên dương) mà thôi.
Người đầu tiên nghiên cứu về các phưuowng trình loại này là nhà toán học
Hylap Điophantus sống ở thành phố Alexandria vào khoảng năm 250 sau công
nguyên. Toàn bộ sự nghiệp của ông dành cho việc thu thập những bài toán
loại này,sáng tạo thêm những bài toán mới rồi sau đó biên soạn thành một
bộ sách tên là Arithmetica (Số học) gồm 13 tập. Trải qua những biến loạn
thời Trung cổ, đến thời Phục hưng bảy tập đã bị thất lạc chỉ còn sáu tập là
còn lưu giữ được và đã truyền cảm hứng và trở thành Kinh thánh cho các
nhà Toán học thời Phục hưng trong đó có Ferma. Ông đã xem xét kỹ lưỡng
sự trình bày của Diophante về bộ ba các số Pitago (x, y, z) là nghiệm nguyên
dương của phương trình x2 + y 2 = z 2 . Ông đã ghi lại bên lề cuốn sách nhận
xét sau đây của mình

66
Một số lập phương không thể viết dược dưới dạng tổng của hai lập phương.
Một lũy thừa bậc bốn không thể viết dược dưới dạng tổng của hai lũy thừa bậc
bốn. Một cách tổng quát, một số là lũy thừa bậc lớn hơn hai không thể viết
dưới dạng tổng của hai lũy thừa cùng bậc. Tôi đã có mọt chứng minh thực sự
tuyệt vời mệnh đề này nhưng do lề quá hẹp nên không thể viết hết ra được.

Như vậy vào năm 1637 Fermar đã phát biểu và tin rằng ông đã chứng
minh được định lý sau đây, mà sau này thường được gọi là Định lý cuối cùng
của Ferma (mà đúng ra phải gọi là Giả thuyết cuối cùng của Ferma).
Với n > 2 thì phương trình Điophant

xn + y n = z n

không có nghiệm nguyên dương.


Trong hơn ba thế kỷ, những nỗ lực tìm kiếm chứng minh định lý này đều
thất bại và Định lý cuối cùng của Ferma đã trở thành một bài toán khó giải
nhất của toán học.Nhiều nhà toán học xuất sắc đã thất bại ê chề khi cố gắng
tấn công bài toán này. Mấy dòng ghi chú bên lề của Ferma như một lời thách
thức nhiều thế hệ các nhà toán học.
Mãi đến năm 1994 (tức là 350 năm sau) Định lý cuối cùng của Ferma
này mới chính thức được chứng minh bởi nhà toán học Mỹ Andrew Wiles.
Chứng minh của ông dài 130 trang, là một tuyệt phẩm của toán học hiện
đại, sử dụng những kỹ thuật toán học mới nhất và tinh tế nhất của toán học
thế kỷ 20 như phương trình eliptic,dạng modular, nhóm Galoir, phương pháp
Kolyvagin-Flax, giả thuyết Taniyama-Shimura..
Như vậy hai khả năng có thể xảy ra : Hoặc là chứng minh tuyệt vời của
ông thực ra là một chứng minh sai. Hoạc là ông đã có một chứng minh thiên
tài chỉ dựa trên những kỹ thuật sơ cấp toán học của thế kỷ 17 với một suy

67
luận cực kỳ độc đáo tới mức từ Ơle dến đến Wiles không ai nghĩ ra được.
Hiện nay một số nguời vẫn tin vào khả năng này và họ vẫn đang tìm cách
phát minh lại chứng minh của Ferma.

2 Phương trình bậc nhất

2.1 Phương trình bậc nhất hai ẩn

Phương trình Điophant đơn giản nhất là phương trình bậc nhất hai ẩn

Ax + By = C (1)

trong đó a, b, c ∈ Z .
Định lý 1 a) Phương trình (1) có nghiệm nguyên khi và chỉ khi d = (A, B)
là ước của C .
b) Nếu (x0 , y0 ) là một nghiệm của (1) thì mọi nghiệm của (1) được cho bởi

 x = x0 + B
d
A
y = y0 − dt

ở đó t ∈ Z
Chứng minh a) Nếu (1) có nghiệm (x0 , y0 ) thì Ax0 + By0 = c → d|C .
Đảo lại giả sử d|C . Không giảm tổng quát giả sử B > 0. Ta có A = da, B =
db, C = dc với (a, b) = 1. Khi đó (1) tương đương với

ax + by = c (2)

Vì (a, b) = 1 nên tập {a, 2a, ..., ab} là hệ đầy đủ mod b. Vậy tồn tại x ∈ {1, 2, ...a1 }
sao cho ax ≡ c (mod b) → c − ax = by với y ∈ Z .

68
b)Nếu (x, y) cho bởi công thức trên thì Ax+By = Ax0 +By0 = C . Đảo lại giả sử
(x, y) là một nghiệm của (1) tức là thỏa mãn (2). Khi đó ax + by = ax0 + by0 →
a(x − x0 ) = b(y0 − y) → b|a(x − x0 ). Vì (a, b) = 1 nên b|x − x0 → x − x0 = bt → x =
x0 + bt = x0 + Bd t, t ∈ Z . Từ x − x0 = bt suy ra at = y0 − y → y = y0 − at = y0 − Ad t.
Định lý được chứng minh.
Hệ quả Nếu a, b ∈ N ∗ , (a, b) = 1 thì tồn tại u, v ∈ N ∗ để au − bv = 1.
Chứng minh Theo định lý trên phương trình ax + by = 1 có nghiệm

x = x0 + bt
y = y0 − at, t ∈ Z

do đó phương trình au − bv = 1 có nghiệm

u = bt + x0 , v = at − y0 , t ∈ Z

Chọn t ∈ Z : t > −x0 /b, t > y0 /a ta có u, v ∈ N ∗ .


Từ định lý trên ta thấy việc tìm nghiệm của (1) quy về việc tìm UCLN
d = (A, B) và tìm một nghiệm riêng (x0 , y0 ). Sau đây ta sẽ trình bày một
thuật toán sử dụng liên phân số để tìm một nghiệm riêng (x0 , y0 ).
Ta nhắc lại một số kiến thức về liên phân số. Biểu thức có dạng
1
a0 +
1
a1 +
. 1
a2 + . . +
1
an−1 +
an
trong đó a0 , a1 , ..., an ∈ R và a1 , ..., an 6= 0 được ký hiệu là [a0 ; a1 , ..., an ]. Từ
định nghĩa suy ra

1
[a0 ; a1 , ..., ak+1 ] = a0 +
[a1 ; .a2 , ..., ak+1 ]

69
Nếu a0 ∈ Z và a1 , ..., an ∈ N ∗ ta nói [a0 ; a1 , ..., an ] là một liên phân số hữu hạn
có độ dài n. Rõ ràng một liên phân số hữu hạn là số hữu tỷ. Ngược lại ta có
Bổ đề 1Mỗi số hữu tỷ là một liên phân số hữu hạn
Chứng minh Giả sử x = a/b trong đó a, b ∈ Z và b > 0. Đặt r0 = a, r1 = b.
Thực hiện thuật toán O-cơ -lit

r0 = r1 q1 + r2 0 < r2 < r1

r1 = r2 q2 + r3 0 < r3 < r2

.......

rn−2 = rn−1 qn−1 + rn 0 < rn < rn−1

rn−1 = rn qn

Từ đó
r0
x= = [q1 ; q2 , ..., qn ]
r1
Ví dụ1 Biểu diễn số x = 62/23 thành giản phân. Ta có

62 = 2.23 + 16

23 = 1.16 + 7

16 = 2.7 + 2

7 = 3.2 + 1

2 = 2.1

Do đó
62
x= = [2; 1, 2, 3, 2]
23
Cho liên phân số [a0 ; a1 , ..., an ]. Với mỗi k 6 n liên phân số Ck = [a0 ; a1 , ..., ak ]
được gọi là giản phân thứ k của [a0 ; a1 , ..., an ].
Ta có định lý cơ bản sau của liên phân số

70
Định lý Cho liên phân số [a0 ; a1 , ..., an ]. Xét dãy số p0 , p1 , ...pn và q0 , q1 , ..., qn
xác định truy hồi theo cách sau

p 0 = a0 q0 = 1

p 1 = a0 a1 + 1 q 1 = a1

....

pk = ak pk−1 + pk−2 qk = ak qk−1 + qk−2

Khi đó
pk
Ck =
qk
pk qk−1 − pk−1 qk = (−1)k=1

Để giải phương trình


Ax + By = C (1)

ta thực hiên theo các bước sau

• Dùng thuật toán O-cơ-lit tìm d = (A, B). Nếu d không là ước của C
phương trình vô nghiệm. Nếu d|C viết A = da, B = db, C = dc và (1)
tương đương với
ax + by = c (2)

ở đó (a1 , b1 ) = 1

• Ta biểu diễn a/|b| thành liên phân số

a
= [a0 ; a1 , a2 , ..., an ]
|b|

Giả sử pn−1 /qn−1 và pn /qn là hai giản phân cấp n − 1 và cấp n. Ta có


a/|b| = pn /qn , (a, b) = 1, (pn , qn ) = 1 nên a = pn , |b| = qn . Theo định lý ta

71

pn qn−1 − pn−1 qn = (−1)n−1 →

aqn−1 − |b|pn−1 = (−1)n−1 →

a(−1)n−1 qn−1 + |b|(−1)n pn−1 = 1

a(−1)n−1 cqn−1 + |b|(−1)n cpn−1 = c

• Vậy nếu b > 0 thì phương trình (1) có nghiệm riêng là

x0 = (−1)n−1 cqn−1 ; y = (−1)n cpn−1

Nếu b < 0 thì phương trình (1) có nghiệm riêng là

x0 = (−1)n−1 cqn−1 ; y = (−1)n−1 cpn−1

Ví dụ Giải phuơng trình 342x − 123y = 15 (10 ).


Giải

• Ta có (342, 123) = 3 do đó a = 114, b = −41 . Do đó (1’) tương đương với

114x − 41y = 5 (20 )

• Biểu diễn 114/41 thành liên phân số

114 = 2.41 + 32

41 = 1.32 + 9

32 = 3.9 + 5

9 = 1.5 + 4

5 = 1.4 + 1

4 = 4.1

72
Do vậy
62
= [2; 1, 3, 1, 1, 4]
23
Ta có n = 5, p4 /q4 = [2, 1, 3, 1, 1] = 25/9 → p4 = 25, q4 = 9.

• Vì b = −41 < 0 nên (1’) có nghiệm riêng là x0 = (−1)4 5.9 = 45, y0 =


(−1)4 5(25) = 125. Vậy nghiệm của (1’) là

x = 45 + 41t
y = 125 + 114t

với t ∈ Z .

2.2 Phương trình bậc nhất nhiều ẩn

Phương trình bậc nhất k ẩn (k ≥ 2) là phương trình dạng

A1 x 1 + A2 x 2 + · · · + A k x k = C (4)

trong đó a, b, c ∈ Z .
Định lý 2 i) Phương trình (4) có nghiệm nguyên khi và chỉ khi d = (A1 , A2 , ..., Ak )
là ước của C .
ii) Nếu (4) có nghiệm nó sẽ có vô số nghiêm.
Chứng minh i) Điều kiện cần hiển nhiên .Chứng minh điều kiện đủ bằng quy
nạp: Với k = 2 khẳng định đúng. Giả sử đúng với k . Xét phương trình với
d = (A1 , ..., Ak , Ak+1 )|C . Đặt H = (A1 , ..., Ak ) → d = (H, Ak+1 )|C . Theo định lý
1 tồn tại t, xk+1 ∈ Z sao cho Ht + Ak+1 xk+1 = C . Vì H|Ht nên theo giả thiết
quy nạp tồn tại x1 , ..., xk sao cho
k
X
Ai xi = Ht
i=1

73
Vậy
k+1
X k
X
Ai x i = Ai xi + Ak+1 xk+1 = Ht + Ak+1 xk+1 = C
i=1 i=1

ii) Chứng minh bằng quy nạp.Giả sử đúng với k . Tồn tại t1 , ..., tk , tk+1 ∈ Z sao
cho
k+1
X
Ai t i = C
i=1

Xét phương trình


k
X
Ai xi = C − Ak+1 tk+1 (5)
i=1

Phương trình này có nghiệm (t1 , ..., tk ). Theo quy nạp (5) có vô số nghiệm.
Với mỗi nghiệm (x1 , ..., xk ) của (5) thì ((x1 , ..., xk , tk+1 ) là nghiệm của phương
trình
k+1
X
A i xi = C (6)
i=1

Do đó (6) có vô số nghiệm.
Có thể chứng minh được nghiệm tổng quát của (4) có dạng

x1 = α1 + Pk−1 Li1 ti
i=1

x2 = α2 + Pk−1 Li2 ti
i=1
...
 Pk−1
x = αk + Lik ti
k i=1

trong đó (α1 , . . . , αk ) là một nghiệm riêng. (Lij ), i = 1, ..., k − 1; j = 1, 2, . . . , k là


các hệ số nguyên và t1 , ..., tk−1 là các số nguyên tùy ý.
Tuy nhiên không có công thức tường minh của các hệ số (Lij ).
Ví dụ Giải phương trình

6x + 15y + 10x = 3 (7)

74
Giải Ta có (7) ⇔ 6(x+z)+15y +4z = 3. Đặt u = x+z ta có 15y +4z = 3−6u. Cố
định u.Ta thấy (−1, 4) là nghiệm riêng của 15y +4z = 1 do đó (−3+6u, 12−24u)
là nghiệm riêng của 15y + 4z = 3 − 6u. Từ đó

y = −3 + 6u + 4t; z = 12 − 24u − 15t, → x = u − z = −12 + 25u + 15t

Dễ kiểm tra mỗi bộ (x, y, z) có dạng trên là nghiệm. Vậy phương trình có
nghiệm 
x = −12 + 25u + 15t

y = −3 + 6u + 4t

z = 12 − 24u − 15t

C. Một số bài toán chon lọc


Bài toán 1 Cho a > 1. Chứng minh rằng

(am − 1, an − 1) = a(m,n) − 1

Giải Xét trường hợp (m, n) = 1. Ta phải chứng minh

(am − 1, an − 1) = a − 1

Đặt d = (am − 1, an − 1). Vì a − 1|am − 1, a − 1|an − 1 nên a − 1|d. Đảo lại giả sử
m > n. Theo hệ quả tồn tại u, v ∈ N ∗ để mu−nv = 1. Vì am ≡ 1 (mod d), an ≡ 1
(mod d) → amu ≡ 1 (mod d), anv ≡ 1 (mod d) → amu − anv ≡ 0 (mod d) Khi đó

d|(amu − anv = anv (amu−nv − 1) = anv (a − 1)

Vì a − 1|d nên (d, a) = 1 do đó d|a − 1.Vậy d = a − 1 Với (m, n) bất kỳ đặt


g = (m, n) ta có m = gm1 , n = gn1 . Đặt b = ag . Khi đó

(am − 1, an − 1) = (bm1 − 1, bn1 − 1) = b − 1 = a(m,n) − 1

75
Bài toán 2 Cho a > 1 và m, n ∈ N ∗ . Giả sử m = 2i r, n = 2j s với r, s là các số
lẻ.Chứng minh rằng

a(m,n) + 1 nếu i = j

(am + 1, an + 1) = 2 nếu i 6= j , a lẻ

1 nếu i 6= j , a chẵn

Giải

• Nếu m, n lẻ , (m, n) = 1 .Ta chứng minh

(am + 1, an + 1) = a + 1

Đặt d = (am +1, an +1). Vì a+1|am = 1, a = 1|an +1 nên a+1|d. Đảo lại giả
sử m > n.Theo hệ quả tồn tại u, v ∈ N ∗ để mu−nv = 1. Do m, n lẻ nên u, v
khác tính chẵn lẻ. Giả sử u lẻ, v chẵn. Ta có am ≡ −1 (mod d), an ≡ −1
(mod d) → amu ≡ −1 (mod d), anv ≡ 1 (mod d)amu + anv ≡ 0 (mod d).Khi
đó
d|(amu + anv = anv (amu−nv + 1) = anv (a + 1)

Vì a + 1|d nên (d, a) = 1 do đó d|a + 1.Vậy d = a + 1

• Nếu m, n lẻ . Đặt g = (m, n) ta có m = gm1 , n = gn1 . Đặt b = a(m,n) = ag .


Khi đó

(am + 1, an + 1) = (bm1 + 1, bn1 + 1) = b + 1 = a(m,n) + 1

i
• Nếu m = 2i r, n = 2i s với r, s là các số lẻ. Đặt b = a2 . Khi đó (am + 1, an +
i
1) = (br + 1, bs + 1) = b(r,s) + 1 = a2 (r,s) + 1 = a(m,n) + 1

• Nếu m, n khác tính chẵn lẻ, chẳng hạn m chẵn, n lẻ. Ta có am ≡ −1


(mod d), an ≡ −1 (mod d) → amn ≡ −1 (mod d), anm ≡ 1 (mod d) → −1 ≡

76
1 (mod d) → 2 ≡ 0 (mod d). Vậy d = 1 hoặc d = 2. Nếu a lẻ thì 2|d → d =
2. Nếu a chẵn thì d 6= 2 → d = 1.

• Nếu m = 2i r, n = 2j s với r, s là các số lẻ và i 6= j , chẳng hạn i > j . Đặt


j i−j
b = a2 → an = bs , am = b2 r = bt , ở đó t = 2i−j r chẵn. Khi đó nếu a lẻ
thì b lẻ do đó (am + 1, an + 1) = (bt + 1, bs + 1) = 2.Nếu a chẵn thì b chẵn
do đó (am + 1, an + 1) = (bt + 1, bs + 1) = 1

Bài toán 3 Cho A, B ∈ N ∗ , (A, B) = 1 . Ký hiệu A là tập các số n ∈ N ∗ sao


cho phương trình
Ax + By = n

không có nghiệm nguyên dương.Chứng minh rằng


i) max A = ab.
ii) Tìm |A|
Giải
/ A. Tồn tại x, y ∈ N ∗ sao
i) Chứng minh AB ∈ A: Giả sử trái lại AB ∈
cho Ax + By = AB → By = A(x − B) → A|By → A|y → y ≥ A. Tương
tự x ≥ B . Vậy AB = Ax + By ≥ 2AB . Mâu thuẫn. Tiếp theo ta chứng
minh nếu n > AB thì n ∈
/ A. Phương trình Ax + By = n có nghiệm là
x = x0 + Bt, y = y0 − At. Ta chứng minh tồn tại t ∈ Z sao cho x > 0, y > 0 →
−x0 /B < t < y0 /A ⇔ t ∈ (−x0 /B; y0 /A). Vì khoảng (−x0 /B; y0 /A) có độ dài
y0 /A+x0 /B = (Ax0 +By0 )/AB = n/AB > 1 nên tồn tại t ∈ Z, t ∈ (−x0 /B; y0 /A).
ii) Xét n 6 AB . Ta chứng minh n ∈ A nếu và chỉ nếu m = AB + A + B − n ∈
/ A.
Tồn tại 1 6 x 6 B sao cho Ax ≡ n (mod B) → n = Ax + By . Thành thử với
mỗi n ∈ N ∗ ta luôn có biểu diễn

n = Ax + By, 1 6 x 6 B, y ∈ n∗

77
Nếu n ∈ A thì y ≤ 0. Vậy m = AB + A + B − Ax − By = A(B + 1 − x) + B(1 − y) =
Ax1 + By1 ở đó x1 = B + 1 − x ≥ 1, y1 = 1 − y ≥ 1. Vậy m ∈
/ A. Đảo lại
/ A → m = Ax1 + By1 , x1 , y1 ∈ N ∗ . Nếu trái lại n ∈
giả sử m ∈ / A → n =
Ax2 + By2 , x2 , y2 ∈ N ∗ → AB = m − A − B + n = Ax1 + By1 − A − B + Ax2 + By2 =
A(x1 + x2 − 1) + B(y1 + y2 − 1) → AB ∈
/ A. Mâu thuẫn với i). Ta có với
1 6 n 6 A + B − 1 → n ∈ A. Xét n ∈ I = [A + B; AB]. Ánh xạ f : I → I
cho bởi f (n) = AB + A + B − n là một song ánh và n ∈ A ⇔ f (n) ∈
/ A .Vậy
|I ∩ A| = |I|/2 = (AB − A − B + 1)/2. Thành thử

AB − A − B + 1 AB + A + B − 1
|A| = A + B − 1 + =
2 2

Bài toán 4 Cho A1 , ..., Ak ∈ N ∗ , (A1 , ..., Ak ) = 1 . Ký hiệu A là tập các số


n ∈ N ∗ sao cho phương trình
k
X
Ak x k = n
i=1

không có nghiệm nguyên dương.Chứng minh rằng A là tập hữu hạn và

|A| 6 (k − 1)A1 ...Ak

Giải Ta chứng minh quy nạp rằng:


Nếu n > (k − 1)A1 ...Ak thì n ∈
/A
Với k = 2 khẳng định đúng (bài toán 3). Giả sử đúng với k . Không giảm tổng
quát giả sử A1 6 A2 6 ... 6 Ak+1 . Đặt d = (A1 , ..., Ak ), Bi = Ai /d. Khi đó
(d, Ak+1 ) = 1, (B1 , . . . , Bk ) = 1. Do (d, Ak+1 ) = 1 nên tồn tại xk+1 , y ∈ Z với
1 6 xk+1 6 d sao cho
dy + Ak+1 xk+1 = n

Ta chứng minh
y > (k − 1)B1 ...Bk (7)

78
Ta có y > (k−1)B1 ...Bk ⇔ dy > (k−1)dB1 ...Bk ⇔ n > (k−1)dB1 ...Bk +Ak+1 xk+1 .
Vì xk+1 6 d nên chỉ cần chứng minh

n > (k − 1)B1 ...Bk d + Ak+1 d (8)

Thật vậy do Ai = dBi → Ak+1 ≥ Ai ≥ d, Ai ≥ Bi (i = 1, 2, ..k) nên

A1 ...Ak Ak+1 ≥ A1 ...Ak d ≥ B1 ...Bk d

A1 ...Ak Ak+1 ≥ Ak Ak+1 ≥ dAk+1

→ kA1 ...Ak Ak+1 = (k − 1)A1 ...Ak Ak+1 + A1 ...Ak Ak+1

≥ (k − 1)B1 ...Bk d + Ak+1 d

→ n > kA1 ...Ak Ak+1 ≥ (k − 1)B1 ...Bk d + Ak+1 d

Vậy (8) do đó (7) được chứng minh. Theo quy nạp tồn tại x1 , ..., xk ∈ N ∗ sao
cho
k
X k
X
Bk xk = y → Ak xk = dy
i=1 i=1
k
X
→ Ak xk + Ak+1 xk+1 = dy + Ak+1 xk+1 = n
i=1
k+1
X
→ Ak x k = n
i=1

Đặt

M (A1 , . . . , Ak ) = max A

G(A1 , . . . , Ak ) = |A|

Ở bài toán 3 ta có

M (A1 , A2 ) = A1 A2
A1 A 2 + A 1 + A2 − 1
G(A1 , A2 ) =
2

79
Từ bài toán 3 và 4 suy ra M (A1 , . . . , Ak ) 6 (k − 1)A1 ...Ak và dấu bằng xảy ra
với k = 2. Tuy nhiên với k > 2 dấu bằng không xẩy ra . Biểu thức tưởng minh
biểu diễn M (A1 , . . . , Ak ) và G(A1 , . . . , Ak ) theo A1 , ..., Ak vẫn chưa biết trong
trường hợp tổng quát.
Bài toán 5 Giả sử A = bc, B = ca, C = ab trong đó a, b, c ∈ N ∗ đôi một nguyên
tố cùng nhau. Chứng minh rằng

i)M (A, B, C) = 2 ABC

2 ABC + A + B + C − 1
ii)G(A, B, C) =
2
√ √
Giải i) Chứng minh 2 ABC ∈ A: Giả sử trái lại 2 ABC ∈
/ A. Tồn tại x, y, z ∈

N ∗ sao cho Ax+By+Bz = 2 ABC ⇔ bcx+acy+abz = 2abc → abz = 2abc−c(bx+
ay) → c|abz → c|z → z ≥ c. Tương tự y ≥ b; x ≥ a. Vậy 2abc = bcx + cay + abz ≥

3abc. Mâu thuẫn. Tiếp theo ta chứng minh nếu n > 2 ABC = 2abc thì n ∈ / A.
Do n > abc, (ab, c) = 1 nên theo bài toán 3 tồn tại 1 6 z 6 c, t ∈ N ∗ sao cho
ct + abz = n. Ta có ct = n − abz > 2abc − abc = abc → t > ab. Theo bài toán 3
tồn tại x, y ∈ N ∗ sao cho t = bx + ay . Từ đó n = ct + abz = c(bx + ay) + abz =
bcx + acy + abz → n ∈
/ A.

ii) Xét n 6 2 ABC = 2abc. Ta chứng minh n ∈ A nếu và chỉ nếu m =
/ A. Vì {bcx}ax=1 là hệ đầy đủ (mod a) nên tồn tại
2abc + bc + ca + ab − n ∈
1 6 x 6 a sao cho bcx ≡ n (mod a) → n = bcx + at, t ∈ Z .Vì {cy}by=1 là hệ đầy
đủ (mod b) nên tồn tại 1 6 y 6 b sao cho cy ≡ t (mod b) → t = cy + bz, z ∈ Z .
Thành thử với mỗi n ∈ N ∗ có biểu diễn

n = bcx + acy + abz 1 6 x 6 a, 1 6 x 6 b, z ∈ Z

Nếu n ∈ A thì z ≤ 0. Vậy m = 2abc + bc + ca + ab − n = 2abc + bc + ca + ab −


bcz − acy − abz = bc(a + 1 − x) + ac(b + 1 − y) + ab(1 − z) = Ax1 + By1 + Cz1 ở
đó x1 = a + 1 − x ≥ 1, y1 = b = 1 − 1 ≥ 1, z1 = 1 − z ≥ 1. Vậy m ∈
/ A.

80
/ A → m = bcx1 + acy1 + abz1 , x1 , y1 , z1 ∈ N ∗ . Nếu trái lại
Đảo lại giả sử m ∈
/ A → n = bcx2 + acy2 + abz2 , x2 , y2 , z2 ∈ N ∗ → 2abc = m − bc − ca − ab + n =
n∈
bcx1 + acy1 + abz1 − bc − ca − ab + bcx2 + cay2 + abz2 = bc(x1 + x2 − 1) + ca(y1 +
y2 − 1) + ab(z1 + z2 − 1) → 2abc ∈
/ A. Mâu thuẫn với i).
Ta có với 1 6 n 6 bc + ca + ab − 1 → n ∈ A. Xét n ∈ I = [bc + ca + ab; 2abc].
Ánh xạ f : I → I cho bởi f (n) = 2abc + bc + ca + ab − n − n là một song ánh và
n ∈ A ⇔ f (n) ∈
/ A .Vậy |I ∩ A| = |I|/2 = (2abc − bc − ca − ab + 1)/2. Thành thử

2abc − bc − ca − ab + 1
|A| = bc + ca + ab − 1 +
2
2abc + bc + ca + ab − 1
=
√ 2
2 ABC + A + B + C − 1
=
2

2.3 Bài tập

1. Cho a, b ∈ N ∗ với (a, b) = 1. Chứng minh rằng phương trình ax + by = 1


có vô số nghiệm (x, y) với (x, a) = (y, b) = 1

2. Tìm tất cả nghiệm nguyên của phương trình

6x + 45y + 6z − 10t = 13

3. Cho các số nguyên dương a, b thỏa mãn điều kiện 5a ≥ 7b. Chứng tinh
rằng tồn tại các số tự nhiên x, y, z, t sao cho x + 2y + 3z + 7t = a và
y + 2z + 5t = b.

4. Chứng minh rằng


i) M (2, 4, 5) = 14
ii) M (7, 8, 9) = 44

81
5. Chứng minh rằng nếu A 6 B 6 C thì

M (A, B, C) 6 AC + B

6. Cho a1 , a2 , ..., ak là các số nguyên dương đôi một nguyên tố cùng nhau.
Đặt A = a1 ...ak và
A
Ai =
ai
Chứng minh rằng
p
M (A1 , ..., Ak ) = (k − 1) k−1 A1 A2 ...Ak

3 Phương trình Pell

3.1 Phương trình Pell loại 1

Phương trình Pell loại 1 là phương trình có dạng

x2 − Dy 2 = 1 (I)

trong đó d ∈ N ∗ và ta yêu cầu tìm nghiệm x, y ∈ N ∗ . Trong tiết này khi nói
đến nghiệm của (I) ta hiểu là nghiệm nguyên dương
Định lý 1(Điều kiện tồn tại nghiệm). Phương trình (I) có nghiệm nguyên
dương khi và chỉ khi D là số không chính phương.
Chứng minh Giả sử D = m2 . Khi đó

x2 − Dy 2 = x2 − m2 y 2 = 1 → (x − my)(x + my) = 1

→ x − my = x + my = 1 → x = 1, y = 0

Vậy (I) không có nghiệm nguyên dương.


Ngược lại giả sử Dlà số không chính phương. Ta có các bổ đề sau

82
Bổ đề 1 Cho α ∈
/ Q . Khi đó tồn tại vô số cặp số nguyên (h, k) vớik > 0 sao
cho
a − < 1
h
k k2
Chứng minh Ta sử dụng nhận xét đã biết sau: Với mỗiq ∈ N ∗ tồn tại cặp số
nguyên (h, k) với 1 6 k 6 q sao cho

a − < 1
h
k kq

Ký hiệu
h 1
A = {(h, k) : α − < 2 }.
k k
Ta phải chứng minh |A| = ∞. Giả sử trái lại |A| < ∞. Khi đó tồn tại  sao cho

α − hk >  với mọi (h, k) ∈ A. Chọn q ∈ N ∗ sao cho
1
< . (1)
q
Theo nhận xét trên tồn tại cặp số nguyên (h0 , k0 ) với 1 6 k0 6 q sao cho

α − < 1 6 1
h 0
(2)
k0 2
k0 q k0

Từ (2) ta có (h0 , k0 ) ∈ A → α − hk00 > . Nhưng 1
q ≥ 1
k0 q > α− h0
k0 > . Mâu
thuẫn với (1).
Bổ đề 2 Tồn tại vô số cặp số nguyên dương (x, y) sao cho

|x2 − Dy 2 | < 1 + 2 D

Chứng minh. Theo bổ đề 1 tồn tại vô số cặp số nguyên (x, y) sao cho


x 1
0 < D − < 2.
y y
Suy ra
√ √ √ √

+ D = − D + 2 D < 1 + 2 D
x x
y y 2 y

83
Vậy
√ √ x √ x √

|x − Dy | = |x − y D||x + y D| = y 2 − D + D
2 2
y y
1 1 √ 1 √
< y 2 2 ( 2 + 2 D) = 2 + 2 D
y y y

<1+2 D

Chứng minh định lý Từ bổ đề 2 (x, y) tồn tại vô số cặp số nguyên dương (x, y)
sao cho

|x2 − Dy 2 | < 1 + 2 D
√ √
Đặt I = [−1 − 2 d; 1 + 2 d]. Với mỗi k ∈ I ký hiệu

Ak = {(x, y) ∈ N ∗ : x2 − Dy 2 = k}

Do đó tồn tại k ∈ I để |Ak | = ∞. Suy ra tồn tại (x1 , y1 ) 6= (x1 , y1 ) ∈ Ak để

x1 ≡ x2 (mod |k|) y1 ≡ y2 (mod |k|)

x21 − Dy12 = x22 − Dy22 = k

Xét tích
√ √ √
(x1 − y1 D)(x2 + y2 D) = x1 x2 − Dy1 y2 + D(x1 y2 − x2 y1 ) (3)

x1 x2 − Dy1 y2 ≡ x21 − Dy12 ≡ 0 (mod |k|)

x1 y2 − x2 y1 ≡ x1 y1 − x1 y1 ≡ 0 (mod |k|)

Vậy tồn tại u, v ∈ Z sao cho

x1 x2 − Dy1 y2 = ku (4)
x1 y2 − Dy1 x2 = kv (5)

84
Từ (3), (4), (5) suy ra
√ √ √
(x1 − y1 D)(x2 + y2 D) = k(u + v D)
√ √ √
(x1 + y1 D)(x2 − y2 D) = k(u − v D)

Nhân hai đẳng thức trên với nhau và chú ý rằng (x1 , y1 ), (x2 , y2 ) ∈ Ak →
x21 − dy12 = x22 − dy22 = k ta được

k 2 = k 2 (u2 − Dv 2 ) −→ u2 − Dv 2 = 1

Ta chứng minh u, v > 0. Rõ ràng u > 0. Nếu trái lại v = 0 thì u = ±1 →


√ √ √ √
(x1 − y1 D)(x2 + y2 D) = ±k = ±(x21 − Dy12 ) = ±(x1 − y1 D)(x1 + y1 D) →
√ √
x2 + y2 D = x1 + y1 D −→ x1 = x2 , y1 = y2 Ta có mâu thuẫn. Vậy (u, v) là
nghiệm nguyên dương của phương trình (I)
Định lý 2(Công thức nghiệm) Ký hiệu (a, b) là nghiệm nhỏ nhất của phương
trình
x2 − Dy 2 = 1

Khi đó dãy (xn , yn ) cho bởi


√ √
(a + b D)n + (a − b D)n
xn =
√ n 2 √
(a + b D) − (a − b D)n
yn = √
2 D
cho ta tất cả các nghiệm của (I).
Dãy nghiệm (xn , yn ) cũng có thể xác định theo công thức truy hồi sau

x0 = 1, x1 = a, xn+2 = 2axn+1 − xn (6)


y0 = 0, y1 = b, yn+2 = 2ayn+1 − yn (7)

Chứng minh. Ta có
√ √ √ √
xn + yn D = (a + b d)n , xn − yn D = (a − b D)n . (8)

85
Suy ra (x2n − Dyn2 ) = (a2 − Db2 )n = 1.
Đảo lại giả sử (x, y) là nghiệm bất kỳ của (I). Ta chứng minh tồn tại n ∈ N ∗
để x = xn ; y = yn . Vì D không chính phương nên điều này tương đương
Tồn tại n ∈ N ∗ để
√ √ √
x + y D = xn + yn D = (a + b D)n

Chứng minh bằng phản chứng. Giả sử trái lại


√ √
x + y D 6= (a + b D)n ∀n ∈ N ∗

Khi đó tồn tại m ∈ N ∗ sao cho


√ √ √
(a + b D)m < x + y D < (a + b D)m+1

Nhân hai vế với (a − b D)m ta được
√ √ √
1 < (x + y D)(a − b D)m < a + b D

Do (8) ta có
√ √ √ √
(x + y D)(a − b D)m = (x + y D)(xm − ym D)

= (xxm − Dyym ) + (xm y − ym x) D

=u+v D

ở đó u = xxm − Dyym , v = xm y − ym x. Vậy


√ √
1 < u + v d < a + b d. (9)

Ta có

u2 − Dv 2 = (xxm − Dyym )2 − D(xm y − ym x)2

= (x2 − Dy 2 )(x2m − Dym


2
)=1

86
√ √ √ √
Lại có x > y d, xm > ym d nên u > 0. Lại có (u − v D)(u + v D) = 1 và
√ √ √
1 < u + v d nên 0 < u − v d < 1 < u + v d → v > 0. Vậy (u, v) là nghiệm của
√ √
(I) do đó a 6 u, b 6 v → a + b d 6 u + v d trái với (9).Định lý được chứng
minh.
Từ định lý trên ta thấy việc tìm nghiệm của phương trình Pell (I) quy về tìm
nghiệm nhỏ nhất (a, b) của nó. Cách đơn giàn nhất là thử bằng tay: Thay lần
lượt y = 1, 2, .. vào biểu thức 1 + Dy 2 cho tới khi nào được số chính phương
thì dừng lại. Vì phương trình (I) có nghiệm nên chắc chắn quá trình này sẽ

dừng lại sau b phép thử. Khi đó nghiệm nhỏ nhất là (a, b) với a = 1 + Db2 .
Nếu nghiệm nhỏ nhất b này lớn thì cách thử này không khả thi. Thí dụ với
phương trình x2 − 61y 2 thì cặp nghiệm nhỏ nhất là

a = 1766319049

b = 226153980

Sau đây ta sẽ trình bày một thuật toán sử dụng liên phân số để tìm một
nghiệm nhỏ nhất (a, b) của (I).
Định nghĩa Cho a0 , a1 , a2 , ... là dãy số nguyên trong đó ai > 0, i ≥ 1. Đặt

Ck = [a0 ; a1 , ..., ak ]

Khi đó tồn tại giới hạn


lim Ck = α
k→∞

Ta gọiα là giá trị của liên phân số vô hạn [a0 ; a1 , a2 .., ] và viết

α = [a0 ; a1 , a2 , ...]

Định lý α = [a0 ; a1 , a2 , ...] là một số vô tỷ . Ngược lại mỗi số vô tỷ đều biểu


diễn duy nhất dưới dạng một liên phân số vô hạn.

87
Định nghĩa Ta gọi liên phân số vô hạn α = [a0 ; a1 , a2 , ...] là tuần hoàn nếu
dãy (an ) tuần hoàn kể từ một chỉ số nào đó tức là: Tồn tại các số nguyên
dương m và k sao cho với mọi n ≥ m ta có an = an+k . Số nguyên dương k được
gọi là chu kỳ của liên phân số α = [a0 ; a1 , a2 , ...] . Khi đó ta viết

α = [a0 ; a1 , a2 , ..., am−1 , am , am+1 , ..., am+k−1 ]



Nếu D là số không chính phương, biểu diễn liên phân số của D được cho
bởi

Định lý Biểu diễn liên phân số của D là tuần hoàn và có dạng

D = [a; a1 , a2 , ..., an , 2a]

với a = [ D] . Hơn nữa có công thức tường minh để xác định dãy (a1 , . . . , an ).
Chú ý rằng dãy (a1 , . . . , an ) là đối xứng tức là

a1 = an , a2 = an−1 , ...

Ví dụ

23 = [4; 1, 3, 1, 8]

29 = [5; 2, 1, 1, 2, 10]

31 = [5; 1, 1, 3, 5, 3, 1, 1, 10]

46 = [6; 1, 2, 1, 1, 2, 6, 2, 1, 1, 2, 1, 12]

76 = [8; 1, 2, 1, 1, 5, 4, 5, 1, 1, 2, 1, 16]

97 = [9; 1, 5, 1, 1, 1, 1, 1, 1, 5, 1, 18]

Định lý 3 Cho phương trình Pell

x2 − Dy 2 = 1. (I)

88

• Biểu diễn D thành liên phân số


D = [a; a1 , a2 , ..., an , 2a]

• Nếu chu kỳ n là số chẵn ta tính giản phân thứ n − 1

pn−1
Cn−1 =
qn−1

Khi đó (pn−1 , qn−1 ) là nghiệm nhỏ nhất của (I)

• Nếu chu kỳ n là số lẻ ta tính giản phân thứ 2n − 1.

p2n−1
C2n−1 =
q2n−1

Khi đó (p2n−1 , q2n−1 ) là nghiệm nhỏ nhất của (I)


Ví dụ Tìm nghiệm nhỏ nhất của phương trình x2 − 14y 2 = 1. Ta có 14 =
[3; 1, 2, 1, 6]. Chu kỳ n = 4 là số chẵn. Vậy ta tính giản phân

1
C3 = [3, 1, 2, 1] = 3 +
1
1+
1
2+
1
15
=
4

Vậy nghiệm nhỏ nhất là (14, 4)



Ví dụ Tìm nghiệm nhỏ nhất của phương trình x2 − 13y 2 = 1. Ta có 13 =
[3; 1, 1, 1, 1, 6] = [3, 1, 1, 1, 1, 6, 1, 1, 1, 1, 6, .... Chu kỳ n = 5 là số lẻ. Vậy ta tính

89
giản phân

C9 = [3, 1, 1, 1, 1, 6, 1, 1, 1, 1]
1
=3+
1
1+
. 1
1 + .. +
1
1+
1
649
=
180

Vậy nghiệm nhỏ nhất là (649, 180)


Trở lại phương trình x2 − 61y 2 = 1. Ta có


76 = [7; 1, 4, 3, 1, 2, 2, 1, 3, 4, 1, 14]

Chu kỳ n = 11 là số lẻ. Ta tính giản phân

C21 = [7, 1, 4, 3, 1, 2, 2, 1, 3, 4, 1, 14, 1, 4, 3, 1, 2, 2, 1, 3, 4, 1]


1
=7+
1
1+
1
4+
. 1
3 + .. +
1
4+
1
1766319049
=
226153980

Vậy nghiệm nhỏ nhất là (1766319049, 226153980)

90
3.2 Phương trình Pell loại 2

Phương trình Pell loại 2 là phương trình

x2 − Dy 2 = −1 (II)

ở đó D ∈ N ∗ . Nghiệm của (II) luôn được hiểu là nghiệm nguyên dương.


Định lý 1 Điều kiện cần để (II) có nghiệm là D không là số chính phương
và D không có ước nguyên tố dạng 4k + 3.
Chứng minh i)) Nếu D = m2 thì (II) trở thành (my−x)(my+x) = 1 → my−x =
my + x = 1 → x = 0 Vậy (II) vô nghiệm.
ii) Nếu D có ước nguyên tố p = 4k + 3 : Giả sử (x, y) là nghiệm. Khi đó
x2 + 1 = Dy 2 → p|x2 + 1. Vìp = 4k + 3 nênp|1. Mâu thuẫn.
Tuy nhiên , đây không là điều kiện đủ.
Định lý 2 Nếu D = p là số nguyên tố thì (II) có nghiệm khi và chỉ khi p = 2
hoặc p 6= 4k + 3.
Chứng minh. Nếu (II) có nghiệm thì từ Định lý 1 suy ra p = 2 hoặc p 6= 4k +3.
Đảo lại nếu p = 2 thì phương trình x2 − 2y 2 = −1 có nghiệm (x, y) = (1; 1). Xét
p ≡ 1 (mod 4). Xét phương trình Pell loại I

x2 − Dy 2 = 1 (10)

Gọi (a, b) là nghiệm nhỏ nhất của (10). Ta có a2 − 1 = pb2 . Nếu a chẵn thì b lẻ
do đó b2 ≡ 1 (mod 4) → a2 ≡ 1 + p ≡ 2 (mod 4). Mâu thuẫn . Vậy a lẻ, b chẵn.
Đặt a = 2a1 + 1, b = 2b1 . Ta có (a − 1)(a + 1) = pb2 ⇔ a1 (a1 + 1) = pb21 . Do p
nguyên tố và (a1 , a1 + 1) = 1 nên a1 = u2 , a1 + 1 = pv 2 hoặc a1 = pu2 , a1 + 1 = v 2 .
Nếu a1 = u2 , a1 + 1 = pv 2 → u2 − pv 2 = −1. Vậy (II ) có nghiệm (u, v).
Nếu a1 = pu2 , a1 + 1 = v 2 → v 2 − pu2 = 1. Vậy (v, u) là nghiệm của (10)). Suy
ra v ≥ a → a1 + 1 = v 2 ≥ v ≥ a = 2a1 + 1. Mâu thuẫn .

91
Định lý 3 Gọi (a, b) là nghiệm nhỏ nhất của (10) . Khi đó (II) có nghiệm
khi và chỉ khi 
a = x2 + Dy 2
(11)
b = 2xy

có nghiệm nguyên dương.


Hơn nữa nếu (11) hệ có nghiệm nó sẽ có nghiệm duy nhất.Nghiệm duy nhất
(x1 , y1 ) này chính là nghiệm nhỏ nhất của (II)
Chứng minh. 1) Giả sử (II) có nghiệm. Gọi (x1 , y1 ) là nghiệm nhỏ nhất của
(II). Đặt u = x21 + Dy12 , v = 2x1 , y1 . Ta chứng minh u = a, v = b do đó (x1 , y1 )
chính là nghiệm của (11). Ta có u2 − Dv 2 = (x21 − Dy12 )2 = 1 Vậy (u, v) là
nghiệm của (10) . Suy ra u ≥ a; v ≥ b. Ta có

(a2 − Db2 )(x1 − Dy12 ) = 1(−1) = −1

⇔ (ax1 − Dby1 )2 − D(ay1 − bx1 )2 = −1.

Dễ thấy (ax1 − Dby1 )2 > 0, (ay1 − bx1 )2 > 0. Do (x1 , y1 ) là nghiệm của ( II) nên

(ax1 − Dby1 )2 ≥ x21 ⇔ a2 x21 + D2 b2 y12 − x21 ≥ 2abDx1 y1

→ x21 (a2 − 1) + D2 b2 y12 ≥ 2abDx1 y1

→ x21 Db2 + D2 b2 y12 ≥ 2abDx1 y1

→ b(x21 + Dy12 ) ≥ 2ax1 y1 → bu ≥ av

→ b2 u2 ≥ a2 v 2 → b2 (Dv 2 + 1) ≥ v 2 (Db2 + 1)

→b≥v→b=v→u=a

2) Đảo lại giả sử (u, v) là nghiệm của (11). Ta có a2 − Db2 = 1 ⇔ (u2 +


Dv 2 )2 − D(2uv)2 = (u2 − Dv 2 )2 = 1. Vậy u2 − Dv 2 = 1 hoặc u2 − Dv 2 = −1. Nếu
u2 − Dv 2 = 1 thì (u, v) là nghiệm của (10) do đó u ≥ a = u2 + Dv 2 . Mâu thuẫn.
Vậy u2 − Dv 2 = −1 do đó (II) có nghiệm (u, v)

92
Tiếp theo ta chứng minh (u, v) là nghiệm nhỏ nhất của (II).Gỉả sử (x1 , y1 )
là nghiệm nhỏ nhất của (II). Theo chứng minh ở 1) ta có a = u2 + Dv 2 =

x21 + Dy12 ; b = 2uv = 2x1 y1 → u2 + Dv 2 + 2uv = x21 + Dy12 + 2x1 y1 → (u + v D)2 =

(x1 + y1 D)2 → u = x1 ; v = y1 . Định lý được chứng minh.
Ví dụ là một áp dụng của định lý 2. Nó cũng chỉ ra rằng điều kiện của định
lý 1 chỉ là điều kiện cần.
Ví dụ Chứng minh rằng phương trình x2 − 34y 2 = −1 vô nghiệm ( Ở đây
34 = 2.17 không là số chính phương và cũng không có ước nguyên tố dạng
4k + 3).
Giải Phương trình x2 − 34y 2 = 1 có nghiệm nhỏ nhất là(a; b) = (35; 6). Xét hệ

35 = x2 + 34y 2
6 = 2xy

Từ phương trình thứ nhất của hệ suy ra (x; y) = (1; 1). Tuy nhiên (1; 1) không
thỏa mãn phương trình thứ hai. Vậy hệ vô nghiệm do đó theo định lý 2 phương
trình x2 − 34y 2 = −1 vô nghiệm.
Ta thừa nhận định lý sau
Định lý 3 Phương trình Pell loại 2

x2 − Dy 2 = −1.


có nghiệm khi và chỉ khi trong biểu diễn D thành liên phân số


D = [a; a1 , a2 , ..., an , 2a]

chu kỳ n là số lẻ. Trong trường hợp đó (pn−1 , qn−1 ) là nghiệm nhỏ nhất của
phương trình,ở đó
pn−1
Cn−1 =
qn−1

93
là giản phân thử n − 1.
Ví dụ

i)Xét phương trình x2 − 13y 2 = −1. Ta có 13 = [3; 1, 1, 1, 1, 6]. Chu kỳ n = 5
là số lẻ. Vậy phương trình có nghiệm. Ta tính giản phân

C4 = [3, 1, 1, 1, 1]
1
=3+
1
1+
1
1+
1
1+
1
3 18
=3+ =
5 5

Vậy nghiệm nhỏ nhất là (18, 5)



ii)Xét phương trình x2 − 34y 2 = −1. Ta có 34 = [5; 1, 4, 1, 10]. Chu kỳ n = 4
là số chẵn. Vậy phương trình vô nghiệm.
Định lý 4 (Công thức nghiệm) Giả sử phương trình Pell loại 2

x2 − Dy 2 = −1 (II)

có nghiệm. Gọi (α, β) là nghiệm nhỏ nhất của nó. Khi đó dãy (xn , yn )∞
n=1 cho
bởi

√ √
(α + β D)2n−1 + (α − β D)2n−1
xn =
√ 2n−1 2 √
(α + β D) − (α − β D)2n−1
yn = √
2 D

94
cho ta tất cả các nghiệm của (II).
Chứng minhGiả sử (xn , yn ) cho bởi công thức trên. Khi đó
√ √
xn + yn D = (α + β D)2n−1
√ √
xn − yn D = (α − β D)2n−1

→ x2n − Dyn2 = (α2 − Dβ 2 )2n−1 = −1

Ngược lại giả sử (x, y) là một nghiệm của (II). Ta có


√ √ √
(x + y D)(α + β D) = (xα + Dyβ) + (yα + xβ) D

=s+t D

ở đó s = xα + Dyβ, t = yα + xβ
→ s2 − Dt2 = (xα + Dyβ)2 − D(yα + xβ)2

= (x2 − Dy 2 )(α2 − Dβ 2 ) = (−1)(−1) = 1

Vậy (s, t) là nghiệm của phương trình Pell loại 1 s2 − Dt2 = 1 . Goi (a, b) là
nghiệm nhỏ nhất của nó.Theo công thức nghiệm của phương trình Pell loại 1
và định lý 3 tồn tại n ∈ N ∗ sao cho
√ √ √ √
(x + y D)(α + β D) = s + t D = (a + b D)n
√ √
= (α2 + Dβ 2 + 2αβ D)n = (α + β D)2 )n

= (α + β D)2n
√ √ √
→ x + y D = (α + β D)2n−1 = xn + yn D

→ x = xn , y = y n

3.3 Một số bài toán chọn lọc

Bài toán 6 Số nguyên dương S được gọi là số Heron nếu nó là diện tích
của một tam giác có ba cạnh là ba số nguyên liên tiếp. Chứng minh rằng S

95
là số Heron khi và chỉ khi S khi là số hạng của dãy (Sn ), n ≥ 1 xác định bởi

S0 = 0, S1 = 6, S2 = 84, Sn+2 = 14Sn+1 − Sn

Giải Gọi S là diện tích của tam giác có ba cạnh là x − 1, x, x + 1 với


x > 2, x ∈ N ∗ . Theo công thức Heron
1 p
S = x 3(x2 − 4) −→ 16S 2 = 3x2 (x2 − 4). (12)
4
Vậy S là số Heron khi và chỉ khi phương trình (12) có nghiệm nguyên dương
(S, x). Dễ thấy x phải chẵn . Đặt x = 2y ta có

16S 2 = 3x2 (x2 − 4) ⇔ S 2 = 3y 2 (y 2 − 1)


p
−→ S = y 3(y 2 − 1) −→ 3(y 2 − 1) = h2

−→ h = 3z −→ 3(y 2 − 1) = 9z 2

−→ y 2 − 3z 2 = 1, S = 3yz

Ngược lại nếu (y, z) là nghiệm của phương trình Pell

y 2 − 3z 2 = 1 (13)

thì x = 2y, y > 1, S = 3yz là nghiệm của (12)(12) Nghiệm nhỏ nhất của (13)
là (2, 1) Vậy tất cả nghiệm của (13)(yn , zn ) cho bởi
√ n √
(2 +3) + (2 − 3)n
yn =
√ n 2 √
(2 + 3) − (2 − 3)n
zn = √
2 3

Do đó

3 √ √
Sn = 3yn zn = ((7 + 4 3)n − (7 − 4 3)n )
4
→ Sn+2 = 14Sn+1 − Sn , S0 = 0, S1 = 6, S2 = 84

96
Bài toán 7 Tìm tất cả các số nguyên dương n sao cho 2n + 1 và 3n + 1 đều
là số chính phương.
Giải Vì (2n + 1, 3n + 1) = 1 nên 2n + 1 và 3n + 1 đều là số chính phương khi
và chỉ khi (2n + 1)(3n + 1) = y 2 , y ∈ N ∗ Suy ra (12n + 5)2 − 24y 2 = 1 . Đặt
x = 12n + 5 ta có x2 − 24y 2 = 1. Gọi (xk , yk ) là nghiệm của nó. Nghiệm nhỏ
nhất của phương trình Pell này là (5, 1). Do đó (xk ) cho bởi hệ thức

x0 = 1, x1 = 5, xk+2 = 10xk+1 − xk .

Dễ chứng minh xk ≡ 5 (mod 12) khi và chỉ khi k lẻ. Vậy n = nk , k ≥ 1 ở đó


x2k+1 − 5
nk = .
12
Ta xác định hệ thức truy hồi của dãy (nk )
Đặt uk = x2k+1 = 12nk + 5. Ta có

x2k+3 = 10x2k+2 − x2k+1 = 10(10x2k+1 − x2k ) − x2k+1

= 100x2k+1 − 10x2k − x2k+1 = 99x2k+1 − x2k+1 − x2k−1

= 98x2k+1 − x2k−1

→ uk+2 = 98uk+1 − uk

⇔ 12nk+2 + 5 = 98(12nk+1 + 5) − 12nk − 5

nk+2 = 98nk+1 − nk + 40

với n1 = 40, n2 = 3960


Bài toán 8 Cho dãy (xn , yn ) xác định như sau (x0 , y0 ) = (0, 1), (x1 , y1 ) = (3, 5)
và 
xn+1 = 3xn + 2yn + 1
(14)
yn+1 = 4xn + 3yn + 2
Chứng minh rằng (xn , yn ) là tất cả các nghiệm nguyên dương của phương
trình
x2 + (x + 1)2 = y 2

97
Giải Phương trình đã cho tương đương với

(2x + 1)2 − 2y 2 = −1 (15)

Đặt u = 2x + 1, → u2 − 2y 2 = −1 . Nghiệm nhỏ nhất của phương trình này là


(1, 1). Do vậy dãy nghiệm (un , yn ) cho bởi

√ √
(1 + 2)2n+1 + (1 − 2)2n+1
un =
√ 2 √
(1 + 2)2n+1 − (1 − 2)2n+1
yn = √
2 2

Từ đó

u0 = 1, u1 = 7, uk+2 = 6uk+1 − uk

y0 = 1, y1 = 5, yk+2 = 6yk+1 − yk

Ta có un = 2xn + 1 → 2xk+2 + 1 = 6(2nk+1 + 1) − 2nk − 1 → x0 = 0, x1 = 3, xk+2 =


6xk+1 − xk + 2. Thành thử dãy nghiệm (xn , yn ) của (15) cho bởi

x0 = 0, x1 = 3, xn+2 = 6xn+1 − xn + 2

y0 = 1, y1 = 5, yn+2 = 6yn+1 − yk

Thành thử ta chỉ cần chứng minh dãy (14) thỏa mãn quan hệ

xn+2 = 6xn+1 − xn + 2

yn+2 = 6yn+1 − yk

98
Thật vậy đặt zn = 2xn + 1. Khi đó dễ kiểm tra

zn+1 = 3zn + 4yn

yn+1 = 2zn + 3yn

→ zn+2 = 3zn+1 + 4yn+1 = 3zn+1 + 8zn + 12yn

= 3zn+1 + 8zn + 3zn+1 − 9zn = 6zn+1 − zn

→ 2xn+2 + 1 = 6(2xn+1 + 1) − 2xn − 1

→ xn+2 = 6xn+1 − xn + 2

Tương tự yn+2 = 6yn+1 − yk

3.4 Bài tập

n(n+1)
1. Tìm tất cả các số nguyên dương n sao cho 2 là số chính phương.

2. Tìm tất cả các số nguyên dương n sao cho

12 + 22 + 32 + · · · + n2
n

là số chính phương

3. Cho n ∈ N ∗ thỏa mãn: 3n + 1 và 4n + 1 đều là số chính phương. Chứng


minh rằng n chia hết cho 56 .

4. Tìm cặp số nguyên dương (m, n) thỏa mãn 1+2+· · ·+m = (m+1)+· · ·+n

99
4 Phương trình Pitago

4.1 Phương trình Pitago

Phương trình Pitago là phương trình sau

x2 + y 2 = z 2 (16)

Bộ ba số nguyên dương (x, y, z) thỏa mãn (16) được gọi là bộ ba Pitago. Bộ


ba Pitago (x, y, z) được gọi là nguyên thủy nếu (x, y, z) = 1.
Bổ đề Nếu (x, y, z) là bộ ba Pitago nguyên thủy thì (x, y) = (x, z) = (y, z) = 1
và x, y không cùng tính chẵn lẻ .
Chứng minh Giả sử (x, y) > 1 và p là ước nguyên tố chung của x, y thì p2 |x2 +
y 2 = z 2 → p|z → p|(x, y, z). Mâu thuẫn .Vậy (x, y) = 1. Tương tự (x, z) =
1, (y, z) = 1.
Vì (x, y) = 1 nên x, y không cùng chẵn. Nếu x, y cùng lẻ thì x2 ≡ y 2 ≡ 1
(mod 4) → z 2 ≡ 2 (mod 4). Mâu thuẫn. Vậy x, y không cùng tính chẵn lẻ.
Định lý Tập hợp tất cả các bộ ba Pitago (x, y, z) được cho bởi công thức

x = t(m2 − n2 )

y = 2tmn

z = t(m2 + n2 )

hoặc

x = 2tmn

y = t(m2 − n2 )

z = t(m2 + n2 )

100
trong đó t, m, n ∈ N ∗ , m > n, (m, n) = 1 và m, n khác tính chẵn lẻ.
Chứng minh Dễ kiểm tra nếu (x, y, z) cho bởi công thức trên thì x2 + y 2 = z 2 .
Ngược lại giả sử (x, y, z) là bộ ba Pitago.
i) Trường hợp (x, y, z) = 1 : Theo bổ đề x, y khác tính chẵn lẻ. Không giảm
tổng quát giả sử x lẻ, y chẵn do đó z lẻ. Ta có y 2 = z 2 − x2 = (z + x)(z − x).
Vì x, z lẻ nên z + x = 2k, z − x = 2t, y = 2h, k, h ∈ N ∗ . Thay vào (16) ta
đượch2 = kt.Ta có z = k + t, x = k − t. Theo bổ đề (x, z) = 1 → (k, t) = 1. Vậy
tồn tại m, n sao cho k = m2 , t = n2 → h = mn. Vậy x = k − t = m2 − n2 , y =
2h = 2mn, z = k + h = m2 + n2 . Giả sử (m, n) > 1 và p là ước nguyên tố chung
của m, n p|m, p|n thỉ p2 |m2 , p2 |n2 → p|x, p|z . Mâu thuẫn. Vậy (m, n) = 1. Vì z
lẻ nên m, n khác tính chẵn lẻ.
ii) Trường hợp (x, y, z) là bộ ba Pitago bất kỳ: Đặt t = (x, y, z) → x = tx1 , y =
ty1 , z = tz1 , (x1 , y1 , z1 ) = 1, x21 + y12 = z12 . Vậy (x1 , y1 , z1 ) là bộ ba Pitago nguyên
thủy. Áp dụng i) ta có x = tx1 = t(m2 − n2 ); y = ty1 = 2tmn; z = tz1 =
t(m2 + n2 ).Định lý được chứng minh

4.2 Một số bài toán chọn lọc

Bài toán 9 Chứng minh rằng phương trình

x4 + y 4 = z 2 (17)

Giải. Giả sử (17) có nghiệm. Gọi (x0 , y0 , z0 ) là nghiệm sao cho z0 nhỏ nhất.
Ta có
i) (x0 , y0 ) = 1.Thật vậy gọi p là ước nguyên tố chung của x0 , y0 . Ta có p4 |x40 +
y04 = z02 → p2 |z0 → x0 = px1 , y0 = py1 , z0 = p2 z1 → x41 + y14 = z12 . Vậy (x1 , y1 , z1 )
là nghiệm của (17) với z1 < z0 . Mâu thuẫn

101
ii) Vậy (x20 , y02 , z0 ) = 1. Giả sử y0 chẵn, x0 lẻ. Theo định lý ta có

x20 = m2 − n2 (18)
y02 = 2mn

z0 = m2 + n2

trong đó t, m, n ∈ N ∗ , m > n, (m, n) = 1 Từ (19)suy ra (x0 , n, m) là bộ ba Pitago


nguyên thủy.Theo định lý ta có

x 0 = a2 − b2

n = 2ab

m = a2 + b2 (19)

trong đó a, b ∈ N ∗ a > b, (a, b) = 1.


Đặt y0 = 2y1 . Từ (19)ta có y02 = 4y12 = 2mn = 4ab(a2 + b2 ) → y12 = ab(a2 + b2 ) =
abm. Lại có (a, b) = 1 → (a, m) = (b, m) = 1 → a = a21 , b = b21 , m = m21 .
Thay vào (19) ta cóm21 = a41 + b41 . Vậy (a1 , b1 , m1 ) là nghiệm của (17) với
m1 6 m21 = m < m2 + n2 = z0 . Mâu thuẫn với cách chọn (x0 , y0 , z0 ) .
Bài toán 10 Chứng minh rằng không tồn tại hai số nguyên dương x, y mà
tổng bình phương của chúng và hiệu bình phương của chúng đề là số chính
phương.
Giải Giả sử trái lại hệ 
 x2 + y 2 = z2
(20)
x2 − y 2 = t2

có nghiệm nguyên dương (x, y, z, t) và gọi (x0 , y0 , zo , t0 ) là nghiệm mà x20 + y02


bé nhất.
+)Ta có (x0 , y0 ) = 1. Thật vậy giả sử trái lại gọi p là ước nguyên tố chung của
x0 , y0 . Ta có x0 = px1 , y0 = py1 → z0 = pz1 , t0 = pt1 và (x1 , y1 , z1 , t1 ) là nghiệm

102
của (20). Mà x20 + y02 = p2 (x21 + y12 ) > x21 + y12 .Mâu thuẫn .
+)Từ (20) suy ra z02 + y02 = 2x20 . Vậy z0 , t0 có cùng tính chẵn lẻ. Đặt
z0 + t0 z0 − t0
u= , v=
2 2
Suy ra
u2 + v 2 = x20 (21)

Ta có (u, v) = 1. Thật vậy giả sử trái lại gọi p là ước nguyên tố chung của u, v .
Khi đó p2 |x20 → p|x0 , p|u + v = z0 → p2 |z02 − x20 = y02 → p|y0 → (x0 , y0 ) > 1. Mâu
thuẫn.
+) Từ (21) và định lý ta suy ra

u = (m2 − n2 )

v = 2mn

x0 = (m2 + n2 )

hoặc

u = 2mn

v = (m2 − n2 )

x0 = (m2 + n2 )

trong đó t, m, n ∈ N ∗ , m > n, (m, n) = 1, m, n khác tính chẵn lẻ. Trong cả hai


trường hợp ta đều có uv = 2mn(m2 − n2 ). Ta có

2y02 = z02 − t20 = (u + v)2 − (u − v)2 = 4uv

= 8mn(m2 − n2 )

→ y02 = 4mn(m2 − n2 ) → y0 = 2k

k 2 = mn(m2 − n2 ) (22)

103
Vì (m, n) = 1 nên dễ thấy (m, n) = (m, m2 − n2 ) = (n, m2 − n2 ) = 1. Từ (22)
suy ra tồn tại a, b, c ∈ N ∗ để m = a2 , n = b2 , m2 − n2 = c2 . Vì (m, n) = 1, m, n
khác tính chẵn lẻ nên (m + n, m − n) = 1. Mà (m + n)(m − n) = c2 nên tồn tại
r, s ∈ N ∗ để

m + n = s2
m − n = r2

Suy ra 
 a2 + b 2 = s2
 a2 − b2 = r2

Vậy (a, b, s, r) là nghiệm của (20). Mặt khác


z0 + t0
a2 + b2 = m + n 6 2m 6 2mn = u = < z0 6 z02 = x20 + y02
2
Ta có mâu thuẫn.

4.3 Bài tập

1. Chứng minh rằng phương trình

x4 − y 4 = z 2

không có nghiệm nguyên dương

2. Giải phương trình x4 − 2y 4 = 14

3. Giải phương trình x4 − 2y 4 = −1

4. Hỏi có tồn tại hay không hình chóp tứ giác đều mà các cạnh,diện tích
toàn phần và thể tích của nó đều là các số nguyên.

104
Bài toán tô màu đồ thị

Vũ Đình Hòa
Trường ĐHSP Hà Nội

1 Định nghĩa đồ thị và ví dụ

1.0.1 Định nghĩa đồ thị

Khái niệm đồ thị trong cuốn sách này là một mô hình toán học có thể
dùng để giải quyết khá nhiều bài toán và vấn đề toán học. Một đồ thị có thể
hiểu đơn giản là một hệ thống các đỉnh và các cạnh nối các đỉnh này với nhau.

Hình 1: Bản đồ khu vực Cầu giấy.

105
Ví dụ. Một bản đồ giao thông là một đồ thị. Các đỉnh của đồ thị là
các nút giao thông (ngã ba, ngã tư đường...), còn các cạnh của nó là các con
đường giao thông nối các nút giao thông với nhau. Trên bản đồ giao thông
các cạnh (các đường đi của nó) có thể có hướng (nếu là đường một chiều)
hoặc không có hướng (nếu nó không phải là đường một chiều, xem hình 1).
Bản đồ trong hình 1 có thể biểu diễn thành một sơ đồ các đường đi và nút
giao thông . Bản đồ giao thông trên có thể biểu diễn thành đồ thị với các đầu
ngã tư và ngã ba là các đỉnh của đồ thị, còn các phố nối chúng là cạnh.

Định nghĩa 1. Một đồ thị được hiểu là một bộ hai tập hợp hữu hạn: Tập hợp
đỉnh và tập hợp cạnh nối các đỉnh này với nhau.

Ví dụ. Người ta hay biểu diễn các công thức hóa học dưới dạng đồ thị.
Trong cuốn sách này chúng ta chỉ nghiên cứu đồ thị hữu hạn, tức là những
đồ thị mà tập đỉnh và tập cạnh của nó là tập hợp hữu hạn mà thôi. Thông
thường chúng ta hay kí hiệu một đồ thị bởi chữ G (chữ cái đầu của từ "graph"
- tức là đồ thị - trong tiếng Đức, ngôn ngữ dùng để viết cuốn sách đầu tiên
về lí thuyết đồ thị). Còn tập đỉnh thường được kí hiệu bởi chữ V (là chữ cái
đầu tiên của từ "vertex", nghĩa là đỉnh) và tập cạnh bởi chữ E (chữ cái đầu
tiên của từ "edge", nghĩa là cạnh). Đồ thị không có cạnh có hướng thường
được kí hiệu là G = (V, E), còn đồ thị chỉ có cạnh có hướng được kí hiệu là
G = [V, E]. Việc dùng kí hiệu này không bắt buộc, mà chỉ là một thói quen
mà thôi.

1.0.2 Phân loại đồ thị

Thông thường người ta hay biểu diễn đồ thị lên mặt phẳng với đỉnh của
đồ thị là các điểm được tô đậm hoặc khuyên tròn ... , còn cạnh là các đường
cong hoặc đoạn thẳng nối các đỉnh này với nhau. Với hai đỉnh a và b của đồ

106
thị, ta kí hiệu cạnh không có hướng nối a với b bởi (a, b) và cạnh có hướng
nối chúng bởi [a, b]. Trong trường hợp có nhiều cạnh nối a với b thì ta gọi các
cạnh này là cạnh kép và kí hiệu cạnh thứ n nối chúng bởi (a, b, n) (nếu đó là
cạnh vô hướng) hoặc [a, b, n] (nếu đó là cạnh có hướng).
Những cạnh dạng (e, e) với e là đỉnh của đồ thị vô hướng được gọi là khuyên
của đồ thị.
Đồ thị được phân loại theo tính chất cạnh của chúng. Một đồ thị được gọi
là đồ thị vô hướng nếu tất cả các cạnh của chúng đều là cạnh vô hướng. Hai
đỉnh khác nhau của đồ thị vô hướng được gọi là kề nhau hoặc láng giềng của
nhau, nếu như chúng được nối với nhau bởi một cạnh. Nếu hai đỉnh a và b
của một đồ thị G = (V, E) là kề nhau, ta có thể viết (a, b) ∈ E .
Đồ thị được gọi là đồ thị có hướng nếu tất cả các cạnh của nó là đều là
cạnh có hướng. Đỉnh xuất phát của một cạnh có hướng còn được gọi là đỉnh
đầu, và đỉnh kết thúc của cạnh được gọi là đỉnh cuối của nó.
Trong trường hợp một đồ thị có cả cạnh vô hướng cũng như cạnh có hướng
thì đồ thị được gọi là đồ thị hỗn hợp. Một đồ thị được gọi là đồ thị đơn nếu
nó không có khuyên và không có cạnh kép. Ngoài ra, ta gọi đồ thị điểm là đồ
thị có đúng một đỉnh và không có cạnh nào. Đồ thị rỗng dùng để gọi một đồ
thị không có đỉnh và cạnh nào cả.
Ở đây chúng ta chỉ xét đồ thị vô hướng. Có hai đồ thị đặc biệt là đồ thị
đầy đủ Kn là đồ thị có n đỉnh và giữa hai đỉnh chỉ có đúng một cạnh nối
chúng mà thôi.

1.0.3 Biểu diễn phẳng của đồ thị và đẳng cấu

Như chúng ta đã thấy trong các ví dụ trên, đồ thị thường được biểu diễn
trên mặt phẳng: các đỉnh được tô đậm và các cạnh nối các đỉnh là các đoạn

107
thẳng hoặc là các đường cong. Khi thiết lập một mô hình đồ thị và biểu diễn
phẳng để thể hiện nó, chúng ta phải lưu ý rằng một đồ thị có thể có nhiều
biểu diễn phẳng khác nhau.

Định nghĩa 2. Hai đồ thị G1 = (V1 , E1 ) và G2 = (V2 , E2 ) được coi là đẳng cấu
với nhau (được coi chỉ là một đồ thị) nếu như tồn tại một song ánh f : V1 → V2
sao cho hai đỉnh a và b kề nhau trong đồ thị G1 (tức là (a, b) ∈ E1 khi và chỉ
khi f (a) và f (b) kề nhau trong đồ thị G2 (tức là khi (f (a), f (b)) ∈ E2 ).

1.1 Các yếu tố cơ bản của đồ thị

1.1.1 Đồ thị con, đồ thị thành phần

Cho trước một đồ thị G với tập đỉnh X và tập cạnh E .

Định nghĩa 3. Một đồ thị G0 với tập đỉnh X 0 và tập cạnh E 0 được gọi là đồ
thị con của đồ thị G nếu như X 0 là tập hợp con của X và E 0 là tập hợp con
của E . Trong trường hợp X 0 là tập hợp con của X và E 0 là tập hợp tất các
cạnh của G nối hai đỉnh của X 0 , thì G0 được gọi là đồ thị thành phần của G
và còn được gọi là đồ thị sinh bởi tập đỉnh X ’.

Đương nhiên đồ thị rỗng là đồ thị thành phần của mọi đồ thị cho trước.

1.1.2 Bậc của đỉnh

Ta gọi bậc của một đỉnh là số cạnh xuất phát từ đỉnh đó (các khuyên được
tính gấp đôi). Đương nhiên bậc của một đỉnh là một số nguyên không âm.
Một đỉnh được gọi là đỉnh cô lập nếu nó không có cạnh nào cả, tức là khi
đỉnh đó có bậc là 0. Đỉnh có bậc bằng 1 được gọi là đỉnh treo.
Định lý sau tuy đơn giản, nhưng được sử dụng khá nhiều trong lý thuyết
đồ thị.

108
Định lý 1. Trong một đồ thị vô hướng G tùy ý tổng bậc của tất cả các đỉnh
gấp đôi số cạnh của đồ thị.

Chứng minh. Thật vậy, trong tổng bậc tất cả các đỉnh của một đồ thị thì mỗi
cạnh được tính đúng hai lần bởi hai đỉnh của nó. Do đó tổng này là gấp đôi
số cạnh của đồ thị.

Định lí đã chứng minh trên là cơ sở quan trọng để giải nhiều bài toán
chứng minh tồn tại những đỉnh có tính chất đặc biệt trong một đồ thị cho
trước. Chẳng hạn, với những bài toán như sau ta có thể áp dụng định lí đã
được chứng minh một cách hiển nhiên và trực tiếp:
Ví dụ. Trong một đất nước từ mỗi thành phố có một số con đường giao
thông nối nó với các thành phố khác. Một thành phố được gọi là thành phố
lẻ nếu nó có một số lẻ con đường xuất phát từ nó. Chứng minh rằng số các
thành phố lẻ là một số chẵn.

1.1.3 Đường đi

Định nghĩa 4. Cho trước một đồ thị G với tập đỉnh V và tập cạnh E . Một
dãy cạnh dạng ei = (Ai , Ai+1 ) với i = 1, 2, ..., m được gọi là một đường đi nếu
các đỉnh A1 , A2 , ..., Am đôi một khác nhau. Đặc biệt một đường đi qua tất cả
các đỉnh của đồ thị được gọi là đường Hamilton. Một đường đi thường được kí
hiệu là H = (A1 , e1 , A2 , e2 , ..., em , Am+1 ).

Trong trường hợp G là một đồ thị đơn thì ta có thể biểu diễn một đường
đi bởi các đỉnh của chúng, chẳng hạn đường đi H của ta ở trên được kí hiệu
đơn giản là: H = (A1 , A2 , ..., Am+1 ).
Số các cạnh của đường đi được gọi là độ dài của đường đi đã cho.

109
1.1.4 Liên thông

Trong phần này chúng ta làm quen với một khái niệm khá quan trọng của
lí thuyết đồ thị là khái niệm liên thông.
Khi biểu diễn một đồ thị trên mặt phẳng, chúng ta thấy có nhiều khi hình
biểu diễn của chúng là những cụm tách rời nhau không được nối với nhau.
Tương ứng với mỗi hình rời nhau như vậy là một đồ thị thành phần của đồ
thị đã cho mà ta sẽ gọi là một thành phần liên thông của đồ thị cho trước.
Để chính xác hoá khái niệm liên thông, trước hết chúng ta nói hai đỉnh của
một đồ thị cho trước là liên thông với nhau nếu có một dãy cạnh kế tiếp nối
chúng với nhau trong đồ thị đã cho. Tất nhiên là một đỉnh cho trước luôn
được coi là liên thông với chính nó (được nối với chính nó bởi một dãy cạnh
kế tiếp có độ dài 0).

Định nghĩa 5. Một đồ thị được gọi là liên thông nếu hai đỉnh bất kì của nó
liên thông với nhau.

Quan hệ liên thông của các đỉnh có những tính chất cơ bản sau:
a) Mỗi đỉnh a của đồ thị liên thông với chính nó.
b) Nếu a liên thông với b thì b liên thông với a.
c) Nếu a liên thông với b và b liên thông với c, thì a liên thông với c.
Thực chất quan hệ liên thông giữa các đỉnh là một quan hệ tương đương
trong tập hợp các đỉnh của đồ thị. Quan hệ tương đương này chia tập đỉnh
của đồ thị thành các lớp có hai tính chất sau:
1) Các đỉnh thuộc cùng một lớp thì liên thông với nhau.
2) Các đỉnh không cùng thuộc một lớp không liên thông với nhau.
Các lớp đỉnh này là đỉnh của các đồ thị thành phần liên thông trong đồ
thị cho trước, được gọi là thành phần liên thông của đồ thị đã cho.

110
Hình 2: Đồ thị Petersen và hai cách biểu diễn trên mặt phẳng.

1.1.5 Chỉ số liên thông

Ở phần trên chúng ta đã làm quen với khái niệm liên thông. Thế nhưng
ngay trong các đồ thị liên thông cũng có sự phân biệt. Khái niệm sau đây cho
ta một thước đo về sự liên thông của đồ thị.

Định nghĩa 6. Cho trước một đồ thị G và một số tự nhiên k ≥ 2, ta nói G là


một đồ thị k -liên thông (đỉnh), nếu như G là một đồ thị liên thông và nếu như
bỏ đi một số t < k đỉnh tùy ý, đồ thị thu được vẫn là một đồ thị liên thông.

Ví dụ. Đồ thị Petersen trong hình 2 là một đồ thị 3-liên thông, vì nó là


một đồ thị liên thông,và chỉ khi bỏ đi tới 3 đỉnh, ta mới có thể thu được một
đồ thị không liên thông và khi bỏ đi 2 đỉnh tùy ý, đồ thị Petersen vẫn còn
liên thông.
Tương tự khái niệm liên thông đỉnh, ta gọi một đồ thị liên thông là đồ thị
k-liên thông cạnh nếu như bỏ đi ít hơn k cạnh, từ đồ thị ban đầu ta vẫn thu
được một đồ thị liên thông. Khái niệm liên thông cạnh ít được nghiên cứu tới
hơn khái niệm liên thông đỉnh.
Thông thường ta kí hiệu số thành phần liên thông của một đồ thị G cho
trước bởi ω(G). Nếu G là một đồ thị có n đỉnh rời nhau, thì ω(G) = n.

Định nghĩa 7. Cho trước một đồ thị G. Số tự nhiên bé nhất thỏa mãn điều

111
kiện: G là đồ thị k -liên thông (đỉnh) nhưng không k + 1- liên thông (đỉnh),
được gọi là chỉ số liên thông (đỉnh) của đồ thị G.

Ví dụ. Đồ thị Petersen trong hình 2 có chỉ số liên thông là 3.


Tương tự như vậy, ta có thể định nghĩa chỉ số liên thông cạnh của một đồ
thị G cho trước, là số nhỏ nhất sao cho G là một đồ thị k -liên thông cạnh mà
không k + 1- liên thông cạnh. Đồ thị Petersen trong hình 2 là một đồ thị có
chỉ số liên thông cạnh là 3.

1.1.6 Chu trình của đồ thị

Khi định nghĩa đường đi nối hai đỉnh a và b của một đồ thị, ta luôn giả
thiết rằng các đỉnh a và b này phải khác nhau. Trong trường hợp a và b được
nối với nhau bởi một cạnh, thì khi thêm cạnh (a, b) vào, ta thu được từ con
đường đã cho một chu trình. Như vậy chu trình là một dãy cạnh kế tiếp khép
kín sao cho mỗi đỉnh của đồ thị được đi qua không quá một lần.
Chu trình được kí hiệu bởi việc đưa ra các cạnh và các đỉnh liên tiếp nhau
trên chu trình. Chẳng hạn, nếu chu trình C đi qua các đỉnh p1 , p2 , ..., pk và các
cạnh e1 , e2 , ..., ek thì ta viết

C = (p1 , e1 , p2 , e2 , ..., pk , ek , p1 ).

Trong trường hợp đồ thị là một đồ thị đơn, thì thay vì viết rõ các cạnh và
các đỉnh, chu trình được xác định duy nhất qua việc gọi tên các đỉnh nó đi
qua. Chẳng hạn, chu trình C đề cập ở trên có thể viết thành:

C = (p1 , p2 , . . . , pk , p1 ).

Số cạnh của chu trình được gọi là độ dài của chu trình và thông thường hay
được kí hiệu bởi `(C). Một khuyên lập thành một chu trình có độ dài. Dễ

112
thấy rằng một đồ thị cho trước chỉ có chu trình có độ dài 2 nếu như nó có
cạnh kép. Trong một đồ thị đơn mỗi chu trình có độ dài ít nhất là 3. Một đồ
thị không đơn hiển nhiên luôn có ít nhất một chu trình (có độ dài 1 hoặc 2).
Trong đồ thị đơn không phải lúc nào ta cũng có thể tìm thấy một chu trình.
Chẳng hạn trong các cây là các đồ thị ta sẽ nói tới ở chương sau ta không
tìm được một chu trình nào cả.

BÀI TẬP

1. Trong một kì nghỉ hè, có 7 người bạn đi nghỉ mát ở xa. Họ hứa với nhau
rằng trong suốt kì nghỉ mỗi người phi viết thư cho đúng ba người trong
số họ. Chứng minh rằng có một người không viết thư cho người viết cho
mình.

2. Tồn tại hay không một đồ thị đơn vô hướng với 6 đỉnh và bậc của các
đỉnh lần lượt là

a) 2, 3, 3, 3, 4, 4.

b) 2, 3, 3, 4, 4, 4.

3. Lập một đồ thị có mỗi đỉnh tương ứng với một tập con của tập con của
tập hợp {1, 2, 3, ..., 2001} và hai đỉnh được nối với nhau bởi một cạnh nếu
hai tập con tương ứng với chúng không có điểm chung. Hãy tính số cạnh
và số đỉnh của đồ thị thu được.

4. Chứng minh rằng một đồ thị G không liên thông khi và chỉ khi nó có ít
nhất 2 thành phần liên thông.

113
1.2 Một số loại đồ thị đơn vô hướng

1.2.1 Đồ thị đầy đủ

Một đồ thị đơn vô hướng với n đỉnh được gọi là đồ thị đầy đủ nếu giữa
hai đỉnh bất kì của nó có đúng một cạnh nối. Trong hình trên ta có một đồ
thị đầy đủ có 9 đỉnh. Nếu số đỉnh n của đồ thị cho trước thì chỉ có đúng một
đồ thị đầy đủ với n đỉnh mà thôi. Do đó, người ta vẫn kí hiệu đồ thị đầy đủ
n đỉnh là Kn . Với kí hiệu này, thì đồ thị đầy đủ trong hình trên là đồ thị K9 .
Trong đời sống chúng ta hay gặp các đồ thị đầy đủ. Chẳng hạn, đồ thị
biểu diễn quan hệ quen biết của các học sinh trong một lớp học là một đồ
thị đầy đủ, vì đương nhiên học sinh của một lớp học bao giờ cũng quen biết
nhau hết. Nhiều khi mô hình đồ thị đầy đủ khó nhận biết hơn, chẳng hạn khi
phải xét một xếp đặt tối ưu mà một cặp đôi bất kì đều phải có mặt.

1.2.2 Đồ thị đều

Một đồ thị đơn vô hướng G được gọi là đồ thị đều bậc t nếu như mỗi đỉnh
của đồ thị G có bậc là t. Đồ thị Petersen là đồ thị đều bậc 3. Đặc biệt một
đồ thị đầy đủ với n đỉnh là một đồ thị đều bậc n − 1. Một đồ thị chỉ có các
đỉnh rời được gọi là đồ thị đều bậc 0. Đồ thị đầy đủ K2 là đồ thị đều bậc 1
duy nhất. Mỗi đồ thị đều liên thông bậc 2 là một chu trình. Một loạt những
tính chất thú vị chứa đựng trong đồ thị đều bậc 3. Những tính chất sau đây
là những tính chất cơ bản của các đồ thị đều:

Định lý 2. Số đỉnh của đồ thị đều bậc lẻ luôn là một số chẵn.

Định lí này được suy ra một cách hiển nhiên từ mệnh đề quen biết là số
đỉnh bậc lẻ của một đồ thị luôn là số chẵn. Ngoài ra chúng ta cũng dễ chứng
minh định lí sau:

114
Định lý 3. Cho trước G là một đồ thị đều bậc g với n đỉnh và c cạnh. Khi
đó ta có n.g = 2c.

Định lí được chứng minh dễ dàng nhờ điều đã biết là tổng các bậc của
các đỉnh của một đồ thị bằng đúng hai lần số cạnh. Rõ ràng một đồ thị với
2 đỉnh và 3 cạnh kép nối các đỉnh này với nhau là một đồ thị đều bậc 3. Cho
trước một số tự nhiên m ≥ 1, thì hợp của m đồ thị đều bậc 3 như vậy là một
đồ thị đều bậc 3 với 2m đỉnh. Rõ ràng không tồn tại đơn đồ thị đều bậc 3
với 2 đỉnh. Đồ thị đầy đủ K4 là đồ thị đơn và đều bậc 3 duy nhất với đúng 4
đỉnh.

1.2.3 Đồ thị lưỡng phân

Đồ thị lưỡng phân là đồ thị G = (V ; E) mà tập đỉnh V có thể phân hoạch


thành hai tập hợp X, Y sao cho tập cạnh E chỉ gồm các cạnh nối hai đỉnh
không cùng một tập hợp.
Ta có thể thấy dễ dàng hai tính chất sau đây của đồ thị lưỡng phân:
1) Mỗi đồ thị con của đồ thị lưỡng phân là một đồ thị lưỡng phân.
2) Đồ thị lưỡng phân không có khuyên.
Ta có thể tìm thấy trong cuộc sống có rất nhiều ví dụ về đồ thị lưỡng phân.
Chẳng hạn đồ thị biểu diễn quan hệ yêu đương khác giới là đồ thị lưỡng phân.
Một tính chất cơ bản để nhận biết đồ thị lưỡng phân là định lí sau đây:

Định lý 4. Một đồ thị G là đồ thị lưỡng phân khi và chỉ khi mọi chu trình
của nó có độ dài chẵn.

Chứng minh. Giả sử G = (X, Y ; E) là một đồ thị lưỡng phân. Khi đó dọc theo
chu trình bất kì của G các đỉnh thuộc tập X và tập hợp Y lần lượt kế tiếp
nhau. Do đó, khi trở về đỉnh xuất phát đầu tiên, ta phải đi qua một số chẵn
các đỉnh, và do đó số cạnh (bằng số đỉnh) của chu trình là một số chẵn.

115
Đảo lại, giả sử rằng G là một đồ thị có tính chất là tất cả các chu trình
của G đều có độ dài chẵn. Ta sẽ chứng minh rằng tất cả các thành phần liên
thông của G đều là các đồ thị lưỡng phân, và do đó G cũng là đồ thị lưỡng
phân.
Thật vậy, giả sử rằng G1 là một thành phần liên thông của G và P0 là một
đỉnh của đồ thị G1 . Với mỗi đỉnh P của đồ thị G1 ta chọn một đường đi W
nối đỉnh P0 với đỉnh P . Nếu đường đi W có độ dài (số cạnh của W ) chẵn thì
đỉnh P thuộc tập X , còn nếu đường đi W có độ dài lẻ thì đỉnh P được lấy vào
tập hợp Y . Sự phân loại các đỉnh của đồ thị G1 không phụ thuộc vào cách
chọn đường đi W . Thật vậy, nếu có hai con đường W với độ dài chẵn và con
đường đi W 0 với độ dài lẻ nối đỉnh P0 với đỉnh P thì đồ thị G1 sẽ có chu trình
với độ dài lẻ, mâu thuẫn với giả thiết ban đầu là đồ thị G chỉ có chu trình với
độ dài chẵn.
Với cách thiết lập tập hợp X và Y này, các đỉnh của đồ thị G1 hoặc thuộc
tập hợp X hoặc thuộc tập hợp Y . Bây giờ ta chứng minh rằng chỉ có các cạnh
nối các đỉnh không cùng một tập hợp với nhau mà thôi. Thật vậy, giả sử rằng
có hai đỉnh P và Q kề nhau trong đồ thị G1 , thì chúng không thể cùng thuộc
một tập hợp X hoặc Y , nếu không từ P0 ta có thể đi tới đỉnh P rồi tới đỉnh
Q bởi cạnh (P, Q) và trở về đỉnh P0 với một đường đi lẻ cạnh, là điều không
thể xảy ra trong đồ thị G, do đồ thị G chỉ có chu trình với số chẵn cạnh mà
thôi. Như vậy đồ thị G là đồ thị lưỡng phân với hai tập đỉnh X và Y , và với
tập cạnh E mà ta có thể viết như đã qui địnhG = (X, Y ; E).

Nếu trong đồ thị lưỡng phân G = (X, Y ; E) giữa hai đỉnh bất kì không
cùng trong một lớp đỉnh luôn có đúng một cạnh nối, thì ta gọi G = (X, Y ; E)
là đồ thị lưỡng phân đầy đủ và kí hiệu nó bởi Km,n , với n là số đỉnh của X
và m là số đỉnh của Y .

116
1.2.4 Cây

Một cây là một đồ thị đơn vô hướng liên thông không có chu trình với ít
nhất một đỉnh. Các đồ thị loại này đã được nhiều nhà khoa học và nhiều nhà
toán học nghiên cứu từ rất sớm do những ứng dụng của nó trong lí thuyết
mạng điện.
Hình trên biểu diễn 4 cây khác nhau. Nếu quan sát kĩ các biểu diễn của
cây, ta có thể nhận thấy một cây bất kì với ít nhất 2 đỉnh luôn có những đỉnh
có bậc đúng bằng 1. Những đỉnh này được gọi là đỉnh treo và chúng đóng một
vai trò hết sức quan trọng trong lí thuyết về cây. Trước hết, ta chứng minh
sự tồn tại của các đỉnh treo trong cây có ít nhất 2 đỉnh.

Định lý 5. Một cây bất kì với ít nhất hai đỉnh có ít nhất hai đỉnh treo.

Chứng minh. Trong cây chỉ có hữu hạn con đường. Ta xét W = (P1 , P2 , ..., Pk )
là một con đường có nhiều cạnh nhất. Khi đó ta có k > 1, do cây phải có ít
nhất hai đỉnh.
Hai đỉnh cuối của W , P1 và Pk , rõ ràng là hai đỉnh treo. Giả sử ngược lại
là P1 không phải là đỉnh treo thì đỉnh P1 được nối với đỉnh Q 6= P2 nào đó.
Khi đó đỉnh Q = Pi với i > 2 nào đó, do W có nhiều cạnh nhất có thể. Nhưng
khi đó ta sẽ thu được một chu trình K = (P1 , P2 , ..., Pi , P1 ), trái với giả thiết
ban đầu là G không có chu trình.

Dựa trên sự tồn tại của các đỉnh treo, ta có thể xác định được số cạnh của
cây thông qua định lí sau đây.

Định lý 6. Một cây có n đỉnh có đúng n − 1 cạnh.

Chứng minh. Chứng minh bằng quy nạp theo số đỉnh n của cây.
Với n = 1 rõ ràng cây với 1 đỉnh không có cạnh nào cả.

117
Giả sử một cây tùy ý với n đỉnh có đúng n − 1 cạnh.
Xét G là một cây có n + 1 đỉnh tùy ý. Theo định lí đã chứng minh ở trên,
G có ít nhất một đỉnh treo P nào đó. Xét đồ thị G − {P }. Vì đỉnh P là đỉnh
treo, nên đồ thị G − {P } là một đồ thị liên thông. Giả sử ngược lại là đồ thị
G − {P } có ít nhất hai thành phần liên thông G1 và G2 nào đó. Do G là đồ thị
liên thông, cho nên có một con đường G1 với G2 trong G. Rõ ràng con đường
này phải đi qua đỉnh P và nhận đỉnh P làm đỉnh trong của nó. Vậy P có bậc
ít nhất là 2, mâu thuẫn với giả thiết là đỉnh P là đỉnh treo trong đồ thị G.
Do G không có chu trình, cho nên đồ thị G − {P } không có chu trình. Tóm
lại là đồ thị G − {P } là một cây có n đỉnh. Theo giả thiết quy nạp thì đồ thị
G − {P } có đúng n − 1 cạnh. Suy ra đồ thị G có đúng n cạnh, vì bậc của P
trong đồ thị G bằng 1.

Định lý 7. Cho trước một đồ thị G = (V, E), khi đó sáu tính chất sau là tương
đương
1) G là một cây (G là đồ thị liên thông và không có chu trình)
2) G không có chu trình và có n − 1 cạnh
3) G liên thông và có n − 1 cạnh
4) G không có chu trình và nếu thêm vào một cạnh nối 2 đỉnh không kề nhau
thì G xuất hiện duy nhất một chu trình.
5) G liên thông và nếu bỏ đi một cạnh tuỳ ý thì đồ thị nhận được sẽ không
liên thông.
6) Mỗi cặp đỉnh trong G được nối với nhau bằng một đường duy nhất.

Chứng minh. Ta chứng minh theo trình tự sau: 1) ⇒ 2) ⇒ 3) ⇒ 4) ⇒ 5) ⇒


6) ⇒ 1).
1) ⇒ 2) Cho G là đồ thị liên thông và không có chu trình. Ta phải chứng minh
2). Điều này hiển nhiên đúng theo định lý đã được chứng minh về số cạnh

118
của cây.
2) ⇒ 3) Cho trước G không có chu trình và có n − 1 cạnh, ta phải chứng minh
3), tức là chỉ còn cần phải chứng minh G liên thông. Thật vậy nếu G có p
thành phần liên thông, thì mỗi thành phần liên thông là một cây và số cạnh
của G theo định lý về số cạnh của cây sẽ là |V | − p (lấy tổng các cạnh của
các cây thành phần lại). Do số cạnh của G được cho biết là n − 1, nên ta có
|V | − p = n − 1, vậy p = 1 hay nói cách khác là G là đồ thị liên thông.
3) ⇒ 4) Cho trước G liên thông và có n − 1 cạnh, ta phải chứng tỏ rằng nếu
thêm một cạnh nối 2 đỉnh không kề nhau bất kỳ trong G thì đồ thị thu được
sẽ có chu trình. Trước hết ta thấy G là cây, vì nếu không có thể lân lượt bỏ
bớt các cạnh khỏi G sao cho đồ thị thu được là đồ thị liên thông, và đến
lúc nào đó không còn có thể bỏ được cạnh nào nữa, lúc đó G không còn chu
trình (nếu không có thể bỏ bất kỳ cạnh nào khác trên chu trình vẫn không
nh hưởng tính liên thông của đồ thị thu được). Đồ thị thu được sau cùng là
cây, và sẽ có n − 1 cạnh theo định lý về số cạnh của đồ thị. Suy ra số cạnh
bỏ đi là 0, tức là G là cây và không có chu trình. Sử dụng định lý về đường
đi giữa 2 đỉnh tùy ý x và y của G, ta có chu trình khi thêm cạnh nối 2 đỉnh
không kề nhau này.
4) ⇒ 5) : Cho trước G không có chu trình và biết rằng nếu thêm vào một cạnh
nối 2 đỉnh không kề nhau thì G xuất hiện duy nhất một chu trình, ta phải
chứng minh G liên thông và bỏ đi cạnh tùy ý thì G không còn liên thông.
Giả sử ngược lại G không liên thông, tức là tồn tại cặp đỉnh x, y trong G
mà không có đường nào nối x với y . Khi đó nối x và y bởi 1 cạnh, đồ thị nhận
được vẫn không có chu trình điều này mâu thuẫn với 4). Hay G là liên thông.
Nếu bỏ đi 1 cạnh trong G mà đồ thị vẫn liên thông thì nếu khôi phục lại
cạnh này đồ thị sẽ có chu trình. Điều này mâu thuẫn với 4). Vậy ta có 5).
5) ⇒ 6) : Cho trước đồ thị G liên thông và bỏ đi cạnh tùy ý thì G không còn

119
liên thông, ta phải chứng minh rằng mỗi cặp đỉnh trong G được nối với nhau
bằng một đường duy nhất. Thật vậy, giả sử ngược lại, nếu trong G có tồn
tại cặp đỉnh x, y không nối với nhau bằng đường nào cả, chứng tỏ G không
liên thông mâu thuẫn với 5). Vậy mỗi cặp đỉnh đều có đường đi nối với nhau.
Đường nối đó là duy nhất vì nếu có nhiều hơn, thì sau khi bỏ đi 1 cạnh trên
chu trình xuất hiện bởi 2 đường nối 2 đỉnh này thì đồ thị vẫn liên thông, trái
với 5).
6) ⇒ 1) : Cho trước một đồ thị G với tính chất là mỗi cặp đỉnh trong G được
nối với nhau bằng một đường duy nhất, ta phải chứng minh G là cây.
Do mỗi cặp đỉnh nối với nhau bởi một đường nên G là liên thông. Giả sử
G có chu trình thì xét cặp đỉnh x, y trên chu trình đó. Khi đó x, y có 2 cặp
đường nối với nhau, mâu thuẫn với 6).

Cây có rất nhiều ứng dụng trong đời sống và khoa học. Những ứng dụng
về cây sẽ được giới thiệu riêng trong một mục sau.

1.2.5 Đồ thị phẳng

Đôi khi người ta còn muốn biểu diễn đồ thị trên mặt phẳng sao cho không
có 2 cạnh được biểu diễn nào cắt nhau. Mong muốn này đã trở thành một
trong những vấn đề quan trọng của lí thuyết đồ thị. Không phải đồ thị nào
cũng có thể biểu diễn như vậy, và những đồ thị có thể biểu diễn được thỏa
mãn điều kiện đó được gọi là đồ thị phẳng. Ngay từ những năm 1930 nhà
toán học Kuratowski, người Ba lan, đã phân lớp được những lớp đồ thị như
vậy.
Ví dụ. Có ba cái nhà và ba cái giếng. Mỗi nhà có ba con đường đi từ nó
tới ba cái giếng. Hỏi có thể làm những con đường đi như vậy sao cho không
có hai con đường nào cắt nhau hay không?

120
Để giảibài toán này, chúng ta phải giả sử là các nhà là các điểm A1 , A2 , A3
trên mặt phẳng và các giếng là các điểm B1 , B2 , B3 nào đó. Các con đường đi
là các đường (liên tục) nối các đỉnh Ai với các đỉnh Bi như trong hình biểu
diễn. Bằng cách thiết lập mô hình này, chúng ta đã thiết lập một đồ thị có 6
đỉnh và 9 cạnh.
Khi đó, câu hỏi của bài toán là liệu có những đường nối các điểm Ai tới
các điểm Bj trên mặt phẳng sao cho không có hai con đường nào cắt nhau
hay không?
Thuộc lớp đồ thị phẳng có cây, bụi (hợp của những cây không có đỉnh
chung) và đồ thị biểu diễn của các khối đa diện lồi. Cho lớp đồ thị biểu diễn
của các khối đa diện lồi, nhà toán học Steinitz đã chứng minh:

Định lý 8. Lớp các đồ thị biểu diễn các khối đa diện lồi chính là lớp các đồ
thị phẳng và 3-liên thông.

Các đồ thị quen biết nhất không thể biểu diễn phẳng được chính là đồ thị
đầy đủ K5 và đồ thị lưỡng phân đầy đủ K3,3 . Bạn đọc hẳn cũng nhận ra rằng
đồ thị lưỡng phân đầy đủ K3,3 có liên quan với bài toán dân gian nổi tiếng về
vấn đề 3 ngôi nhà và 3 cái giếng mà ta đã nói ở trên. Vấn đề làm đường cho
3 ngôi nhà dẫn tới 3 cái giếng được chứng minh là không thực hiện được.
Nếu một đồ thị được biểu diễn phẳng sao cho không có hai cạnh nào của
chúng cắt nhau, thì những phần mặt phẳng được chia ra bởi những cạnh của
đồ thị được gọi là miền . Trong trường hợp đồ thị là đồ thị biểu diễn của một
khối đa diện lồi thì miền của nó cũng tương ứng với các mặt của khối đa diện
lồi đã cho. Một định lí c sở cho việc nghiên cứu các đồ thị phẳng là định lí
của Euler:

Định lý 9. Cho đồ thị biểu diễn khối đa diện lồi có d đỉnh, c cạnh và m miền,
ta có đẳng thức d + m = c + 2.

121
Để chứng minh định lí này, ta gọi một cạnh của đồ thị phẳng cho trước G
là cầu nếu bỏ nó đi thì số thành phần liên thông của đồ thị tăng lên ít nhất
1 và là cạnh biên, nếu cạnh này không phải là cầu của đồ thị G. Ta thấy hiển
nhiên là một cạnh e của một đồ thị phẳng G cho trước là một cạnh biên khi
và chỉ khi nó là cạnh chung của hai miền khác nhau của đồ thị G.
Dễ thấy một cạnh bất kì của đồ thị phẳng chỉ có thể hoặc là cầu hoặc là
cạnh biên mà thôi.
Bây giờ ta chứng minh định lí tổng quát hơn định lí của Euler về quan hệ
giữa số đỉnh, số cạnh và số miền của một đồ thị phẳng:
Định lý 10. Cho trước đồ thị G là một đồ thị phẳng với d đỉnh, c cạnh và m
miền. Khi đó ta có d + m = c + ω(G) + 1.
Chứng minh. Ta chứng minh bằng quy nạp theo số cạnh c của đồ thị.
Khẳng định của định lí hiển nhiên đúng cho trường hợp c = 0, vì trong
trường hợp này ta có d = ω(G) và m = 1 cho mọi đồ thị phẳng không có cạnh.
Bây giờ ta giả sử rằng đẳng thức

d + m = c + ω(G) + 1,

đúng cho mọi đồ thị phẳng G với 0 cạnh.


Giả sử đồ thị phẳng G với c + 1 cạnh cho trước. Chúng ta chọn một cạnh
e tùy ý trong đồ thị G và khảo sát đồ thị G − e, sinh ra khi bỏ cạnh e ra khỏi
đồ thị G. Chỉ có hai trường hợp sau đây có thể xảy ra:
1) Cạnh e là một cầu của đồ thị G.
Trong trường hợp này đồ thị G − e có đúng m miền, do cạnh e không là cạnh
chung của hai miền nào cả, và rõ ràng chỉ có đúng c cạnh. Theo giả thiết quy
nạp, ta có d + m = c + ω(G − e) + 1.
Do ω(G − e) = ω(G) + 1, vì e là cầu của đồ thị G, ta có được:

d + m = (c + 1) + ω(G) + 1,

122
là điều cần phải chứng minh.
2) Cạnh e là biên giới của đồ thị G. Khi đó đồ thị G − e chỉ có đúng m − 1
miền, do hai miền có cạnh chung e sẽ trở thành một miền duy nhất trong đồ
thị G − e, và có đúng c − 1 cạnh. Theo giả thiết quy nạp, ta có

d + m − 1 = c + ω(G − e) + 1.

Do ω(G − e) = ω(G), vì e không phải là cầu của đồ thị G, ta có:

d + m − 1 = c + ω(G) + 1,

và do đó d + m = (c + 1) + ω(G) + 1, là điều cần phải chứng minh.


Trong cả hai trường hợp trên ta có

d + m = (c + 1) + ω(G) + 1.

Định lí được chứng minh.

Ta có thể thấy dễ dàng là định lí của Euler là một trường hợp riêng của
định lí trên.

BÀI TẬP

1. Mỗi đồ thị phẳng có một đỉnh có bậc không vượt quá 5. Hãy chứng minh
điều đó.

2. Mỗi cạnh của một khối đa diện lồi được quy định một chiều đi sao cho
tại mỗi đỉnhcó ít nhất một cạnh đi ra và một cạnh đi vào. Hãy chứng
minh rằng tồn tại hai mặt khác nhau của khối đa diện này sao cho trên
mỗi mặt chiều đi của các cạnh kế tiếp nhau để ta có thể đi một vòng
trên đường biên của nó theo chiều đi được quy định của các cạnh.

123
3. Hãy xác định tất cả các đồ thị lưỡng phân phẳng liên thông đều bậc 3.

4. Tất cả các mặt của một khối đa diện lồi là hình tam giác. Chứng minh
rằng người ta có thể tô màu các cạnh của hình khối đa diện này bởi màu
đỏ và màu xanh sao cho giữa hai đỉnh bất kì của khối đa diện người ta
luôn tìm được hai con đường đơn màu khác nhau, một con đường chỉ
gồm các cạnh màu đỏ và một con đường chỉ gồm các cạnh màu xanh,
nối hai đỉnh này.

5. Cho trước một hình khối đa diện lồi có tất cả các mặt là các hình tam
giác. Chứng minh rằng người ta có thể tô màu các mặt bởi hai màu xanh
đỏ sao cho không có hai mặt cùng màu nào có cạnh chung khi và chỉ khi
người ta có thể tô màu các đỉnh của tam giác bởi ba màu vàng, xanh và
đỏ sao cho không có hai đỉnh nào của một cạnh được tô bởi một màu.

2 Các bài toán tô màu

2.1 Lịch sử ra đời

Các bài toán tô màu có 3 loại chủ yếu:

1. Tô màu các đỉnh (sắc số)

2. Tô màu cạnh (chỉ số Ramsey)

3. Tô màu miền (bài toán 4 màu, giả thiết Hardwiger)

2.1.1 Sắc số

sectionTô màu đỉnh và cạnh

124
2.2 Sắc số

Cho trước một đồ thị G, ta nói một đỉnh a của G được tô màu ổn định
nếu như không có láng giềng nào của a được tô màu của a. Một cách tô màu
các đỉnh của đồ thị G được gọi là một cách tô màu ổn định nếu như đỉnh nào
của đồ thị G cũng được tô màu ổn định; nghĩa là không có hai đỉnh kề nhau
nào của đồ thị G được tô màu giống nhau cả.
Giả sử một đồ thị G cho trước được tô màu ổn định. Xét phân hoạch các đỉnh
của đồ thị G vào các tập hợp theo quan hệ cùng màu, thì trong mỗi tập hợp
con chứa các đỉnh cùng màu này, không có tập hợp nào chứa hai đỉnh được
nối với nhau bởi một cạnh cả. Số nhỏ nhất các tập hợp có tính chất là cạnh
bất kì của đồ thị G chỉ nối hai đỉnh thuộc tập hợp khác nhau chính là số nhỏ
nhất các màu có thể tô các đỉnh của đồ thị G một cách ổn định - được gọi là
sắc số của đồ thị G và được kí hiệu bởi χ(G).
Trong mục này chúng ta chỉ xét đồ thị đơn, vì rõ ràng là với đồ thị có
khuyên thì ta không có cách tô màu ổn định các đỉnh của nó, và nếu đồ thị
có cạnh kép thì có thể bỏ bớt các cạnh kép để thu được một đồ thị có cạnh
đơn mà không ảnh hưởng gì đến vấn đề tô màu các đỉnh của đồ thị, vì nếu
đồ thị G có cạnh kép thì ta có thể thay thế cạnh kép bằng các cạnh đơn để
nhận được đồ thị đơn G∗ và rõ ràng là ta có

χ(G∗ ) = χ(G).

Với mỗi đồ thị đơn G với n đỉnh người ta có thể tô màu ổn định các đỉnh của
chúng bởi n màu: ta gán cho mỗi đỉnh trong số n đỉnh của đồ thị bởi một
trong số n màu khác nhau cho trước, và cách tô màu hiển nhiên này rõ ràng
là một cách tô màu ổn định. Ngoài ra ta có thể chỉ xét đồ thị liên thông, bởi
lẽ nếu đồ thị có nhiều thành phần liên thông thì số màu cần thiết để tô ổn
định nó chính là số màu lớn nhất có thể tô ổn định các thành phần liên thông

125
của nó. Dễ thấy rằng đồ thị điểm có sắc số bằng 1 và đồ thị rỗng có sắc số
bằng 0. Nếu như không nói rõ, thì mặc nhiên những đồ thị được xét trong
phần tiếp của mục này là các đồ thị liên thông và đơn.
Rất nhiều bài toán tô màu chứng tỏ rằng chúng là những bài toán rất khó
và thú vị, chẳng hạn như bài toán 4 màu. Từ lâu cuối thế kỉ 18, những người
thợ tô bản đồ ở Anh đã nhận thấy rằng để tô màu bản đồ chỉ cần 4 màu là
đủ sao cho hai quốc gia có biên giới chung sẽ được tô màu bởi những màu
khác nhau. Định lí gần như là hiển nhiên sau đây cho ta một phương pháp
để xác định được cận dưới của sắc số.

Định lý 11. Cho mỗi đồ thị G với bậc lớn nhất ∆, ta có

χ(G) ≤ ∆ + 1.

Chứng minh. Nếu như ta có ∆ + 1 màu để tô, thì ta có thể tô ổn định n


đỉnh của đồ thị G bằng một cách tùy ý từ đỉnh 1 tới đỉnh n như sau. Khi tô
một đỉnh nào đó bởi một màu nào đó, ta tô tiếp một đỉnh x kề nó một cách
ổn định bởi một màu khác tất cả các màu láng giềng của nó. Việc tô màu
này luôn tiến hành được bởi vì ta có ∆ + 1 màu để lựa chọn mà số láng giềng
của đỉnh x không vượt quá ∆. 
Cho phần tiếp theo, ta gọi chu trình là một dãy các đỉnh kế tiếp nhau sao
cho hai đỉnh liên tiếp được nối với nhau bởi một cạnh. Thông thường chu
trình được biểu diễn thông qua các đỉnh và các cạnh mà nó đi qua. Với đồ
thị đơn, chu trình được xác định duy nhất qua việc gọi tên các đỉnh của nó.
Chẳng hạn, chu trình C đi qua các đỉnh p1 , p2 , · · · , pk có thể biểu diễn bởi:

C = (p1 , p2 , · · · , pk , p1 ).

Số cạnh của chu trình C được gọi là độ dài của chu trình và thông thường

126
hay được kí hiệu bởi `(C). Trong đồ thị đơn mỗi chu trình có độ dài ít nhất
là 3.
Rõ ràng là đồ thị duy nhất có sắc số 1 là các đồ thị chỉ có đỉnh rời rạc (tức
là không có cạnh nào cả). Có một điều kiện cần và đủ cho đồ thị với sắc số 2.

Định lý 12. Một đồ thị cho trước G với ít nhất một cạnh có sắc số bằng 2
khi và chỉ khi G không có chu trình lẻ cạnh.

Chứng minh. Rõ ràng ta chỉ cần chứng minh định lí cho các đồ thị liên
thông là đủ.
Nếu như đồ thị G có sắc số bằng 2 thì các đỉnh của chu trình C bất kì của
đồ thị G sẽ đan màu nhau dọc theo chu trình và do đó chu trình C có độ dài
chẵn.
Đảo lại cũng đúng. Nếu như G không có chu trình có lẻ cạnh thì ta sẽ chứng
minh rằng ta có thể tô màu ổn định các đỉnh của đồ thị G bằng 2 màu. Trước
hết ta chọn một đỉnh p0 của G. Với mỗi đỉnh p của đồ thị G ta chọn một con
đường W nối đỉnh p0 với đỉnh p. Ta sẽ tô màu đỉnh p bởi màu đỏ, nếu như
độ dài `(W ) là độ dài chẵn, trong trường hợp ngược lại, ta tô màu đỉnh p bởi
màu xanh.
Màu của đỉnh p không phụ thuộc vào độ dài của con đường W . Bởi vì nếu có
một con đường lẻ cạnh và một con đường chẵn cạnh nối đỉnh p0 với đỉnh p,
thì tồn tại trong đồ thị G một dãy cạnh kế tiếp khép kín có lẻ cạnh. Dễ thấy
khi đó tồn tại một chu trình có lẻ cạnh trong đồ thị G, mâu thuẫn với giả
thiết của định lí là đồ thị G không có chu trình lẻ cạnh nào cả. Ngoài ra, hai
đỉnh p và q được nối trong đồ thị G bởi một cạnh k , thì ta xét một con đường
W0 có ít cạnh nhất nối đỉnh p0 với đỉnh p. Rõ ràng là khi bổ sung thêm cạnh
k vào con đường W0 này hoặc bỏ bớt cạnh k - tùy theo trường hợp đỉnh q có
nằm trên đường W0 hay không- ta sẽ thu được một con đường nối đỉnh p0 với

127
đỉnh q với độ dài `(W0 ) + 1 hoặc `(W0 ) − 1. Do tính chẵn lẻ của con đường nối
đỉnh p0 với đỉnh p và con đường nối đỉnh p0 với đỉnh q khác nhau, cho nên
màu của đỉnh p và màu của đỉnh q khác nhau. Do đó đỉnh p được tô màu ổn
định, và cách tô màu các đỉnh của đồ thị G như đã trình bày là một cách tô
màu ổn định. 

BÀI TẬP

1. Hãy chứng minh rằng các đỉnh của một ngũ giác đều không thể tô ổn
định bằng hai màu.

2. Hãy xác định sắc số của đồ thị Petersen.

3. Chứng minh rằng với mỗi số tự nhiên k cho trước luôn tồn tại một đồ
thị có sắc số bằng k .

4. Chứng minh bất đẳng thức χ(G1 ) ≤ χ(G2 ) nếu đồ thị G1 là đồ thị con
của đồ thị G2 .

n2
5. Chứng minh rằng nếu ta tô đỏ nhiều hơn 2 ô của một bàn cờ gồm n × n
ô, thì luôn tồn tại hai ô đỏ có chung một cạnh.

6. Một đồ thị G không có khuyên với ít nhất một cạnh có sắc số 2 khi và
chỉ khi G không có chu trình độ dài lẻ cạnh!

2.2.1 Các bài toán tô màu cạnh

Ramsey xét bài toán chia tập hợp các cạnh của một đồ thị đầy đủ vào hai
ngăn kéo bằng cách tô màu các cạnh của đồ thị đầy đủ bởi hai màu xanh và
đỏ và khẳng định rằng với mỗi cặp hai số tự nhiên m và n luôn tồn tại một

128
số tự nhiên p sao cho với mọi cách tô các cạnh của đồ thị đầy đủ Kp bởi hai
màu nói trên hoặc ta sẽ có một đồ thị đầy đủ Kn màu đỏ hoặc một đồ thị
đầy đủ Km màu xanh.
Số p nhỏ nhất với tính chất đó ta muốn kí hiệu là R(m, n). Ta có thể thấy
rằng R(3, 3) ≥ 6, vì ta có thể tô màu các cạnh của đồ thị đầy đủ K5 với hai
màu mà không thu được một đồ thị đầy đủ 3 đỉnh K3 nào cả. Định lí sau đây
cho ta thấy R(3, 3) = 6.

Định lý 13. Nếu tô màu các cạnh của đồ thị đầy đủ 6 đỉnh K6 với hai màu
xanh hoặc đỏ, thì luôn tồn tại một đồ thị đầy đủ 3 đỉnh K3 là đồ thị con của
đồ thị này và tất cả các cạnh của nó hoặc cùng màu đỏ hoặc cùng màu xanh.

Định lý 14. Cho mỗi số tự nhiên k , ta có bất đẳng thức

Rk+1 (3) ≤ (k + 1)(Rk (3) − 1) + 2.

BÀI TẬP

1. Sáu điểm trên mặt phẳng được xếp sao cho khoảng cách giữa chúng đôi
một khác nhau. Chứng minh rằng trong các tam giác có đỉnh là ba điểm
trong số sáu điểm này luôn tồn tại một tam giác mà cạnh nhỏ nhất của
nó là cạnh lớn nhất của một tam giác khác có các đỉnh là ba trong số
sáu điểm đã cho.

2. Chứng minh rằng trong sáu người bất kì luôn tồn tại ba người đôi một
quen nhau hoặc đôi một không quen nhau.

3. Có 17 nhà bác học viết thư cho nhau, mỗi người đều viết thư cho tất cả
các người khác. Các thư chỉ trao đổi về ba đề tài, từng cặp hai nhà bác

129
học chỉ viết thư cho nhau về cùng một đề tài. Chứng minh rằng không
ít hơn ba người viết thư cho nhau về cùng một đề tài. (Vô địch Toán
quốc tế lần thứ sáu)

4. Chia các số của tập hợp {1; 2; · · · ; 16} vào ba tập hợp. Chứng minh rằng
tồn tại trong chúng hai số a và b cùng một tập hợp sao cho số |a − b|
cũng thuộc tập hợp này.

5. Một hội quốc tế có hội viên thuộc sáu nước khác nhau. Danh sách các
hội viên gồm 1978 người đánh số thứ tự 1, 2, · · · 1978. Chứng minh rằng
ít nhất một hội viên mà số thứ tự bằng tổng các số thứ tự của hai hội
viên thuộc cùng một nước với hội viên đó, hoặc bằng hai lần số thứ tự
của một hội viên thuộc cùng một nước với hội viên đó. (Kì thi vô địch
Toán quốc tế năm 1978)

2.3 Tô màu các miền

2.3.1 Giả thuyết Hardwiger

Định nghĩa 8. Đồ thị G0 là rút gọn của một đồ thị G cho trước nếu G0 có
thể thu được từ G bằng cách bỏ bớt đỉnh, cạnh hoặc đồng nhất 2 cạnh của đỉnh
bậc 2 thành một cạnh duy nhất .

Giả thuyết Hardwiger. Mọi đồ thị có sắc số χ(G) = k đều có thể rút
gọn thành đồ thị đầy đủ k đỉnh.
Giả thuyết Hardwiger vẫn chưa được chứng minh cho trường hợp k tổng
quát. Mới được chứng minh cho k = 5.

130
3 Bài toán 4 màu

3.1 Lịch sử ra đời

Vấn đề này lần đầu tiên được đề cập vào năm 1852 bởi Francis Guthrie
khi ông thử tô màu bản đồ nước Anh và ông nhận ra rằng chỉ cần bốn màu
khác nhau là đủ. Ông đã đem vấn đề này hỏi người anh trai là Fredrick, lúc
đó đang là sinh viên của trường Đại học Học viện London (UCL). Fredrick
đã đưa vấn đề này hỏi thầy của mình là nhà toán học Augustus De Morgan
nhưng người thầy cũng chưa biết rõ vấn đề này.
Người đầu tiên giới thiệu vấn đề ra trước công chúng là nhà toán học
Arthur Cayley vào năm 1878 tại Hội Toán học London, ông đã chỉ ra người
đề cập vấn đề là De Morgan.
Người đầu tiên chứng minh định lý này là Alfred Kempe vào năm 1879.
Năm 1880, có thêm một cách chứng minh khác của Peter Guthrie Tait. Nhưng
đến năm 1890 Percy Heawood đã chỉ ra sai lầm trong cách chứng minh của
Kempe, và đến năm 1891 Julius Petersen chỉ ra sai lầm trong cách chứng
minh của Tait.
Trong việc chỉ ra sai lầm của Kempe, Heawood còn chứng minh rằng tất
cả các Đồ thị phẳng có thể tô được bởi năm màu khác nhau.
Trong những năm 1960 và 1970, nhà toán học người Đức là Heinrich Heesch
đã phát triển phương pháp sử dụng máy vi tính cho việc chứng minh vấn đề.
Năm 1976, cuối cùng thì định lý cũng được chứng minh bởi Kenneth Appel
và Wolfgang Haken tại trường Đại học Illinois với sự trợ giúp của máy vi tính.
Bài toán 4 màu. Người ta có thể dùng 4 màu để tô các miền của một
đồ thị phẳng sao cho không có 2 miền nào cùng màu có cạnh chung?
Lưu ý. Bài toán 4 màu là trường hợp đặc biệt của giả thuyết Hardwiger.

131
Hình 3: Bản đồ không tô được bởi 4 màu.

Định lý bốn màu là định lý lớn đầu tiên được chứng minh bằng máy vi
tính. Tuy nhiên một số nhà toán học không đồng tình với cách chứng minh
này, bởi vì con người không thể kiểm chứng trực tiếp được cách chứng minh.
Do vậy, muốn tin vào chứng minh này thì người ta phải công nhận sự chính
xác của Trình biên dịch và phần cứng máy tính được sử dụng để chạy chương
trình chứng minh.
Tuy vậy cũng phải nói rõ là định lý chỉ đúng với những quốc gia là vùng
miền liên tục, không đúng trong trường hợp các quốc gia có thể cấu tạo từ
các vùng đất xé lẻ như trong hình 3.

BÀI TẬP

1. Trên mặt phẳng ô vuông có đánh dấu một số ô vuông. Chứng minh rằng
có thể dùng 4 màu để tô màu các ô vuông này sao cho không có 2 ô
vuông nào cùng màu có đỉnh chung.

2. Trên mặt phẳng có một đồ thị phẳng các mặt là các tam giác được tô

132
màu bởi 2 màu đen trắng sao cho các mặt cùng màu không có cạnh
chung. Chứng minh rằng có thể đánh số các đỉnh của nó bởi 1, 2 và 3
sao cho mỗi tam giác không có 2 đỉnh nào được đánh bởi cùng một số.

3. Trên mặt phẳng có một đồ thị phẳng các mặt là các tam giác mà các
đỉnh của nó được đánh số bởi 1, 2 và 3 sao cho mỗi tam giác không có 2
đỉnh nào được đánh bởi cùng một số. Chứng minh rằng có thể tô màu
các mặt bởi 2 màu đen trắng sao cho các mặt cùng màu không có cạnh
chung.

4. Chứng minh rằng các mặt của đồ thị phẳng tô được 4 màu sao cho
không có 2 miền nào cùng màu có cạnh chung nếu như nó có chu trình
Hamilton (chu trình đi qua tất cả các đỉnh của đồ thị).

Tài liệu
[1] Sach, H, Graphentheorie Leipzig 1970.

[2] Flachsmeyer, J., Kombinatorik VEB Deutscher Verlag der Wis-


senschaften Berlin 1972.

[3] Vũ Đình Hòa, Toán rời rạc, NXB ĐHSPHN 2010.

133
Một cách tiếp cận tới bài toán tổ hợp

Nguyễn Vũ Lương
Trường Trường THPT Chuyên KHTN, Hà Nội

Thực chất bài giảng là một sự chia sẻ kinh nghiệm giảng dạy nội dung "Tổ
hợp" của tác giả và các đồng nghiệp. Trong bài này tác giả trình bày một
cách tiếp cận của mình tới các bài toán tổ hợp theo sơ đồ:
1. Cơ bản:
+ Khái niệm, nguyên lý cơ bản, nhị thức Newton
+ Tính tổng
+ Bài toán đếm
+ Nguyên lý Diricle.
Tuy nhiên phụ thuộc vào thời gian tác giả trình bày "bài toán đếm" làm minh
họa.
2. Nâng cao:
+ Một số kỹ năng giải bài toán tổ hợp
+ Một ví dụ về cách xây dựng một bài toán tổ hợp mới.

1 Phần cơ bản

Phần này trình bày các kiến thức, kỹ năng giải cơ bản giúp cho học sinh
làm quen với các bài toán cơ bản và dễ tiếp thu các bài toán khó hơn sau này.

134
Chúng ta xét một số ví dụ minh họa:
*) Nguyên lý bao gồm và loại trừ.

Bài toán 1.1 (China 1991). Cho tập S = {1, 2, 3, . . . , 280}, tìm số nguyên n
nhỏ nhất sao cho mọi tập con n phần tử của S đều chứa 5 số đôi một nguyên
tố cùng nhau.

Giải. Ta ký hiệu
A1 = {k ∈ S : 2|k}

A2 = {k ∈ S : 3|k}

A3 = {k ∈ S : 5|k}

A4 = {k ∈ S : 7|k}

và A = A1 ∪ A2 ∪ A3 ∪ A4 .
Ta có: |A1 | = 140, |A2 | = 93, |A3 | = 56, |A4 | = 40
Tương tự
|A1 ∩ A2 | = 46, |A1 ∩ A3 = 28|, |A1 ∩ A4 | = 20
|A2 ∩ A3 | = 18, |A2 ∩ A4 | = 13, |A3 ∩ A4 | = 8
|A1 ∩ A2 ∩ A3 | = 9, |A1 ∩ A2 ∩ A4 | = 6
|A1 ∩ A3 ∩ A4 | = 4, |A2 ∩ A3 ∩ A4 | = 2
|A1 ∩ A2 ∩ A3 ∩ A4 | = 1.

Áp dụng nguyên lý bao gồm và loại trừ ta có số phần tử của A bằng

|A| = (140 + 93 + 56 + 40)−


−(46 + 28 + 20 + 18 + 13 + 8) + (9 + 6 + 4 + 2) − 1 = 216.

135
Với 5 số hạng bất kỳ trong A theo nguyên lý Dirichlet phải có 2 số thuộc
Ai nào đó (1 ≤ i ≤ 4). Suy ra số này không nguyên tố cùng nhau. Suy ra
n > 216.
Mặt khác đặt B1 = A r {2, 3, 5, 7}
B2 = {112 , 11.13, 11.17, 11.19, 11.23, 132 , 13.17, 13.19}
và P = S r {B1 ∪ B2 }

Ta có |P | = |S| − |B1 | − |B2 | = 60 và P bao gồm tất cả các số nguyên tố


trong S và 1.
Giả sử T là một tập hợp con của S và có số phần tử bằng |T | = 217. Ta chứng
minh T chứa 5 số đôi một nguyên tố cùng nhau.
Hiển nhiên trong trường hợp |T ∩ (S r P )| ≥ 217 − 4 = 213 tức là trong tập
hợp các hợp số của S(|S r P | = 220) có nhiều nhất 7 số không có trong T . Kí
hiệu
M1 = {2.23, 3.19, 5.17, 7.13, 11.11}
M2 = {2.29, 3.23, 5.19, 7.17, 11.13}
M3 = {2.31, 3.23, 5.23, 7.19, 11.17}
M4 = {2.37, 3.31, 5.29, 7.23, 11.19}
M5 = {2.41, 3.37, 5.31, 7.29, 11.23}
M6 = {2.43, 3.41, 5.37, 7.31, 11.29}
M7 = {2.47, 3.43, 5.41, 7.37, 11.31}
M8 = {22 , 32 , 52 , 72 , 132 }
Hiển nhiên Mi ⊂ S r P (i = 1, 8)
Theo nguyên lý Dirichlet có tồn tại 1 ≤ i0 ≤ 8 sao cho Mi0 ⊂ T và khi đó 5 số
của Mi0 là đôi một nguyên tố cùng nhau.
Đáp số n = 217.

136
Nhận xét: Bài toán trên sẽ rất khó nếu học sinh không thành thạo một
kỹ năng đếm là nguyên lý bao gồm và loại trừ. Trong phần này chúng ta
trình bày một nội dung vừa rất cơ bản (giành cho các kỳ thi đại học) và cũng
có thể rất khó như bài toán trên. Những kỹ năng tuy đơn giản nhưng thực
sự cần thiết và không thể thiếu được khi tìm lời giải. Thật khó đánh giá kỹ
năng nào là quan trọng nên chúng ta tạm sắp xếp theo trình tự từ đơn giản
được sử dụng nhiều đến phức tạp.

I. Phương pháp liệt kê


Chia tập hữu hạn cần đếm thành những tập con rời nhau dễ đếm hơn.

Ví dụ 1. Xét tập {0, 1, 2, 3, 4, 5, 6, 7, 8}, tìm số các số gồm 3 chữ phân biệt
của A, chia hết cho 3.

Giải. Ta có {0, 3, 6} là các số chia hết cho 3


{1, 4, 7} chia hết cho 3 dư 1
{2, 5, 8} chia cho 3 dư 2
Tổng 3 chữ số chia hết cho 3 gồm các trường hợp:
Trường hợp 1 (3 số chia hết cho 3)
d1 = 3! − 2! = 4 (trừ trường hợp số 0 đứng đầu)
Trường hợp 2 (3 số chia 3 dư 1)
d2 = 3! = 6
Trường hợp 3 (3 số chia 3 dư 2)
d3 = 3! = 6
Trường hợp 4 (1 số dư 0, 1 số dư 1, 1 số dư 2 )
Có hai cách chọn 1 số chia 3 dư 0 khác 0, có 3 cách chọn 1 số chia 3 dư 1, 3

137
cách chọn 1 số chia 3 dư 2. Vậy số cách chọn là 2.3.3 (quy tắc nhân). Nên

d4 = 2.3.3.3! = 108.

Do đó, 1 số chia hết cho 3 là 0, 3 cách chọn 1 số chia 3 dư 1, 3 cách chọn 1


số chia 3 dư 2. Số cách chọn 3 × 3
Suy ra
d5 = 3.3(3! − 2!) = 36

Đáp số: d = 4 + 6 + 6 + 108 + 36 = 160.

Ví dụ 2. Kí hiệu A là tập tất cả các số giữa 1 và 700 và chia hết cho 3,


B là tập các số giữa 1 và 300 chia hết cho 7. Tìm số các cặp có thứ tự (a, b)
sao cho a ∈ A, b ∈ B, a 6= b, a + b là số chẵn.
Giải. Ta có A có 233 phần tử (3, 6, 9, . . . , 699) trong đó có 177 số lẻ, 116 số
chẵn.
B có 42 phần tử (7, 14, 21, . . . , 294) trong đó có 21 số lẻ, 21 số chẵn.
A ∩ B có 14 phần tử (3.7, 3.7.2, . . . , 3.7.14)
Số cặp cần tìm bằng
d = |(a, b)|a ∈ A, b ∈ B, a + b chẵn | − |(a, b) : a ∈ A, b ∈ B, a = b|
suy ra d = |(a, b)|a ∈ A, b ∈ B a chẵn, b chẵn | + |(ab) : a ∈ A, b ∈ B, a lẻ, b lẻ
−|(a, b)|a ∈ A, b ∈ B, a = b|
= 116.21 + 117.21 − 14 = 4879.

II. Đếm các phần tử của phần bù

Khi số các phần tử của phần bù dễ đếm, chúng ta xác định số phần tử
của tập cần đếm A qua số phần tử của phần bù.

|A| = |X| − |X\A| (A là tập con của X).

138
Ví dụ 3. Xét tập {0, 1, 2, 3, 4, 5}, tìm số các số gồm 4 chữ số của A sao cho
trong 4 chữ số có ít nhất 2 số giống nhau.

Giải. Số các số gồm 4 chữ số của A là: d1 = 5.63 .


Ta tìm số các số gồm 4 chữ số phân biệt của A.
Có C64 .4! bộ 4 số phân biệt của A. Ta phải trừ đi số bộ 4 số có số 0 đứng đầu
bằng C53 .3!. Vậy số các số gồm 4 chữ số phân biệt của A bằng

(C64 .4! − C53 .3!) = d2

Số các số thỏa mãn đề bài bằng

d = d1 − d2 = 5.63 − C64 .4! + C53 .3! = 780 (số).

Ví dụ 4. Xét đa giác đều 10 đỉnh, hỏi có bao nhiêu tam giác có 3 cạnh là 3
đường chéo đa giác.
3 tam giác có 3 đỉnh là đỉnh đa giác. Những tam giác này
Giải. Ta có C10
chia làm 3 loại.
Loại 1: Những tam giác chỉ có một cạnh là đường chéo và 2 cạnh là 2 cạnh
liên tiếp của đa giác. Ta có d1 = 10.
Loại 2: Tam giác có 2 cạnh là đường chéo. Ta có 10 cạnh. Hai đầu mút của
mỗi cạnh được nối với 6 đỉnh không kề liền tạo thành 6 tam giác. Số tam giác
có 2 cạnh là đường chéo bằng d2 = 10.6 = 60.
Loại 3 (Tam giác có 3 cạnh là đường chéo): chính là loại tam giác cần đếm
theo yêu cầu của đề bài.
Ta có
3 10.9.8
d3 = C10 − (d1 + d2 ) = − (60 + 10) = 50.
3!
III. Sử dụng các công thức tổ hợp

139
Những công thức và các đẳng thức tổ hợp là công cụ rất hiệu quả giúp
chúng ta tìm ra lời giải của bài toán.

Ví dụ 5. Xét một lưới ô vuông (n × n) trên hệ trục tọa độ. Xuất phát
từ điểm (0, 0) ta đi trên các cạnh ô vuông sang phải và lên trên đến điểm
(n × n). Hỏi có bao nhiêu đường đi từ (0, 0) đến điểm (n, n).
Giải. Để đi từ (0, 0) đến điểm (n, n) ta cần đi ngang phải n lần và đi lên
n lần (mỗi lần đi một đoạn bằng 1 là độ dài cạnh của ô vuông). Mỗi lần đi
ngang ta gán số 0 và đi lên ta gán số 1. Như vậy mỗi đường đi tương ứng
1 − 1 với bộ 2n số trong đó có đúng n số 0 và n số 1. Để có một bộ số như vậy
ta chỉ cần chọn ra n vị trí trong 2n vị trí để đặt số 0 và còn lại số 1. Số cách
n .
chọn ra n vị trí trong 2n vị trí bằng C2n
n .
Đáp số C2n

Ví dụ 6. Giả sử X là tập n phần tử, tìm số cặp không thứ tự [A, B] với
A, B là những tập con của X thỏa mãn A 6= B, A ∪ B = X ([A, B] và [B, A] là
trùng nhau và được đếm là 1 cặp).

Giải. Giả sử A có r phần tử 0 ≤ r ≤ n, khi đó có Cnr cách chọn các tập A.


Ứng với tập A thì tập B phải chứa n − r phần tử còn lại của X và một số
phần tử của A. Khi đó B = (X\A) ∪ C (C là tập con của A). Như vậy số cách
chọn B bằng số cách chọn các tập con C của tập A gồm r phần tử bằng

Cr0 + Cr1 + Cr2 + · · · + Crr = 2r

Theo quy tắc nhân thì số cách chọn cặp [A, B] ứng với mỗi r bằng Cnr .2r . Suy

140
ra số cặp phần tử phải tìm (A có thể bằng B ) bằng
n
X
d1 = 2r .Cnr = (1 + 2)n = 3n
r=0

Trong số cách chọn này chỉ có duy nhất một trường hợp A = B = X . Suy ra
số cặp thỏa mãn yêu cầu của bài toán là
Đáp số d = 3n − 1.

IV. Sử dụng công thức truy hồi

Ví dụ 7. Trên mặt phẳng có n đường thẳng sao cho không có hai đường thẳng
song song, không có ba đường thẳng nào đồng quy. Hỏi các đường thẳng chia
mặt phẳng thành mấy phần?
Giải. Kí hiệu S(n) là số phần mặt phẳng được chia bởi n đường thẳng.
Đường thẳng thứ n + 1 theo giả thiết sẽ cắt tất cả n đường thẳng trước tại
đúng n điểm và đường thẳng này bị chia thành (n + 1) đoạn bởi n điểm chia
đó. Mỗi đoạn biến một miền cũ thành 2 miền mới nên số miền tăng lên là
n + 1.
Suy ra
S(n + 1) = S(n) + n + 1

Ta có

S(n) = n + S(n − 1) = n + (n − 1) + S(n − 2) = · · ·

= n + (n − 1) + · · · + 2 + S(1)
n(n + 1)
= 1 + (1 + 2 + · · · + n) = 1 +
2
Ví dụ 8. Xét đa giác đều 12 đỉnh A1 , A2 , . . . , A12 tâm O. Chúng ta tô mầu
các miền tam giác OAi Ai+1 (1 ≤ i ≤ 12) (A13 = A1 ) bằng 4 mầu đỏ, xanh,

141
vàng, đen sao cho hai miền tam giác kề nhau được tô bởi 2 mầu khác nhau.
Hỏi có bao nhiêu cách tô mầu như vậy.

Giải. Có 4 cách tô mầu tam giác thứ nhất, 3 cách tô mầu tam giác thứ 2,
. . . 3 cách tô mầu tam giác thứ 12. Vậy có tất cả 4.311 cách tô. Nếu tam giác
cuối cùng khác mầu với tam giác đầu tiên ta có một cách tô đúng. Nếu tam
giác cuối cùng cùng mầu với tam giác đầu tiên ta nhận được một cách tô sai.
Đối với một cách tô sai ta bỏ đi đỉnh A1 được tam giác cùng màu OA12 A2 và
nhận được một cách tô đúng của đa giác 11 đỉnh. Suy ra công thức truy hồi

S(12) = 4.311 − S(11)

= 4.311 − (4.310 − S(10))

= 4.311 − 4.310 + (4.39 − S(9))

= 4.311 − 4.310 + 4.39 − 4.38 + 4.37 − 4.36 + 4.33 − S(3)

= 312 + 3

(Với S3 = 4.32 − 4.3).

V. Xây dựng phần tử đếm

Để có kết quả chính xác khi đếm chúng ta cần mô tả phần tử đếm sau
đó mới sắp xếp nhờ hoán vị. Như vậy giải bài toán đếm gồm hai bước chính.
1. Mô tả (xây dựng) phần tử đếm
2. Hoán vị

Ví dụ 9. Cho tập A = {1, 2, 3, 4, 5} có bao nhiêu số gồm 6 chữ số trong


đó có 3 số a, 2 số b, 1 số c với a ∈ A, b ∈ B, c ∈ C, a 6= b 6= c.
Giải.

142
Có C53 cách lấy 3 phần tử phân biệt của A
Có 3 cách chỉ định số nào xuất hiện 3 lần, có 3 cách chỉ định một trong 2 số
còn lại xuất hiện 2 lần, còn lại là số chỉ xuất hiện 1 lần.
Suy ra có C53 .3.2 cách chọn ra số xuất hiện 3 lần là số nào, số xuất hiện hai
lần là số nào và số xuất hiện 1 lần là số nào (Ví dụ như (3, 3, 3, 2, 2, 4)).
Hoán vị các bộ số nhận được ta thu được đáp số của bài toán

6!
d = C53 .3.2. = 3600 (số).
3!2!

Ví dụ 10. Cho tập A = {0, 1, 2, 3, 4, 5} có bao nhiêu số gồm 5 chữ số của A


mà mỗi số có đúng 3 chữ số giống nhau.
Giải.
Trước hết ta tìm số bộ 5 chữ số thỏa mãn yêu cầu của bài toán sau đó trừ đi
các bộ số có số 0 đứng đầu.
Có C53 cách chọn 3 vị trí trong 5 vị trí để đặt 3 chữ số giống nhau. Có 6 cách
chọn 1 trong 6 số của A đặt vào 3 vị trí được chọn (gọi số xuất hiện 3 lần là
a ∈ A). Có 5 cách chọn b ∈ A, b 6= A, 5 cách chọn c ∈ A, c 6= a (b, c là hai số còn
lại của bộ 5 số và b, c có thể giống nhau).
Vậy số bộ 5 chữ số thỏa mãn yêu cầu của bài toán là

d1 = C53 .6.5.5.

Xét trường hợp có số 0 đứng đầu, ta xét 4 số còn lại


Trường hợp 1 (3 số giống nhau khác 0). Có C43 chọn 3 vị trí để đặt 3 số giống
nhau, có 5 cách chọn 3 số giống nhau a ∈ A (a 6= 0) là số nào để đặt vào 3 vị
trí đã chọn, có 5 cách chọn b ∈ A, b 6= A (b 6= a) để đặt vào vị trí còn lại. Vậy
số bộ 5 số trong trường hợp này bằng

d2 = C43 .5.5

143
Trường hợp 2 (3 số giống nhau là 3 số 0): Có C42 cách chọn 2 vị trí để đặt
thêm 2 số 0, có 5 cách chọn b ∈ A, b 6= 0, 5 cách chọn c ∈ A, c 6= 0 để đặt vào 2
vị trí còn lại. Vậy số bộ 5 số trong trường hợp này bằng

d3 = C42 .5.5

Đáp số: d = d1 − (d2 + d3 ) = C53 .6.5.5 − C43 .5.5 − C42 .5.5


= 25(60 − 4 − 6) = 25.50 = 1250.

VI. Phương pháp đánh số

Khi chọn các vị trí để sắp xếp các phần tử theo yêu cầu của bài toán đặt ra
phức tạp chúng ta nên đánh số các vị trí và thay thế mỗi cách chọn một bộ
số tương ứng có tính chất tương ứng với các yêu cầu của bài toán. Việc tìm
các bộ số với tính chất cho trước là rất đơn giản.

Ví dụ 11. Một tổ học sinh có 7 nam, 4 nữ, hỏi có bao nhiêu cách sắp
xếp tổ thành một hang ngang sao cho 2 em nữ không đứng cạnh nhau.
Giải.
Ta đánh số các vị trí từ 1 đến 11, khi đó việc chọn 4 vị trí không kề liền nhau
để sắp các em nữ tương ứng với việc chọn 4 số a, b, c, d thỏa mãn tính chất
sau:
4 ≤ a + 3 < b + 2 < c + 1 < d ≤ 11

Để có bộ 4 số (a, b, c, d) thỏa mãn yêu cầu của bài toán ta chỉ cần chọn 4 số
phân biệt a + 3, b + 2, c + 1, d trong 8 số từ 4 đến 8. Số cách chọn bằng C84 .
Suy ra có C84 cách chọn 4 vị trí không kề nhau để xếp các em nữ. Có 4! cách
xếp 4 nữ, 7! cách xếp 7 nam.

144
Vậy số cách xếp bằng
d = C84 .4!.7!

Ví dụ 12. Xét đa giác đều n đỉnh (n ≥ 12), hỏi có bao nhiêu tứ giác có 4
cạnh là 4 đường chéo đa giác.
Giải.
Ta đánh số các đỉnh A1 , A2 , . . . , An tương ứng với các chỉ số.
Ta đếm các tứ giác thỏa mãn yêu cầu của bài toán có đỉnh A1 . Các đỉnh
A2 , An sẽ không được chọn vì A1 A2 , A1 An là cạnh đa giác. Ta cần chọn thêm
3 đỉnh tương ứng với bộ ba số (a, b, c) thỏa mãn tính chất

5≤a+2<b+1<c≤n−1

(Vì giữa 2 đỉnh phải có ít nhất một đỉnh)


Vậy số cách chọn 3 đỉnh bằng số cách chọn 3 số phân biệt trong n − 5 số từ
5 đến (n − 1). Suy ra số các tứ giác đỉnh A1 thỏa mãn yêu cầu của bài toán
3 . Vì có n đỉnh và mỗi tứ giác được đếm lặp lại 4 lần theo 4 đỉnh
bằng Cn−5
nên số tứ giác cần tìm bằng
3
n.Cn−5
d= .
4
VII. Phương pháp xây dựng phép tương ứng 1 - 1

Xét hai tập hữu hạn phần tử A và B, khi đó tồn tại một song ánh (phép
tương ứng 1 - 1) ϕ : A → B khi và chỉ khi |A| = |B|. Như vậy nếu tồn tại song
ánh ϕ : A → B ta có thay thế việc đếm các phần tử của tập A bằng việc đếm
các phần tử của B .

Ví dụ 13. Tìm số bộ 3 số nguyên dương (x, y, z) thỏa mãn đẳng thức

x + y + z = 100 (1)

145
Giải.
Mỗi bộ số nguyên dương (x, y, z) thỏa mãn (1) ta tương ứng 1 - 1 với bộ

(11 . . . 1 0 |11{z
. . . 1} 0 |11{z
. . . 1})
| {z }
x số y số z số

Để có một bộ số như trên ta cần chọn 2 vị trí không kề nhau trong 102 vị trí
để đặt 2 số không, còn lại đặt số 1. Hai vị trí cần tìm tương ứng với 2 số a, b
thỏa mãn
3 ≤ a + 1 < b ≤ 101.

(Vì 2 số 0 không thể đứng đầu và đứng cuối)


Như vậy số bộ số thỏa mãn yêu cầu của bài toán bằng số cách chọn 2 số phân
biệt trong 99 số từ 3 đến 101.
2 .
Đáp số d = C99

Ví dụ 14. Xác định số cách chọn bộ 5 số từ tập 18 số nguyên dương đầu tiên
sao cho bất kỳ một cặp 2 số trong 5 số được chọn có hiệu số giữa số lớn và số
bé lớn hơn hoặc bằng 2.

Giải.
Kí hiệu A là tập hợp các bộ 5 số (a1 , a2 , a3 , a4 , a5 ) thỏa mãn yêu cầu của bài
toán.
Kí hiệu B là tập hợp các bộ 5 số phân biệt của 14 số nguyên dương đầu tiên.
Ta xây dựng ánh xạ ϕ : A → B theo quy tắc sau:

ϕ(a1 , a2 , a3 , a4 , a5 ) = (a, a2 − 1, a3 − 2, a4 − 3, a5 − 4)

trong đó a1 < a2 < a3 < a4 < a5 .

146

a2 − a1 ≥ 2 ⇒ a2 − 1 ≥ a1 + 1 > a1

a3 − a2 ≥ 2 → a3 − 2 ≥ a2 > a2 − 1

a4 − a3 ≥ 2 → a4 − 3 ≥ a3 − 1 > a3 − 2

a5 − a4 ≥ 2 → a5 − 4 ≥ a4 − 2 > a4 − 3

a1 ≥ 1, a5 − 4 ≤ 18 − 4 = 14

⇒ (a1 , a2 − 1, a3 − 2, a4 − 3, a5 − 4) ∈ B

Như vậy với mỗi phần tử của A được ứng với duy nhất một phần tử của B
qua ánh xạ ϕ.
Tương tự với (b1 , b2 , b3 , b4 , b5 ) ∈ B luôn tồn tại duy nhất

(b1 , b2 + 1, b3 + 2, b4 + 3, b5 + 4) = (b1 , b2 , b3 , b4 , b5 )

Suy ra ϕ là phép tương ứng 1 - 1 giữa A và B .


5 .
Vậy số phần tử của A bằng số phần tử của B bằng C14
5 .
Đáp số d = C14

VIII. Sử dụng nguyên lý bao gồm và loại trừ

Khi chúng ta biểu diễn tập cần đếm


n
[
A= Ai
i=1

mà các tập Ai có giao khai tập trống (Ai ∩ Aj 6= ∅) thì khi đó chúng ta không
thể sử dụng quy tắc cộng mà chúng ta cần xây dựng một quy tắc đếm mới
như sau:
Xét hai tập hữu hạn A, B mà A ∩ B 6= ∅, nếu ta lấy số phân tử của A cộng với

147
số phần tử của B thì các phần tử thuộc A ∩ B sẽ được đếm 2 lần.
Suy ra
|A ∪ B| = |A| + |B| − |A ∩ B|

Xét 3 tập hữu hạn A, B, C, A ∩ B 6= ∅, B ∩ C 6= ∅, C ∩ A∅, A ∩ B ∩ C 6= ∅, khi đó


các phần tử của A ∩ B, B ∩ C, C ∩ A được đếm 2 lần, các phần tử của A ∩ B ∩ C
được đếm 3 lần.

Suy ra

|A ∪ B ∪ C| = |A| + |B| + |C| − |A ∩ B| − |B ∩ C| − |C ∩ A| + |A ∩ B ∩ C|

Mở rộng kết quả trên ta thu được kết quả cơ bản sau gọi là nguyên lý bao
gồm và loại trừ.

Định lý 1. Giả sử M1 , M2 , M3 , . . . , Mn là những tập hữu hạn cho trước, khi


đó n
[ X
| Mi | = (−1)r+1 |Mj1 ∩ Mj2 ∩ Mj3 ∩ · · · ∩ Mjr | (1)
i=0

trong đó tổng được lấy theo tổng tất cả các bộ khác trống {j1 , j2 , .., jn } của
tập chỉ số {1, 2, 3, . . . , n}.
Chứng minh.
Tn
Ta sẽ chứng minh mọi phần tử bất kỳ của | i=1 Mi |
được đếm một lần
theo công thức 1. Giả sử m là một phần tử bất kỳ m ∈ ni=1 Mi . Kí hiệu
T

Mk1 , Mk2 , . . . , Mks là các tập chứa m.


Có Cs1 tập chứa m (m được đếm 1 lần)
Có Cs2 tập giao của 2 tập chứa m (m được đếm 2 lần)
................
Có Css tập giao của S tập chứa m (m được đếm s lần)

148
Theo công thức (1) số lần được đếm của m bằng

d = Cs1 − Cs2 + C53 − · · · + (−1)s+1 Css

Ta có
s
X
s
(1 + x) = Csk xk
k=0

Chọn x = −1 ta thu được

0 = Cs0 − Cs1 + Cs2 − · · · + (−1)s Css

Suy ra
Cs1 − Css + Cs3 − · · · + (−1)s+1 Css = Cs0 = 1 = d (đpcm).

Thực chất phương pháp chứng minh chính là kỹ năng giúp chúng ta giải một
số dạng bài toán đếm khá phức tạp sau đây.

Ví dụ 15. Có bao nhiêu cách sắp xếp từ bộ chữ cái MAYMAN sao cho
mỗi cách sắp xếp 2 chữ cái giống nhau không được đứng cạnh nhau.

Giải.
Kí hiệu M1 là số cách sắp xếp mà 2 chữ M đứng cạnh nhau
M2 là số cách sắp xếp mà 2 chữ A đứng cạnh nhau
d0 là tất cả các cách sắp xếp.
Khi đó số các cách sắp xếp phải tìm bằng

d = d0 − |M1 ∪ M2 | = d0 − (|M1 | + |M2 |) + |M1 ∩ M2 |

Ta có
6!
d0 = = 36.5 = 180
2!2!

149
5!
Hai chữ A đứng cạnh nhau coi là một chữ, ta có MAYMN có = 5.4.3 =
2!
60 = M2 , tương tự M1 = 60.
M1 ∩M2 ta coi 2 chữ M, 2 chữ A đứng cạnh nhau như một chữ ta có |M1 ∩M2 | =
4! (Hoán vị của AYMN).
Suy ra
d = 180 − 120 + 24 = 84.

Ví dụ 16. Chứng minh rằng bản báo cáo thành tích cuối năm của một lớp
sau đây là sai.
"Lớp có 45 học sinh, trong đó có 30 em nam. Lớp có 30 em đạt loại giỏi
và trong số này có 16 nam.Lớp có 25 em chơi thể thao và trong số này có 18
em nam và 17 em đạt loại giỏi. Có 15 em nam vừa đạt loại giỏi và chơi thể thao.

Giải.
Kí hiệu
R = 45 số học sinh của lớp
A- số học sinh nam
B - số học sinh đạt giỏi
C - số học sinh chơi thể thao
Khi đó

n = |R| − |A ∪ B ∪ C|

= |R| − (|A| + |B| + |C|) + (|A ∩ B| + |B ∩ C| + |C ∩ A|) − |A ∩ B ∩ C|

= 45 − (30 + 30 + 25) + 16 + 18 + 17) − 15 = −4(mâu thuẫn)

Ví dụ 17. Có bao nhiêu số nguyên dương là bội của 3 hoặc 4 và không chia
hết cho 5 trong 2001 số nguyên dương đầu tiên.

150
Giải.
2001
Có [ ] = 667 số là bội của 3
3
2001
[ ] = 500 là số bội của 4
4
2001
Nhưng có [ ] = 166 bội của 3.4
3.4
Vậy số các số bội của 3 hoặc 4 bằng

667 + 500 − 166 = 1001


2001
Có [ ] = 133 số là bội của 3 và 5
15
2001
Có [ ] = 100 số là bội của 4 và 5
20
2001
Có [ ] = 33 số là bội của 3,4 và 5
60
Kí hiệu
A là tập các bội của 3 chia hết cho 5
B là tập cac bội của 4 chia hết cho 5

⇒ |A ∪ B| = |A| + |B| − |A ∩ B|

= 133 + 100 − 33 = 200

Số các số bội của 3 hoặc 4 không chia hết cho 5 bằng

d = 1001 − 200 = 801

IX. Nguyên lý phân phối các phần tử vào hộp

Định lí 2. Giả sử có N đối tượng khác nhau và K hộp khác nhau T1 , T2 , . . . , Tk .

151
Ta xếp n1 đối tượng vào hộp I1 , n2 đối tượng vào hộp T2 , . . . , nk đối tượng vào
hộp Tk , với n1 + n2 + · · · + nk = N . Giả sử thứ tự giữa các đối tượng trong từng
hộp là không quan tâm, khi đó số tất cả các phân phối các phần tử vào hộp
bằng
N!
P (n1 , n2 , . . . , nk ) = .
n1 !n2 ! . . . nk !

Giải.
Mỗi cách xếp vào hộp được mô tả như sau (tương ứng 1 - 1)
Xếp N phần tử thành một hàng ngang
Xếp n1 đối tượng đầu vào hộp T1
Xếp n2 đối tượng tiếp theo vào hộp T2
...............
Xếp nk đối tượng cuối cùng vào hộp Tk
Vì không quan tâm đến thứ tự của các phần tử trong từng hộp nên hoán vị
của n1 phần tử đầu, n2 phần tử tiếp theo,. . . , nk phần tử cuối cùng trong dãy
hàng ngang không tạo nên cách phân bố mới. Như vậy số cách phân bố các đối
tượng vào hộp bằng số cách hoán vị có lập của N phần tử gồm n1 , n2 , . . . , nk
phần tử giống nhau n1 + n2 + · · · + nk = N .
Ta có
N!
P (n1 , n2 , . . . , nk ) =
n1 !n2 ! . . . nk !

Định lý 3. Xếp N đối tượng vào hộp theo quy tắc sau:
Xếp vào j1 hộp mỗi hộp m1 phần tử (đối tượng)
Xếp vào j2 hộp mỗi hộp m2 đối tượng,. . .
Xếp vào jα hộp mỗi hộp mα đối tượng.
Trong đó j1 + j2 + · · · + jα = k, m1 j1 + m2 j2 + · · · + mα jα = N . Không quan
tâm đến thứ tự của các phần tử trong từng hộp thì số cách phân phối N đối

152
tượng vào k hộp bằng
N!
P (m1 , m2 , . . . , mα , j1 , j2 , . . . jα ) =
(m1 !)j1 .(m2 )j2 . . . (mα !)jα j1 !j2 ! . . . jα !

Giải.
Ta mô tả một cách xếp các đối tượng vào hộp thỏa mãn yêu cầu của bài toán
như sau:
Xếp N phần tử thành một hàng ngang và xếp vào hộp.
Vì thay đổi thứ tự các đối tượng trong từng hộp hoặc thay đối vị trí của jk
hộp có số phần tử bằng nhau không tạo ra phân bố mới nên số phân bố trong
trường hợp này bằng

N!
P (m1 , m2 , . . . ,α , j1 , . . . , jα ) =
(m1 !)j1 .(m 2 !)j2 . . . (mα !)jα .j1 !j2 ! . . . jα
Xét trường hợp m1 = m2 = m3 = · · · mk = n. Khi đó N = kn đối tượng xếp
vào k hộp khác nhau T1 , T2 , . . . , Tk bằng
(kn)!
P1 (n, k) =
(n!)k

Xét trường hợp m1 = m2 = m3 · · · = mα = n được phân phối vào k hộp mà


thứ tự giữa k hộp là không quan tâm bằng
(kn)!
P2 (n, k) =
k!(n!)k

Sau đây chúng ta xét một số trường hợp cụ thể ứng dụng các kết quả này.

Ví dụ 18. Có 10 cặp vợ chồng đi du lịch, các cặp vợ chồng đi trên 5 con


thuyền nhỏ, mỗi thuyền chở được 4 người. Hỏi có bao nhiêu cách xếp sao cho
mỗi thuyền có đúng 2 nam và 2 nữ.
Giải.

153
Đầu tiên ta xếp 10 nam vào 5 thuyền mỗi thuyền 2 nam. Có 10! cách xếp 10
nam thành một hàng ngang. Hai nam đầu ta xếp vào 1 trong 5 thuyền, hai
nam xếp sau ta xếp vào 1 trong 4 thuyền còn lại,. . . , hai nam cuối cùng ta
xếp vào thuyền thứ 5. Vì thay đổi thứ tự 2 nam trong một thuyền không thu
được cách xếp mới. Vì thay đổi thứ tự giữa các thuyền cùng không thu được
10!
cách xếp mới. Suy ra số cách xếp 10 nam vào 5 thuyền bằng P1 = .
5!(2!)5
Sau khi xếp vào mỗi thuyền 2 nam thì mỗi thuyền được đánh số T1 , T2 , T3 , T4 , T5
10!
vì tương ứng với 2 nam cụ thể đã xếp. Khi đó có cách xếp 10 em nữ
(2!)5
vào 5 thuyền T1 , T2 , T3 , T4 , T5 nên trên. Theo quy tắc nhân thì số cách sắp xếp
bằng
10! 10! (10!)2
d= · =
5!(2!)5 (2!)5 5!(2!)10
Chú ý: Khi giữa các bài toán mà số phần tử của mỗi cặp hộp bằng nhau
chúng ta phải phân biệt 2 trường hợp:
Trường hợp 1: Các hộp có đánh số T1 , T2 , . . . , Tk thì mỗi hoán vị của kn = N
phần tử.

Khi thay đổi vị trí của các phần tử của n phần tử trong các bộ T1 , T2 , . . . , Tk
là thu được các hoán vị mới nhưng vẫn cùng một cách xếp vào hộp như trên.
(kn)!
Suy ra số cách xếp là
(n!)k
Trường hợp 2: k hộp phân biệt ta không đánh số hay đặt tên trước nên thay
đổi vị trí giữa các bộ n phần tử không thu được hoán vị mới nhưng vẫn cùng
một cách xếp nên suy ra số cách xếp là
(kn)!
k!(n!)k

Ví dụ 19. Có 6 cặp vợ chồng đi du lịch trên 3 con thuyền nhỏ, mỗi thuyền
4 người. Có bao nhiêu cách xếp mà cặp vợ chồng (X, Y ) được ngồi với nhau

154
trên cùng một thuyền.
Giải.
Xét một thuyền có cặp vợ chồng (X, Y ), ta cho 5 cách chọn 1 nam và 5 cách
chọn 1 nữ từ 5 cặp còn lại vào thuyền này. Còn 2 thuyền còn lại và 4 nam, 4
(4!)2
nữ nên tương tự như ví dụ 1.6.18 ta có .
2!(2!)4

Suy ra số cách sắp xếp bằng


(4!)2
d = 52 · .
2.(2!)4

Ví dụ 20. Từ 24 chữ cái khác nhau, có bao nhiêu cách xây dựng 6 từ mỗi từ
gồm 4 chữ cái phân biệt.
Giải.
Xếp 24 chữ cái thành một hàng ngang, 4 chữ đầu ta được từ đầu, 4 chữ tiếp
ta được từ thứ 2, . . . , 4 chữ cuối cùng ta được từ thứ 6. Thay đổi thứ tự giữa
các nhóm 4 chữ trong một từ ta vẫn thu được 6 từ như cũ nhưng lại là hoán
vị mới. Vậy số cách xây dựng bằng
24!
d= .
6!

Bài tập và hướng dẫn

Bài 1. Cho số a = 111224444 hỏi khi thay đổi các vị trí các chữ số của a chúng
ta nhận được bao nhiêu số mà hai số 2 không đứng cạnh nhau.
Hướng dẫn.
7!
Có d1 = cách xếp 3 số 1, 4 số 4 thành một hàng ngang. Ứng với mỗi cách
3!4!
sắp xếp có 6 vị trí xen kẽ và hai vị trí đầu cuối. Để nhận được một cách thay

155
đổi vị trí thỏa mãn yêu cầu của bài toán chúng ta chỉ cần chọn 2 vị trí trong
8 vị trí xen kẽ hoặc đầu cuối để đặt vào số 2. Suy ra số các số nhận được bằng
7!
d= · C82 = 980.
3!4!
Bài 2. Xét từ TOANTIN, hỏi có bao nhiêu cách đảo chữ mà các nguyên âm
không đứng cạnh nhau.
Hướng dẫn.
4!
Có d1 = cách xếp phụ âm TNTN thành một hàng ngang. Để có một cách
2!2!
đảo chữ thỏa mãn yêu cầu của bài toán chúng ta cần chọn 3 vị trí trong 5 vị
trí (3 vị trí xen kẽ, 1 vị trí đầu, 1 vị trí cuối) để xếp 3 nguyên âm khác nhau
A, O, I . Có 3! cách xếp 3 nguyên âm. Theo quy tắc nhân thì số cách đảo chữ
bằng
4!
d= .C 3 .3! = 6.10.6 = 360 (số).
2!2! 5
Bài 3. Có bao nhiêu cách xếp 4 học sinh nam, 7 học sinh nữ vào 11 ghế xung
quanh một bàn tròn sao cho không có hai học sinh nam ngồi cạnh nhau.
Hướng dẫn.
Ta lấy một ghế bất kì đánh số là A1 và đánh số các ghế tiếp theo là A2 , A3 , . . . , A11 .
Trường hợp A1 là nam: Khi đó A2 phải xếp nữ, A11 phải xếp nữ và ta đưa về
bài toán "có bao nhiêu cách xếp 3 nam 5 nữ thành một hàng ngang sao cho
2 em nam không ngồi cạnh nhau".
Có 5! cách xếp 5 nữ, có C63 cách lấy 3 vị trí trong 6 vị trí gồm 4 xen kẽ, 1 đầu,
1 cuối, có 3! cách xếp 3 em nam vào 3 vị trí. Số cách sắp xếp trường hợp này
bằng d1 = 5!C63 .3!.
Trường hợp A2 là nữ ta đưa về bài toán "Có bao nhiêu cách xếp 4 nam, 6 nữ
thành một hàng ngang mà không có 2 em nam nào ngồi cạnh nhau".
Tương tự như phần trên ta có số cách xếp bằng

d2 = 6!.C74 .4!

156
Đáp số: d = 5!3!C63 + 6!C74 .4!.

Bài 4. Có bao nhiêu cách phát 5 viên bi đen, 7 viên bi đỏ cho 3 học sinh.
Hướng dẫn.
Ta phát 5 viên bi đen cho 3 học sinh. gọi C1 là số viên bi đen phát cho học
sinh thứ 1, C2 là số viên phát cho học sinh thứ 2, C3 là số viên phát cho học
sinh thứ 3.
Ta có
C1 + C2 + C3 = 5 (Ci ≥ 0)

Mỗi cách phát tương ứng 1 - 1 với bộ 3 số (C1 , C2 , C3 ) thỏa mãn tính chất
trên.
Ta tương ứng bộ (C1 , C2 , C3 ) với bộ

(11 . . . 1 0 |11{z
. . . 1} 0 |11{z
. . . 1})
| {z }
C1 số C2 số C3 số

Nếu Ci = 0 ta không viết số nào.


Bộ các số 0, 1 trên gồm

p = C1 + 1 + C2 + 1 + C3 = C1 + C2 + C3 + 2 = 7 số

Để có 1 bộ số như vậy ta cần chọn 2 vị trí trong 7 vị trí để đặt số 0. Suy ra


số cách phát là C72 .
Tương tự số cách phát 7 viên bi đỏ cho 3 học sinh bằng C92 .
Theo quy tắc nhân thì số cách phát bằng

d = C72 .C92

Bài 5. Xét đa giác đều 2n đỉnh (n ≥ 2), hỏi có bao nhiêu tam giác vuông có
3 đỉnh là đỉnh đa giác.

157
Hướng dẫn.
Đa giác đều 2n đỉnh có n đường chéo.
Với mỗi đường chéo ta cần chọn một trong 2n − 2 đỉnh còn lại để tạo thành
tam giác vuông. Suy ra số tam giác vuông là d = 2n(n − 1).

Bài 6. Xét đa giác đều n đỉnh A1 A2 A3 , . . . , An


1) Hỏi có bao nhiêu đường gấp khúc đi qua tất cả các đỉnh mỗi đỉnh một lần
2) Hỏi có bao nhiêu đường gấp khúc đi qua tất cả các đỉnh mỗi đỉnh một lần
sao cho các đường đều có một đoạn là A1 A3 và một đoạn là An−1 An .
3) Hỏi có bao nhiêu đường gấp khúc đi qua tất cả các đỉnh mỗi đỉnh một lần
sao cho đỉnh A1 đứng trước đỉnh An .
Hướng dẫn.
1) Mỗi đường gấp khúc tương ứng 1 - 1 với một hoán vị của các đỉnh. Suy ra
số đường gấp khúc bằng d1 = n!.
2) Ta coi đoạn A1 A3 và An−1 An là 2 đỉnh mới, suy ra số đường gấp khúc đi
qua các đỉnh, mỗi đỉnh một lần có 2 đoạn A1 A3 , An−1 An bằng

d2 = (n − 2)!

3) Số đường gấp khúc mà A1 đứng trước An bằng số đường gấp khúc mà A1


đứng sau An , suy ra số đường gấp khúc có đỉnh A1 đứng trước An bằng
1
d3 = n!
2

Bài 7. Trong sân có 3 con gà, 4 con vịt, 2 con ngỗng, có bao nhiêu cách chọn
ra một nhóm có ít nhất 1 con gà, 1 con vịt, 1 con ngỗng.
Hướng dẫn.
Để có 1 nhóm thỏa mãn yêu cầu của bài toán ta chọn 1 tập con gà khác rỗng,

158
sau đó chọn 1 tập con vịt khác rỗng, chọn 1 tập con ngỗng khác rỗng.
Số tập con gà khác trống bằng

C31 + C32 + C33 = 23 − 1 = 7

Số tập con vịt khác trống bằng

C41 + C42 + C43 + C44 = 24 − 1 = 15

Số tập con ngỗng khác trống bằng

C21 + C22 = 22 − 1 = 3

Suy ra số cách chọn một nhóm có ít nhất 1 con gà, 1 vịt, 1 ngỗng bằng

d = 7.15.3 = 315

Bài 8. Một tổ gồm 7 nam, 4 nữ, có bao nhiêu cách lập đội bóng chuyền 6
em, trong đó có ít nhất hai em nữ.
Hướng dẫn.
Số đội bóng chuyền có 2 nữ: d1 = C42 .C74
Số đội bóng chuyền có 3 nữ: d2 = C43 .C73
Số đội bóng chuyền có 4 nữ: d3 = C44 .C72
Suy ra số cach lập đội bóng chuyền bằng
7.6.5 7.6.5 7.6
n = 6. · +4· +1·
3! 3! 2!
= 7.6.5 + 4.7.5 + 7.3 = 7(30 + 20 + 3)

= 7.53 = 371

Bài 9. Một vũ hội có 12 nam, 15 nữ, hỏi có bao nhiêu cách chọn 4 cặp, mỗi
cặp gồm một nam, một nữ.

159
Hướng dẫn.
4 cách chọn ra 4 em nam
Có C12
4 cách chọn ra 4 em nữ
Có C15
Bốn em nữ có một cách xếp đội với 4 em nam tương ứng với một hoán vị của
4 em nữ. Suy ra số cách xếp 4 cặp bằng

4 4
d = C12 .C15 .4! = 16.216200.

Bài 10. Năm người nói chuyện ở cùng một hội nghị có tên là A, B, C, D, E .
Hỏi có bao nhiêu cách sắp xếp thứ tự phát biểu để người B phát biểu sau A.
Hướng dẫn.
Tổng số các trình tự phát biểu bằng 5!
Số cách xếp A phát biểu trước B và A phát biểu sau B là bằng nhau. Suy ra
5!
số thứ tự phát biểu mà B phát biểu sau A bằng d = .
2

Bài 11. Tìm số các ước số của n = pα1 1 pα2 2 . . . pαk k trong đó p1 , p2 , . . . , pk là
các số nguyên tố phân biệt, αi ∈ N .

Hướng dẫn.
Một ước số của n có dạng
a = pc11 pc22 . . . pckk

trong đó 0 ≤ ci ≤ αi
Để có một ước số ta có :
(1 + α1 ) cách chọn thừa số đầu tiên

p01 , p11 , p21 , . . . , pα1 1

(α2 + 1) cách chọn thừa số thứ 2


. . . . . . . . . . . . . . . . . . . . . (αk + 1) cách chọn thừa số thứ k .

160
Suy ra số ước của n bằng

r(n) = (α1 + 1)(α2 + 1) . . . (αk + 1).

Bài 12. Tìm số cặp số nguyên không âm (x, y) thỏa mãn


(x + y) = n x ≤ y.

Hướng dẫn.
Nếu n chẵn ta có các nghiệm thỏa mãn bài toán là
n n
(0, n), (1, n − 1), . . . , ( , )
2 2
n
suy ra số nghiệm số bằng + 1.
2
Nếu n lẻ ta có các nghiệm thỏa mãn bài toán là
n−1 n+1
(0, n), (1, n − 1), . . . , ( , )
2 2
n−1 n+1
suy ra số nghiệm số bằng +1= .
2 2
Suy ra số bộ số bằng
n
r = [ ] + 1.
2
Bài 13. Có bao nhiêu cách biểu diễn một số nguyên n > 1 dưới dạng
n = x.y; x, y ∈ N y chia hết cho x.

Hướng dẫn.
Biểu diễn n, x, y thỏa mãn yêu cầu của bài toán dưới dạng tích của các số
nguyên tố ta có

n = pγ11 pγ22 . . . pγkk , x = pα1 1 pα2 2 . . . pαk k , y = pβ1 1 pβ2 2 . . . pβk k

Với p1 , p2 , . . . , pk là các số nguyên tố phân biệt,


γi ∈ N ∗ , αi ∈ N0 , βi ∈ N0 và γi = αi + βi , αi ≤ βi

161
Áp dụng quy tắc nhận và kết quả của bài 12 ta thu được số cách biểu diễn
bằng
k
γ1 γ2 γk Y γi
d = ([ ] + 1)([ ] + 1) . . . ([ ] + 1) = ([ ] + 1).
2 2 2 2
i=1

Bài 14. Một viện nghiên cứu có 67 nhà khoa học trong đó có 47 người sử
dụng tốt tiếng Anh, 35 người sử dụng tốt tiếng Đức, 20 người sử dụng tiếng
Pháp. Hơn nữa có 23 người sử dụng tốt 2 thứ tiếng Anh, Đức, 12 người Anh,
Pháp , 11 người Đức, Pháp và 5 người sử dụng tốt cả 3 thứ tiếng. Tìm số nhà
khoa học không sử dụng bất kì thứ tiếng nào.

Hướng dẫn.
Kí hiệu E là tập các nhà khoa học sử dụng tốt tiếng Anh
G là tập các nhà khoa học sử dụng tốt tiếng Đức
F là tập các nhà khoa học sử dụng tốt tiếng Pháp.
Số các nhà khoa học biết ít nhất một thứ tiếng bằng

|E ∪ G ∪ F | = |E| + |G| + |F | − |E ∩ G| − |E ∩ F | − |G ∩ F | + |E ∩ G ∩ F |

= 47 + 35 + 20 − 23 − 12 − 1 + 5 = 61

Suy ra số nhà khoa học không sử dụng bất cứ ngoại ngữ nào bằng 67 − 61 = 6.

Bài 15. Có bao nhiêu cách đảo chữ của từ MATHEMATICAL sao cho không
có bất kỳ hai nguyên âm nào đứng cạnh nhau.

Hướng dẫn.
7!
Ta xét hoán vị của các phụ âm MTHMTCL. Khi đó có d1 = cách xếp
2!2!
(hoán vị).
Ứng với mỗi hoán vị ta có 8 vị trí đầu, cuối và xen kẽ giữa 2 phụ âm. Để có

162
một cách đảo chữ thỏa mãn yêu cầu của bài toán ta cần chọn ra 5 vị trí để
đặt các nguyên âm AEAIA. Có C85 cách chọn số cách xếp A, E, A, I, A vào 5
5!
vị trí đã chọn bằng d2 = .
3!
Đáp số: Số cách đảo chữ thỏa mãn yêu cầu của bài toán bằng
7! 5!
d= · C85 · = 1411200.
2!2! 3!

Bài 16. Bên trong mỗi cạnh của một hình vuông chúng ta lấy n điểm phân
biệt. Tìm số tam giác có ba đỉnh là các điểm trên.

Hướng dẫn.
3 bộ 3 điểm chọn ra từ 4n điểm. Chúng ta phải trừ ra các bộ 3 điểm
Có C4n
thẳng hàng bằng 4.Cn3 .
Số tam giác phải tìm bằng

3
d = C4n − 4Cn3 = 2n2 (5n − 3).

Bài 17. Có bao nhiêu cách chọn 3 đỉnh của n giác đều A1 A2 . . . An (n ≥ 4)
sao cho 3 đỉnh này lập thành một tam giác tù.

Hướng dẫn.
Kí hiệu P (n) số tam giác tù, P1 (n) là số tam giác tù A1 Ai Aj (1 < i < j ≤ n)
(các tam giác có đỉnh A1 theo thứ tự đánh số). Hiển nhiên ta có

P (n) = nP1 (n).

n+1
tam giác A1 Ai Aj là tù ⇔ j ≤ (xem hình vẽ)
2

vẽ hình

163
Khi đó P1 (n) là số cặp 2 số (i, j) (tập con) của tập
n+1
{2, 3, . . . , [ ]}
2
Ta có P (n) số tam giác tù, p1 (n) là số tam giác tù trong đó có một đỉnh là A1
(tam giác A1 Ai Aj ) với A1 , Aj là 1 đỉnh nhọn, Ai là một đỉnh tù.
nP1 (n)
⇒ Pn = (do mỗi tam giác được đếm 2 lần khi mà tính đỉnh Aj )
2
Xét số tam giác A1 Ai Aj nhận được ở 1 nửa của đường tròn ngoại tiếp n giác
đều (như hình vẽ) suy ra n1 = C[2n+1 ]−1 . Lấy đối xứng qua đường kính đó ta
2

cũng được n1 giác tương tự.


Suy ra P (n) = 2C[2n+1 ]−1 ⇒ P (n) = nC[2n+1 ]−1 .
2 2

Ta thu được
P (n) = n.P1 (n) = n.C[2n+1 ]−1
2

n n+1 n+1
= ([ ] − 1)([ ] − 2)
2 2 2
n n−1 n−3
= [ ][ ]
2 2 2
Bài 18. Giả sử không có 3 đường chéo nào của đa giác lồi n đỉnh đồng quy.
Tìm số tam giác có đỉnh là đỉnh đa giác hoặc giao của các đường chéo (cạnh
nằm trên cạnh đa giác hoặc đường chéo).

Hướng dẫn.
Kí hiệu Si (i = 0, 1, 2, 3) là tập hợp các tam giác có i đỉnh là đỉnh đa giác. Khi
đó ta có |S3 | = Cn3 . Ta tính |S2 |, mỗi tam giác như vậy được cấu tạo như sau:
Mỗi lần lấy 4 đỉnh như trên ta được 4 tam giác phụ thuộc (tương ứng 1 - 1)
với 4 đỉnh này.
Suy ra
|S2 | = 4Cn4 .

164
Ta tính |S1 |, mỗi tam giác như vậy được cấu tạo từ 5 đỉnh
Giải:
n(n − 3)
Số đường chéo từ đa giác lồi n đỉnh đường.
2
C 2 (n − 3)
⇒ số giao điểm của các đường chéo n
2
Cn2 (n − 3)
⇒ số đỉnh giao điểm n + .
2
Số tam giác cần tìm có đỉnh là đỉnh của đa giác hoặc giao điểm của các đường
chéo
C3 2 (n−3)
Cn
n+ 2

Suy ra |S1 | = 5.Cn5 .


Ta tính |S0 |, mỗi tam giác như vậy được cấu tạo từ 6 đỉnh. Suy ra |S0 | = Cn6 .
Đáp số: d = Cn3 + 4Cn4 + 5Cn5 + Cn6 .

2 Một số kỹ năng giải bài toán tổ hợp

Trong phần này chúng ta trình bày một số kỹ năng cơ bản quen thuộc sau
đây:
Kỹ năng 1. Sử dụng thứ tự của dãy các số nguyên phân biệt.
Khi cho một dãy số nguyên phân biệt thì khoảng cách giữa 2 số liên tiếp bằng
1.
Ví dụ ai > aj ⇒ ai ≥ aj + 1. Do vậy nhiều bài toán đối với dãy số nguyên
chúng ta chỉ nhận được lời giải khi sử dụng thứ tự này.

165
Bài toán 2.1. . Giả sử A là tập hợp các số nguyên dương sao cho đới với 2
xy
phần tử a, y ∈ A phân biệt bất kỳ ta luôn có |x − y| ≥ . Chứng minh rằng
25
A có chứa nhiều nhất 9 phần tử. Tìm tập A gồm 9 phần tử thỏa mãn yêu cầu
của bài toán.

Giải. Giả sử x, y ∈ A, y > x ≥ 25 ta có


xy
≤ |x − y| = y − x < y ⇒ x < 25 (Mâu thuẫn).
25
Vậy tập A chỉ có nhiều nhất 1 phần tử lớn hơn 24.
*) Ký hiệu A = {x1 , x2 , . . . , xn } với
1 ≤ x1 < x2 < · · · < xn−1 < xn trong đó xn−1 < 25.
Đặt dj = xj + 1 − xj (1 ≤ j ≤ n − 1) ta có
xj xj+1 xj (xj + dj )
dj ≥ =
25 25
x2j
⇔ 25dj ≥ x2j + xj dj ⇔ dj (25 − xj ) ≥ x2j ⇔ dj ≥ (xj ≤ xn−1 < 25)
25 − xj
x2
Xét hàm số h(x) = là hàm tăng trong [1, 34]
25 − x
x(50 − x)
(h0 (x) = > 0) khi 1 ≤ x ≤ 34 .
(25 − x)2
Ta thu được

x5 ≥ 5(x5 ≥ x4 + 1 ≥ x3 + 2 ≥ x2 + 3 ≥ x1 + 4 ≥ 5

x25 52 5
⇒ d5 ≥ ≥ = = h(5) > 1 ⇒ d5 = x6 − x5 > 1
25 − x5 25 − 5 4
72
⇒ x6 > 1 + x5 > 6 ⇒ x6 ≥ 7 ⇒ d6 ≥ >2
25 − 7
⇒ d6 = x7 − x6 > 2 ⇒ x7 > 2 + x6 ≥ 9 ⇒ x7 ≥ 10

166
102
⇒ d7 ≥ > 6 ⇒ x8 − x7 = d7 > 6
25 − 10
172
⇒ x8 > 6 + x7 ≥ 16 ⇒ x8 ≥ 17 ⇒ d8 ≥ > 36
25 − 17
⇒ x9 − x8 = d8 > 36 ⇒ x9 > 36 + x8 ≥ 36 + 37 = 53

x9 ≥ 54 (lớn hơn 24) ⇒ n ≤ 9 (đpcm).

Ta xây dựng tập A gồm 9 phần tử thỏa mãn yêu cầu của bài toán như
sau:

A = {1, 2, 3, 4, 5, 7, 10, 17, 54}.

Bài toán 2.2. . Giả sử A là tập hợp các số tự nhiên có tính chất:
1) Phần tử nhỏ nhất của A bằng 1, phần tử lớn nhất của A bằng 100.
2) Mọi phần tử khác 1 của A luôn bằng tổng hai số của A (có thể bằng nhau).
Ký hiệu N (A) là số phần tử của A. Hãy tìm giấ trị nhỏ nhất của N (A).

Giải.
*) Giả sử A có n phần tử được xếp theo thứ tự

1 = x1 < x2 < · · · < xn = 100

Với mỗi k ∈ {1, 2, . . . , n−1} ta có xk+1 ≤ xk +xk (Vì theo giả thiết xk+1 = xi +xj
với i ≤ k, j ≤ k )
Suy ra x2 ≤ x1 + x1 = 2, x3 ≤ x2 + x2 = 4.
x4 ≤ x3 + x3 ≤ 4 + 4 = 8, x5 ≤ 8 + 8 = 16, x6 ≤ 16 + 16 = 32, x7 ≤ 32 + 32 = 64.
Suy ra n ≥ 8.
*) Giả sử n = 8 ⇒ x8 = m100 (theo giả thiết).
Theo các ước lượng trên suy ra

167
x6 + x7 ≤ 64 + 32 = 96 ⇒ x8 = x7 + x7
⇒ 2x7 = 100 ⇒ x7 = 50
Ta lại có x5 + x6 ≤ 16 + 32 = 48 ⇒ x7 = x6 + x6 ⇒ 2x6 = 50 ⇒ x6 = 25.
25
Ta lại có x4 + x5 ≤ 8 + 16 = 24 ⇒ x6 = x5 + x5 ⇒ 2x5 = 25 ⇒ x5 = (không
2
là số tự nhiên ⇒ mâu thuẫn).
Suy ra n ≥ 9.
*) Ta xây dựng tập A1 = {1, 2, 3, 5, 10, 20, 25, 50, 100} thỏa mãn. Vậy M in N (A) =
9.

Kỹ năng 2. Lập sơ đồ, đồ thị.


Bài số 3. Tìm số hoán vị a1 , a2 , . . . , a10 của bậc số {1, 2, 3, . . . , 10 sao cho
ai > a2i (với i = 1, 5) và ai > a2i+1 (với i = 1, 4).

Giải. Ta biểu diễn các bất đẳng thức thỏa mãn yêu cầu của bài toán theo
sơ đồ: Mũi tên chỉ từ số lớn hơn đến số nhỏ hơn. Từ sơ đồ suy ra a1 = 10.
Có C93 cách chia 9 số còn lại thành 2 tập {a3 , a6 , a7 } và {a2 , a4 , a5 , a8 , a9 , a10 }.
Lấy 3 số trong 9 số và 2 số lớn nhất của 2 nhóm ta gọi là a2 , a3 .
Có 2 cách chộn a6 , a7 và C52 cách chia các số a1 , a5 , a8 , a10 thành 2 tập {a4 , a8 , a9 }
và {a5 , a10 } trong đó a4 , a5 là lớn nhất trong các nhóm. Có 2 cách chọn a8 , a9
trong nhóm {a4 , a8 , a9 }. Vậy số hoán vị thỏa mãn yêu cầu của bài toán là

P = C93 .2.C52 .2 = 3360.

168
3 Một dạng bất đẳng thức đối xứng xây dựng
bằng cách sử dụng nguyên lý Dirichlet

Kỹ năng 3. Nguyên lý Dirichle và bất đẳng thức Khi chứng minh một
số bất đẳng thức đối xứng phức tạp, do vai trò của các biến như nhau chúng
ta có thể giả thiết các biến số có một thứ tự nào đó và sử dụng thứ tự này mới
thu được cách chứng minh. Trong bài này tác giả sử dụng nguyên lý Diriclet
xây dựng nội dung bất đẳng thức đối xứng mà chúng ta khó nhận được cách
giải khi sử dụng các bất đẳng thức cơ bản quen thuộc. Ngay trong trường hợp
đặc biệt chúng ta nhận được những bất đẳng thức đối xứng đơn giản nhưng
khó tìm ra lời giải nếu không sử dụng các kết quả và phương pháp chứng
minh trong bài báo này.

I. Một số kết quả

Trong n ≥ 3 số (xk − 1)(k = 1, n) sẽ có ít nhất hai số cùng dấu. Sử dụng


nhận xét này chúng ta thu được kết quả sau:
Định lý 1. Giả sử xi (i = 1, 2, . . . , n là những số thực dương, với mỗi số nguyên
dương n ≥ 3 ta có bất đẳng thức

n n
X X 1
(n − 2) xn−1
k + (n − 1)Πnk=1 xk +1≥ (n − 1)Πnk=1 xk . .
xk
k=1 k=1

Chứng minh. Trong các giá trị (xk − 1)(k = 1, 2, . . . , n, n ≥ 3 luôn có hai số
cùng dấu. Do vai trò các biến số như nhau ta có thể giả thiết rằng:

(Πn−2
k=1 xk )(xn−1 − 1)(xn − 1) ≥ 0

169
n−1 n n−1 n
⇔ (n − 1)Πnk=1 xk + (n − 1)Πn−1
k=1 xk ≥ Πk=1 xk + Π xk
xn xn−1 k=1
n n−2
X 1 X 1
⇔ (n−1)Πnk=1 xk +(n−1)Πn−2
k=1 xk ≥ (n−1)Πnk=1 xk −(n−1)Πnk=1 xk . (1)
xk xk
k=1 k=1

áp dụng bất đẳng thức AM - GM ta thu được


n
X 1 n
xn−1 ≥ (n − 1) Π xk
i xk k=1
i=1

Cộng (n − 2) bất đẳng thức ứng với k = 1, 2, . . . , n − 2 ta nhận được


n−2  X
n n−2
1
X  X
xn−1
i ≥ (n − 1)Πnk=1 xk .
xk
k=1 i=1 k=1

n−2 n−2
X X 1
⇔ (n − 3) xn−1
k + (n − 2)[xn−1
n−1 + xn−1
n ] ≥ (n − 1)Πnk=1 xk . (2)
xk
k=1 k=1

áp dụng bất đẳng thức AM - GM ta có


n−2
X
1+ xn−1
k ≥ (n − 1)Πn−2
k=1 xk (3)
k=1

Cộng các bất đẳng thức (1), (2), (3) ta thu được
n n
X X 1
(n − 2) xn−1
k + (n − 1)Πnk=1 xk +1≥ (n − 1)(Πnk=1 xk )( ) (đpcm).
xk
k=1 k=1

Xét trường hợp n = 7 khi đó trong 7 số x1 − 1, x2 − 1, x3 − 1, x4 − 1, x5 − 1, x6 −


1, x7 − 1 sẽ có 4 số cùng dấu. Trong trường hợp này ta xây dựng được bất
đẳng thức đối xứng sau:

Bổ đề 1. Bất đẳng thức x5 x6 x7 (x4 − 1)(x3 − 1)(x2 − 1)(x1 − 1) ≥ 0 tương


đương với bất đẳng thức:

6Πnk=1 xk + 6(x5 x6 + x5 x7 + x6 x7 )x1 x2 x3 x4 + 6(x5 x6 x7 + x5 x6 + x5 x7 + x6 x7 )+

170
+(x1 x2 + x1 x3 + x1 x4 + x2 x3 + x2 x4 + x3 x4 ) + 6(x5 + x6 + x7 )×

×(x1 x2 x3 + x1 x2 x4 + x1 x3 x4 + x2 x3 x4 )+
X X
+x1 x2 x3 x4 + x5 x6 x7 ≥ xi1 xi2 xi3 xi4 + xi1 xi2 xi3 xi4 xi5 xi6 (1)
| {z } | {z }
1≤i1 <i2 <i3 <i4 ≤7 1≤i1 <i2 <i3 <i4 <i5 <i6 ≤7

Chứng minh.
Ta có bất đăng thức

x5 x6 x7 (1 − x1 )(1 − x2 )(1 − x3 )(1 − x4 ) ≥ 0

⇔ x5 x6 x7 (1 − (x1 + x2 + x3 + x4 ) + (x1 x2 + x1 x3 + x1 x4 + x2 x3 + x2 x4 + x3 x4 )−

−(x1 x2 x3 + x1 x2 x3 + x1 x3 x4 + x2 x3 x4 ) + x1 x2 x3 x4 ) ≥ 0
7
Y 
⇔6 xk + 6x5 x6 x7 x1 x2 + x1 x3 + x1 x4 + x2 x3 + x2 x4 + x3 x4 + 6x5 x6 x7 >
k=1
 
≥ 6x5 x6 x7 x1 + x2 + x3 + x4 + 6x5 x6 x7 x1 x2 x3 + x1 x2 x4 + x1 x3 x4 + x2 x3 x4 (2)

Ta có
 X 
6x5 x6 x7 x1 x2 x3 x4 = 6 xi1 xi2 xi3 xi4 ˘6 x5 x6 + x5 x7 + x6 x7


x1 x2 + x1 x3 + x1 x4 + x2 x3 + x2 x4 + x3 x4 ˘
 
−6 x5 + x6 + x7 x1 x2 x3 + x1 x2 x4 + x1 x3 x4 + x2 x3 x4 ˘6x1 x2 x3 x4 (3)

Ta có

6x5 x6 x7 x1 x2 x3 + x1 x2 x4 + x1 x3 x4 + x2 x3 x4 =
X 
=6 xi1 xi2 xi3 xi4 xi5 xi6 ˘6 x5 x6 + x5 x7 + x6 x7 x1 x2 x3 x4 (4)
16i1 <i2 <i3 <i4 <i5 <i6 <7

171
Kết hợp (2) (3) (4) ta thu được

7
Y 
6 xk + 6x5 x6 x7 x1 x2 + x1 x3 + x1 x4 + x2 x3 + x2 x4 + x3 x4 +
k=1
 
+6 x5 + x6 + x7 x1 x2 x3 + x1 x2 x4 + x1 x3 x4 + x2 x3 x4 ˘6x1 x2 x3 x4 + 6x5 x6 x7 ≥
X X
> xi1 xi2 xi3 xi4 + xi1 xi2 xi3 xi4 xi5 xi6
16i1 <i2 <i3 <i4 <7 16i1 <i2 <i3 <i4 <i5 <i6 <7

Bổ đề 2: Giả sử xi > 0(i = 1, 2, 3, 4, 5, 6, 7), ta có bất đẳng thức


18 x65 + x66 + x67 ) + 12 x61 + x62 + x63 + x64 + 42 > 6 x5 x6 x7 + x5 x6 + x5 x7 +
 
x6 x7 x1 x2 + x1 x3 + x1 x4 + x2 x3 + x2 x4 + x3 x4 (5)

Chứng minh:

Ta có:
x65 + x66 + x67 + x61 + x62 + 1 > 6x5 x6 x7 x1 x2
x65 + x66 + x67 + x61 + x63 + 1 > 6x5 x6 x7 x1 x3
x65 + x66 + x67 + x61 + x64 + 1 > 6x5 x6 x7 x1 x4
x65 + x66 + x67 + x62 + x63 + 1 > 6x5 x6 x7 x2 x3
x65 + x66 + x67 + x62 + x64 + 1 > 6x5 x6 x7 x2 x4
x65 + x66 + x67 + x63 + x64 + 1 > 6x5 x6 x7 x3 x4

Cộng các bất đẳng thức, ta được:


6 6 6 6 6 6 6

6 x5 + x6 + x7 ) + 3 x1 + x2 + x3 + x4 + 6 > 6x5 x6 x7 x1 x2 + x1 x3 + x1 x4 + x2 x3 +

x2 x4 + x3 x4 (6)

172
Tương tự, ta thu được:
 
12 x65 +x66 +x67 )+9 x61 +x62 +x63 +x64 +36 > 6 x5 x6 x7 +x5 x6 +x5 x7 +x6 x7 x1 x2 +

x1 x3 + x1 x4 + x2 x3 + x2 x4 + x3 x4 (7)

Cộng (5) (6) ta được


 
18 x65 +x66 +x67 )+12 x61 +x62 +x63 +x64 +42 > 6 x5 x6 x7 +x5 x6 +x5 x7 +x6 x7 x1 x2 +

x1 x3 + x1 x4 + x2 x3 + x2 x4 + x3 x4 (5)

Bổ đề 3: giả sử xi > 0 (i= 1,¯7 ), ta có bất đẳng thức:


 
4 x65 + x66 + x67 ) + 9 x61 + x62 + x63 + x64 + 24 > 6 x5 + x6 + x7 x1 x2 x3 + x1 x2 x4 +

x1 x3 x4 + x2 x3 x4 (8)

Chứng minh:
Ta có:
x65 + x61 + x62 + x63 + 1 + 1 > 6x5 x1 x2 x3
x65 + x61 + x62 + x64 + 1 + 1 > 6x5 x1 x2 x4
x65 + x61 + x63 + x64 + 1 + 1 > 6x5 x1 x3 x4
x65 + x62 + x63 + x64 + 1 + 1 > 6x5 x2 x3 x4

Cộng các bất đẳng thức, thu được:


 
4x65 + 3 x61 + x62 + x63 + x64 + 8 > x1 x2 x3 + x1 x2 x4 + x1 x3 x4 + x2 x3 x4 (9)

Tương tự, ta có:


 
4x65 + 3 x61 + x62 + x63 + x64 + 8 > x1 x2 x3 + x1 x2 x4 + x1 x3 x4 + x2 x3 x4 (10)
 
4x65 + 3 x61 + x62 + x63 + x64 + 8 > x1 x2 x3 + x1 x2 x4 + x1 x3 x4 + x2 x3 x4 (11)

Cộng các bất đẳng thức (9) (10) (11) ta thu được bất đẳng thức cần chứng

173
minh.

Bổ đề 4: giả sử xi > 0(i = 1,¯7 ), ta có bất đẳng thức:


 
3 x65 +x66 +x67 )+4 x61 +x62 +x63 +x64 +5 > 6 x5 x6 x7 +x5 x6 +x5 x7 +x6 x7 x1 x2 x3 x4 +
6x1 x2 x3 x4 + 6x5 x6 x7 (12)

Chứng minh:
Ta có:
x65 + x66 + x61 + x62 + x63 + x64 > 6x5 x6 x1 x2 x3 x4
x65 + x67 + x61 + x62 + x63 + x64 > 6x5 x7 x1 x2 x3 x4
x66 + x67 + x61 + x62 + x63 + x64 > 6x6 x7 x1 x2 x3 x4
1 + 1 + x61 + x62 + x63 + x64 > 6x1 x2 x3 x4
x65 + x66 + x67 + 1 + 1 + 1 > 6x5 x6 x7

Cộng tất cả các bất đẳng thức, ta được


 
3 x65 +x66 +x67 )+4 x61 +x62 +x63 +x64 +5 > 6 x5 x6 x7 +x5 x6 +x5 x7 +x6 x7 x1 x2 x3 x4 +
6x1 x2 x3 x4 + 6x5 x6 x7
áp dụng các kết quả của các bổ đề 1, 2, 3, 4 ta thu được kết quả sau:

Định lý 2. giả sử xi > 0(i = 1, 7 ), ta có bất đẳng thức:

7
Y 7
Y X
6 xk + 25 x6k > xi1 xi2 xi3 xi4
k=1 k=1 16i1 <i2 <i3 <i4 <7
X
+ xi1 xi2 xi3 xi4 xi5 xi6 .
16i1 <i2 <i3 <i4 <i5 <i6 <7

Chứng minh.
Trong 7 số (x1 − 1), (x2 − 1), . . . , (x7 − 1) tồn tại ít nhất 4 số cùng dấu. Do vai

174
trò của các biến như nhau trong bất đẳng thức ta giả thiết rằng

x5 x6 x7 (x4 − 1)(x3 − 1)(x2 − 1)(x1 − 1) ≥ 0

Cộng các bất đẳng thức (1), (5), (8), (12) trong các bổ đề 1, 2, 3, 4 chúng ta
thu được bất đẳng thức cần chứng minh.

Hệ quả 1. Với xi (i = 1, n là những số thực dương, ta có bất đẳng thức


n n
X X 1
4(n − 1) x2n−1
k + (2n − 1)Πnk=1 x2k +1≥ 2(2n − 1)Πnk=1 x2k .
xk
k=1 k=1

Chứng minh.
áp dụng kết quả của định lý 1 cho các biến y1 , y2 , . . . , y2n > 0 ta thu được
2n n
X X 1
(2n − 2) yk2n−1 + (2n − 1)Π2n
k=1 yk +1≥ (2n − 1)Π2n
k=1 yk .
yk
k=1 k=1

Đặt y1 = y2 = x1 , y3 = y4 = x2 , . . . , y2n−1 = y2n = xn ta thu được


n n
X X 1
4(n − 1) x2n−1
k + (2n − 1)Πnk=1 x2k +1≥ 2(2n − 1)Πnk=1 x2k . (đpcm).
xk
k=1 k=1

Hệ quả 2. Với xi (i = 1, n) là những số thực dương thỏa mãn điều kiện


Πnk=1 xk = 1, chứng minh rằng
Pn n−1
Pn 1
1) (n − 2) k=1 xk + n ≥ (n − 1) k=1 .
xk

Pn 2n−1
Pn 1
2) 4(n − 1) k=1 xk + 2n ≥ 2(2n − 1) k=1 .
xk

Chứng minh.
Sử dụng kết quả của định lý 1 và hệ quả 1.

175
II. Một số áp dụng.
áp dụng định lý 1 trường hợp n = 4 ta thu được bất đẳng thức.

Bài toán 3.1. . Với a, b, c, d là những số thực dương, chứng minh rằng
1 1 1 1
2(a3 + b3 + c3 + d3 ) + 3abcd + 1 ≥ 3abcd( + + + )
a b c d

áp dụng hệ quả 1 trường hợp n = 4 ta thu được.

Bài toán 3.2. . Với a, b, c, d > 0, chứng minh rằng


1 1 1 1
12(a7 + b7 + c7 + d7 ) + 7a2 b2 c2 d2 + 1 ≥ 14a2 b2 c2 d2 ( + + + )
a b c d

áp dụng định lý 1 trong trường hợp n = 3 ta thu được.

a+b+c
Bài toán 3.3. . Với p = , a, b, c là độ dài các cạnh của tam giác, r, R
2
là độ dài bán kính các đường tròn nội, ngoại tiếp, chứng minh rằng

4r2 + 8Rr(2 − p) ≤ 1.

Chứng minh.
áp dụng định lý 1 trường hợp n = 3 ta thu được bất đẳng thức

(a2 + b2 + c2 ) + 2abc + 1 ≥ 2(ab + bc + ca)

⇔ (a + b + c)2 + 2abc + 1 ≥ 4(ab + bc + ca)

176
Với a, b, c là 3 cạnh của tam giác và sử dụng đẳng thức

ab + bc + ca = p2 + r2 + 4Rr

ta thu được
4p2 + 8Rrp + 1 ≥ 4(p2 + r2 + 4Rr)

4r2 + 8Rr(2 − p) ≤ 1 (đpcm).

Kỹ năng 4. Phương pháp bất biến.

Bài toán được giải nhờ một tính chất, một đại lượng không thay đổi sau
những phép biến đổi.

Bài số 1. Giả sử a1 , a2 , . . . , an là một hoán vị của các số 1, 2, . . . , n. Chứng


minh rằng nếu n là số lẻ thì P = (a1 − 1)(a2 − 2) . . . (an − n) là số chẵn.

Giải.

Giả sử phần chứng minh P là số lẻ, suy ra tất cả các thừa số ai − i là số


lẻ, suy ra S = ni=1 (ai − i) (n lẻ) là tổng của một số lẻ các số lẻ ⇒ S lẻ. Mặt
P
P P
khác S = ai − i = 0 (mâu thuẫn) với P phải là số chẵn.

Bài số 2. Có tồn tại hay không một dãy vô hạn bình phương của các số
nguyên dương và lập thành cấp số cộng.

Giải. Ta có
a21 < a22 < a23 < · · · < a2n < . . .

177
Do dãy lập thành cấp số cộng ta có

a23 − a22 = a22 − a21 ⇔ (a3 − a2 )(a3 + a2 ) = (a2 − a1 )(a2 + a1 )

Vì a3 + a2 > a1 + a2 ⇒ a3 − a2 < a2 − a1

a2 − a1 > a3 − a2 > a4 − a3 > a5 − a4 > . . .

Điều này không xảy ra đối với một dãy giảm thực sự các số nguyên dương.

Bài số 3. Xung quanh một đường tròn ta viết 50 số theo thứ tự 1, 0, 1, 0, 0, . . . , 0.


Cho phép tăng hai số viết liền nhau lên 1 đơn vị. Có thể nhận được dãy tất
cả các số bằng nhau sau một số hữu hạn hay không?

Giải. Xét tổng

I = (x1 − x2 ) + (x3 − x4 ) + (x5 − x6 ) + · · · + (x49 − x50 )

Giá trị của I không thay đổi khi ta thực hiện các phép biến đổi nêu trên. Với
cách viết ban đầu ta có I = 2, nếu nhận 50 số bằng nhau ta phải có I = 0.
Vậy không thể nhận được dãy các số bằng nhau sau một số hữu hạn bước.

Kỹ năng 5. Sử dụng các bất đẳng thức hay các tính chất đại số.

Bài số 4. Với mọi n ≥ 6, chứng minh rằng trong tập n phần tử bất kỳ
X ⊆ {1, 2, . . . , 2n} luôn tồn tại hai số phân biệt mà bội số chung của chúng
nhỏ hơn hoặc bằng 3n + 6.

Giải. *) Nếu tập n phần tử X ⊆ {1, 2, . . . , 2n} có chứa a ≤ n khi đó ta


xét hai trường hợp 2a ∈ X và 2a ∈
/ X.

178
*) Nếu 2a ∈ X : Khi đó 2a là bội số (2a, a) và 2a ≤ 2n < 3n + 6 ⇒ bài toán thỏa
mãn.
Vậy nếu tồn tại a ∈ X mà 2a ∈ X thì bài toán thỏa mãn.

*) Nếu 2a ∈
/ X ta xóa a và thay thế bởi 2a nhận được tập X1 = (X −{a})∪{2a}.
Lập luận tương tự cho đến khi nhận được 2 trường hợp:

+) Bài toán thỏa mãn


+) Tất cả các phần tử của X là các phần tử a ≤ 2n mà 2a > 2n ⇒ a ≥ n + 1.
Vậy tập
X = {n + 1, n + 2, n + 3, . . . , 2n}.

Ta có
n n n n
2([ ] + 1) = 2( − { } + 1) = n + 2 − 2{ } > n + 2 − 2 = n ≥ n + 1
2 2 2 2
n n n n
3([ ] + 1) = 3( − { } + 1) ≤ 3( + 1) ≤ 2n (Với n ≥ 6)
2 2 2 2
Suy ra
n n
2([ ] + 1) ∈ X, 3([ ] + 1) ∈ X
2 2
Bội số chung bằng
n n
6([ ] + 1) ≤ 6( + 1) = 3N + 6 (đpcm).
2 2
Bài số 5. Với n ≥ 2 nguyên dương cho trước xét tập X = {x1 , x2 , . . . , x1 } ⊆ R.
Kí hiệu P (x) là số các giá trị phân biệt của tổng xi + xj (1 ≤ i < j ≤ n). Tìm
giá trị lớn nhất và nhỏ nhất của P (x).

Giải. Giả sử x1 < x2 < x3 < · · · < xn ta có

x1 + x2 < x1 + x3 < · · · < x1 + xn < x2 + xn < · · · < xn−1 + xn

179
Dãy gồm 2n − 3 phần tử phân biệt ⇒ P (x) ≥ 2n − 3
Bây giờ ta chỉ ta tập X1 mà P (X1 ) = 2n + 1.
Ta xét X1 = {1, 2, 3, 4, . . . , n} và thu được

1 + 2 < 1 + 3 < ··· < 1 + n < 2 + n < 3 + n < ··· < n − 1 + n

⇔ 3 < 4 < 5 < · · · < 2n − 1 (gồm 2n − 3 phần tử phân biệt).

n(n − 1) n(n − 1)
*) Vì số các tổng xi + xj bằng Cn2 = ⇒ P (x) ≤
2 2
n(n − 1)
Bây giờ ta xây dựng tập X2 sao cho P (X2 ) = .
2
Ta xét
X2 = {1, 2, 22 , . . . , 2n−1 }

Giả sử
2i + 2j = 2r + 2s (i < j, r < s)

Suy ra
2i (1 + 2j−i ) = 2r (1 + 2s−r ) ⇒ 2i |2r , 2r |2i

(Vì 2i và 1 + 2s−r , 2r và 1 + 2j−i là hai số nguyên tố cùng nhau) ⇒ i = r ⇒


1 + 2j−i = 1 + 2r−s ⇒ j = s ⇒ Mọi tổng 2i + 2j không trùng với 2r + 2s nếu
n(n − 1)
(i, j 6= (r, s)). Vậy số tổng chính là số các giá trị phân biệt bằng Cn2 =
2
(đpcm).

4 Kỹ năng xây dựng bài toán tổ hợp


Xây dựng một bài toán tổ hợp hay thực sự là rất khó. Thông thường đối
với một bài toán đại số, số học sau khi giải xong chúng ta có thể dễ dàng xây

180
dựng được bài toán mới tương tự hoặc khó hơn khi ta kết hợp với các kết quả
khác. Nhưng đối với bài toán tổ hợp thì công việc này không hề đơn giản.
Chính vì lý do này mà tác giả trình bày các bài toán hay từ các tài liệu nước
ngoài theo cách xây dựng để chia sẻ kinh nghiệm với các bạn đọc.
I. Phương pháp chung.
Để xây dựng một bài toán tổ hợp người ta phải kết hợp 2 kết quả
*) Một kết quả của giải tích, số học, hình học
*) Một kết quả của lý thuyết tổ hợp
Chính vì vậy việc giải một bài toán tổ hợp là rất khó vì chúng ta phải biết 2
kết quả ở 2 lĩnh vực khác nhau và kết hợp chúng. Sau đây chúng ta xét một
số bài toán minh họa.
Ta có kết quả số học: "Với n > 4 là hợp số ta luôn có (n − 1)! chia hết cho n"
Chứng minh
Giả sử p là ước lớn nhất của n (p > 1) ta xét 2 trường hợp sau:
*) Trường hợp1: np 6= p Khi đó np và p là 2 ước số của (n-1)! np 6 n − 1, p 6

 n
n − 1 Suy ra (n-1)! chia hết cho · p = n (đpcm)
p
n

*) Trường hợp 2: p = p ⇒ n = p2
Vì n>4 ⇒ p > 2 ⇒ 2p < p2 = n ⇒ p, 2p là ước của (n-1)! ⇒ 2p2 là ước của
(n-1)! ⇒ là ước của (n-1)!
Từ kết quả số học trên chúng ta xây dựng được bài toán sau:

Bài Toán 1
Với n > 4 là hợp số có tồn tại hay không một hoán vị a1 , a2 , · · · , an của các
số 1, 2, 3, . . . , n sao cho a1 , a1 a2 , a1 a2 a3 , · · · , a1 a2 · · · an có phần dư khác nhau
khi chia cho n? Xét trường hợp n = 4
Bài giải
*) Trả lời : không tồn tại

181
Giả sử tồn tại hoán vị a1 , a2 , · · · , an thỏa mãn yêu cầu của bài toán. Ta có
a1 a2 · · · an = n! ≡ 0 (mod n)

Ta phải có an = n vì nếu ai = n thì a1 a2 · · · ai ≡ 0 (mod n)
Suy ra a1 a2 · · · an−1 = (n − 1)! 6≡ 0 (mod n)
Mặt khác theo kết quả số học ta có
(mâu thuẫn)
*) n = 4 bài toán thỏa mãn với hoán vị sau 1, 2, 3, 4
1 ≡ 1(mod4), 1 · 3 ≡ 3(mod4), 1 · 3 · 2 ≡ 2 (mod 4), 1 · 3 · 2 · 4 ≡ 0 (mod 4) .
 Q
Dãy số x1 , x2 , . . . , xn gọi là có tính chất ξ nếu như xi | 1 + k6=i xk


Ta có bài toán: giả sử dãy x1 , x2 , x3 , · · · , xn có tính chất ξ , chứng minh
rằng dãy
n
Y
x1 , x2 , · · · , xn , 1 + xk
k=1

cũng có tính chất ξ

x1 ·x2 ···xn
Chứng minh. Ký hiệu P(i) = xi ,P = x1 · x2 · · · xn ,
xn+1 = 1 + P

Ta có xn+1 | 1 + x1 x2 · · · xn = 1 + P (hiển nhiên theo cách chọn xn+1 )


2
Ta chứng minh xi | 1+ x1 ·xx2i···xn = 1+ P (1+P
xi
)
(với i = 1, . . . , n) ⇔ xi | 1+P (i)+ Pxi .
Vì xi | 1 + P (i) (giả thiết)
P
xi | P = x1 x2 · · · xn ⇒ x2i | P 2 ⇒ xi | xi
2
Vậy xi | 1 + P (i) + Pxi (2)

Từ kết quả trên ta xây dựng được bài toán sau.

182
Bài toán 4.1. . Với n > 2 , chứng minh rằng ta có thể chọn được n số nguyên
thỏa mãn 1 < a1 < a2 < · · · < an sao cho với b1 , b2 , · · · , bn là một hoán vị bất
kỳ của nó, ta luôn có b1 | 1 + b2 b3 · · · bn
Giải. Lời giải của bài toán tổ hợp này chính là dãy số nguyên dương được
xây dựng từ bài toán trên

Tuy nhiên những điều chúng ta trình bày như trên là quá lý thuyết. Điều
chúng ta cần biết là trong đại số, số học, hình học cần chọn các kết quả gì và
kết quả đó thường được kết hợp với nội dung tổ hợp nào. Nghĩ như vậy thì
chúng ta phát hiện được ngay một số sự kết hợp sau đây.

II. Nguyên lý Diricle với phân hoạch của một tập hợp


"Xét tập 1, 2, · · · , 2n từ một tập con n + 1 phần tử luôn tồn tại ít nhất 2
số có tổng bằng 2n + 1"
   
Ta chia tập 1, 2, · · · , 2n thành n cặp 1, 2n , 2, 2n − 1 ,. . . . . . , n, n + 1
⇒ có ít nhất một cặp chứa 2 phần tử x,y của tập con n+1 phần tử thỏa mãn
x+y = 2n+1
Từ kết quả trên ta xây dựng được bài toán sau.

Bài toán 4.2. . Giả sử với mỗi n > 1, xét 2n+1 số nguyên dương phân biệt
có tổng nhỏ hơn (n+1)(3n+1). Chứng minh rằng trong 2n+1 số này có 2 số
mà tổng của nó bằng 2n+1
Giải. Ký hiệu 2n+1 số nguyên đó là x1 < x2 < · · · < x2n+1

183
*) giả sử xn+1 6 2n thì các số nguyên x1 , x2 , · · · , xn+1 thuộc một trong các tập
sau 1,2n, 2,2n-1,. . . , n,n+1
Suy ra có tồn tại 2 số xi , yi thuộc cùng một tập (theo nguyên lý Diricle) và
xi + yi = 2n + 1 (đpcm).
*) giả sử xn+1 > 2n + 1, khi đó
 
S = x1 + x2 + · · · + xn + xn+1 + · · · + x2n+1

> (1 + 2 + · · · + n) + (2n + 1 + 2n + 2 + · · · + (2n + n + 1)
n(n+1 (5n+2)(n+1)
> 2 + 2 = (n + 1)(3n + 1) (mâu thuẫn)
⇒ xn+1 6 2n ⇒ Bài toán được thỏa mãn.

Từ cách giải của bài toán này chúng ta xây dựng cách xây dựng bài toán
mới như sau:
1. Tìm một phân hoạch có tính (P) nào đó của tập các số nguyên dương

1, 2, 3, . . . ,n thành m tập con
2. Xét một dãy số nguyên dương bất kỳ
a1 < a2 < · · · < am+1 với am+1 6 n
Khi đó theo nguyên lý Diricle sẽ tồn tại ai , aj vì cùng thuộc một tập con
3. Xét trường hợp am+1 > n + 1 và loại bỏ trường hợp này bởi 1 điều kiện bất
đẳng thức

Ta xây dựng một số bài toán mới sau.

Bài toán 4.3. . Xét 3n+1 số nguyên dương phân biệt có tổng nhỏ hơn
11n2 +5n+2
2 (n > 2). Chứng minh rằng trong dãy có tồn tại 3 số phân biệt có
tổng chia hết cho 3

184
Giải. Giả sử 3n+1 số nguyên đó là x1 < x2 < · · · < x3n+1

*) Giả sử x2n+1 6 3n, ta chia tập 1, 2, · · · , 3n bởi phân hoạch sau gồm n
  
tập: 1, 1 + n, 1 + 2n , 2, 2 + n, 2 + 2n , · · · , n, 2n, 3n
Mỗi tập con của phân hoạch gồm 3 số phân biệt có tổng chia hết cho 3. Ta
có 2n+1 số x1 < x2 < · · · < x2n+1 suy ra phải có ít nhất 1 tập chứa 3 số của
dãy .
Giả sử là x, y, z và ta có x + y + z ≡ 0 (mod 3)
*) Giả sử x2n+1 > 3n + 1 ta có:
 
S = x1 + x2 + . . . + x2n + (x2n+1 + x2n+2 + . . . + x3n+1
⇒ S > (1 + 2 + 3 + . . . + 2n) + (3n + 1 + 3n + 2 + . . . + 4n + 1)
2n(2n+1) (n+1)(n+2) 11n2 +5n+2
S> 2 + 3n2 + 2 = 2 (mâu thuẫn)

Để câu hỏi của bài toán đa dạng hơn, ta xét đồng thời 2 phân hoạch cho
cùng 1 tập hợp như sau.

Bài toán 4.4. : Chứng minh rằng với mọi n chúng ta có thể chọn ra 3 số phân

biệt từ tập con 2n+1 phần tử bất kỳ của tập Kn = x ∈ Z | 1−2n 6 x 6 2n−1
mà tổng 3 số đó bằng 0.

Bài giải:
Ta phân hoạch Kn như sau:
  
1, 2n − 2 , 2, 2n − 3 , · · · , n − 1, n (loại I)
  
− 1, −2n + 2 , − 2, −2n + 3 , · · · , − n˘1, +n (loại II)
 
2n − 1 , − 2n + 1 − gồm 2n tập ko chứa số 0.


*) Nếu 0 ∈ X (tập con gồm 2n + 1 phần tử) ⇒ X | 0 gồm 2n phần tử
nên chứa 2 số trái dấu x, −x ⇒ (x − x + 0 = 0). Bởi vì nếu ko chứa cặp số trái

185

dấu thì X | 0 chỉ có 2n − 1 phần tử.
*) Xét trường hợp 0 ∈
/ X ⇒ X gồm 2n + 1 phần tử

⇒ X | 2n − 1, 2n gồm 2n − 1 phần tử
Xét 2 phần tử thuộc một tập loại I hoặc loại II

+/ Giả sử x, y ∈ X thuộc tập loại I, x + y = 2n − 1.



Ta có x, y, −(x + y) thỏa mãn nếu −2n + 1 ∈ X
(x, y, x + y) thỏa mãn nếu 2n − 1 ∈ X

+/ Với x, y ∈ X thuộc tập loại II ta lý luận tương tự


*) Vậy chỉ còn trường hợp − 2n + 1, 0, 2n − 1 ∈
/X

Ta lại lập luận tương tự đối với tập − (2n − 2), · · · , −1, 0, 1, · · · , 2n − 2 .

Bài toán 4.5. . Với n số nguyên > 1, hãy tìm số nguyên K nhỏ nhất 3 6 k 6

2n + 1 sao cho từ tập con X gồm K phần tử thuộc 1, 2, . . . , 2n + 1 bất kỳ
luôn chọn ra được 3 số x, y, z phân biệt sao cho x + y = z .
Giải.

*) Ta chọn X = 1, 3, 5, · · · , 2n + 1 gồm n + 1 số lẻ nên ko thể có đẳng thức
x + y = z . Suy ra K > n + 2
*) Giả sử n+2 phần tử của X được xếp theo thứ tự 1 6 x1 < · · · < xn+2 6 2n+1

*) Xét (2n + 2) số x2 ˘x1 , x3 ˘x1 , · · · , xn+2 ˘x1 , x1 , x2 , · · · , xn+2 đều thuộc 1, 2, 3,

· · · , 2n + 1 suy ra theo nguyên lý Diricle sẽ phải có 2 số bằng nhau:
xk ˘x1 = xj ⇔ xj + x1 = xk (đpcm).

Bài toán 4.6. . Giả sử có một cách chia (phân hoạch) của tập các số 1, 2,

. . . , 100 thành 7 lớp. Chứng minh rằng có tồn tại ít nhất 1 lớp có chứa 4
số phân biệt a, b, c, d mà a + b = c + d, hoặc có chứa 3 số phân biệt e, f, g mà

186
e + f = 2g
Bài giải. Vì 14 · 7 = 98 < 100 nên có ít nhất 1 lớp có chứa ít nhất 15 phần tử.
Ký hiệu T là lớp có chứa ít nhất 15 phần tử đó. Xét a, b ∈ T, a − b > 0
2 = 105 hiệu như vậy. Vì hiệu a − b nhận 1 trong các giá trị 1, 2,
Suy ra sẽ có C15
3, . . . , 99 nên theo nguyên lý Diricle sẽ có 2 cặp x, y và u, v mà x−y = u−v > 0
Nếu x, y, u, v là 4 số phân biệt ta có:
x + v = y + u (bài toán thỏa mãn)
Nếu x = v ⇒ 2x = y + u (bài toán thỏa mãn)
y = u ⇒ 2u = x + v (bài toán thỏa mãn) .

Bài toán 4.7. Với mọi n>1, chứng minh rằng từ tập con A gồm (n+2) phần

tử của tập 1, 2, · · · , 3n người ta có thể chọn ra 2 số x, y sao cho n < x−y < 2n.

Giải. Hiệu 2 phần tử của A ko thay đổi nếu ta tăng tất cả các phần tử
của A lên cùng một số. Vậy chúng ta có thể giả sử rằng phần tử lớn nhất của

187
Một số dạng bất phương trình hàm

Trịnh Đào Chiến


Trường CĐSP Gia Lai

Trong các đề thi chọn học sinh giỏi toán các cấp và trong các kỳ Olympic
toán quốc tế, thỉnh thoảng xuất hiện một số bài toán về giải các bất phương
trình hàm. Đây thường là những dạng toán khó, mà tài liệu lý thuyết về vấn
đề này rất hiếm hoi, ngoại trừ cuốn sách chuyên khảo [1] của GS. Nguyễn
Văn Mậu. Với một góc nhìn khác, bài viết này tiếp tục đề cập đến một số
dạng bất phương trình hàm cơ bản với cặp biến tự do và bất phương trình
hàm dạng cộng (nhân) tính.

1 Bất phương trình hàm với cặp biến tự do.


Xét hàm biến số thực f thỏa mãn các tính chất sau

f (x + y) ≥ f (x) .f (y) . (1)

Người ta đã tìm được hàm f thỏa mãn tính chất (1) nếu f thỏa mãn thêm
một số điều kiện ban đầu nào đó, chẳng hạn (xem [1])

f (x) ≥ ax , a > 0.

Trong bài viết này, hàm ax trong bất đẳng thức trên có thể được thay bởi
một số hàm khác, trong đó có những hàm dạng tổng quát hơn. Cụ thể, hàm

188
f thỏa mãn tính chất (1) và điều kiện sau

f (x) ≥ 1 + x. (2)

Ta sẽ thấy rằng, chỉ cần điều kiện (2) thì bất phương trình cơ bản (1) nêu
trên đã giải được trên một khoảng mở chứa 0 xác định.
Định lý 1.1. Giả sử U là khoảng mở chứa 0 và f : U → R thỏa mãn (1)
và (2) với mọi x, y ∈ U sao cho x + y ∈ U. Thế thì f (x) = ex , x ∈ U.
Chứng minh. Bởi (1), ta có
x x
 x
2
f (x) = f + ≥f ≥ 0, x ∈ U.
2 2 2
Nếu f (x0 ) = 0, thì
x x0
 x 
0 0
0 = f (x0 ) = f + ≥ f2 .
2 2 2
x0 x0
 
Do đó f 2 = 0. Quy nạp, ta có f 2n = 0 với mỗi số nguyên dương n.
Tuy nhiên, từ (2) suy ra rằng rằng f (x) > 0 với mọi x ∈ U và x gần 0. Do
đó điều trên là mâu thuẫn. Vậy

f (x) > 0, x ∈ U.

Tiếp theo, từ (1) và (2), ta sẽ thấy rằng f khả vi tại mỗi điểm x ∈ U và
f 0 (x) = f (x) .
Thật vậy, từ (1) và (2), với h > 0 đủ nhỏ, ta có

f (x + h) − f (x) ≥ f (x) .f (h) − f (x) = (f (h) − 1) .f (x) ≥ h.f (x) .

Do đó
f (x + h) − f (x)
≥ f (x) . (3)
h
Mặt khác, cũng từ (1) và (2), với h > 0 đủ nhỏ, ta có

f (x) = f (x + h − h) ≥ f (x + h) .f (−h) ≥ f (x + h) . (1 − h) .

189
Suy ra
(1 − h) .f (x) + h.f (x) ≥ (1 − h) .f (x + h) .

Do đó
h.f (x) ≥ (1 − h) . (f (x + h) − f (x)) ,

hay
f (x + h) − f (x) f (x)
≤ . (4)
h 1−h
Từ (3) và (4), với h > 0 đủ nhỏ, ta có
f (x + h) − f (x) f (x)
f (x) ≤ ≤ .
h 1−h
Tương tự, bất đẳng thức trên cũng đúng đối với chiều ngược lại, với h < 0
đủ nhỏ. Do đó, ta có
f (x + h) − f (x)
f 0 (x) = lim
h→0 h
tồn tại và bằng f (x), với x ∈ U . Nhận xét trên được sáng tỏ.
Với nhận xét trên, ta có
0
f 0 (x) − f (x)

f (x)
= = 0, x ∈ U.
ex ex

Do đó
f (x) = C.ex ( C là hằng số).
Hơn nữa, từ (1) ta có f (0) ≥ f 2 (0) hay f (0) ≤ 1 và từ (2) ta có f (0) ≥ 1.
Do đó C = f (0) = 1. Thử lại, hàm f (x) = ex thỏa mãn các điều kiện (1)
và (2). Định lý được chứng minh.
Bây giờ, ta có thể tổng quát hóa Định lý 1.1 trên, bằng cách làm “yếu” đi
điều kiện (2). Chẳng hạn, ta có kết quả tổng quát hơn như sau
Định lý 1.2. Giả sử U là khoảng mở chứa 0 và f : U → R thỏa mãn điều
kiện (1) với mọi x, y ∈ U sao cho x + y ∈ U. Nếu f bị chặn dưới bởi một hàm g
khả vi tại 0 và thỏa mãn g (0) = 1, thì f (x) = ekx , x ∈ U , trong đó k = f 0 (0) .

190
Từ các kết quả trên, ta có
Hệ quả 1.1. Giả sử U là khoảng mở chứa 0 và f : U → R thỏa mãn điều
kiện (1) với mọi x, y ∈ U sao cho x + y ∈ U. Nếu f (0) = 1 và f khả vi tại 0, thì
f (x) = ekx , x ∈ U.
Hệ quả 1.2. Giả sử F là hàm xác định trên khoảng mở U chứa 0 và thỏa
mãn
F (x + y) ≤ F (x) + F (y) . (5)

với mọi x, y ∈ U sao cho x + y ∈ U. Nếu F bị chặn trên bởi một hàm G khả
vi tại 0 và thỏa mãn G (0) = 0 (và, nói riêng, nếu F khả vi tại 0 và thỏa mãn
F (0) = 0), thì f (x) = kx, x ∈ U , trong đó k là một hằng số.
Chứng minh. Áp dụng Định lý 1.2, với f (x) = e−F (x) và g (x) = e−G(x) , ta
có được điều phải chứng minh.
Bất phương trình hàm (5) chính là dạng tương ứng với phương trình hàm
Cauchy quen thuộc: f (x + y) = f (x) + f (y) .

2 Bất phương trình hàm dạng cộng tính và nhân


tính.

Phần này đề cập đến việc giải các hệ bất phương trình hàm và tương ứng
là hệ phương trình hàm, với các dạng sau đây
- Dạng “cộng”: f (a + x) ≤ α + f (x), f (b + x) ≤ β + f (x), x ∈ R,
- Dạng “cộng - nhân”: f (a + x) ≤ αf (x), f (b + x) ≤ βf (x), x ∈ R,
- Dạng “nhân - cộng”: f (ax) ≤ α + f (x), f (bx) ≤ β + f (x), x ∈ I , I ⊂ R,
- Dạng “nhân”: f (ax) ≤ αf (x), f (bx) ≤ βf (x), x ∈ I , I ⊂ R, trong đó
a, b, α, β là các số thực cho trước.

191
Chú ý rằng, nếu α = f (a), β = f (b), thì hệ bất phương trình hàm dạng
“cộng” trên là sự thu hẹp của bất phương trình hàm Cauchy cổ điển

f (x + y) ≤ f (x) + f (y) , x, y ∈ R.

Trước hết, ta nhắc lại rằng, một tập hợp M trù mật trong tập số thực R
nếu như trong mọi lân cận của một điểm tùy ý của tập R đều có ít nhất một
điểm của tập M. Chẳng hạn, tập Q các số hữu tỷ là tập trù mật trong tập R.
Tính chất sau đây là một kết quả quen thuộc (Định lý Kronecker), có thể
tìm thấy chứng minh ở các tài liệu lý thuyết cơ bản:
“Nếu a và b là các số thực không thông ước với nhau, thì tập A = {ma + nb; m, n ∈ Z}
trù mật trong R”.
Hơn nữa, các kết quả sau cũng đã được chứng minh
Bổ đề 2.1. Giả sử a, b ∈ R và a < 0 < b là các số cho trước. Ký hiệu

A = {ma + nb; m, n ∈ N} .

1) Nếu ab ∈
/ Q, thì tập A trù mật trong R.
2) Nếu ab ∈ Q, thì tồn tại d > 0 sao cho A = {kd; k ∈ Z} .
Dạng “nhân” của bổ đề này như sau
Bổ đề 2.2. Giả sử a, b ∈ R và 0 < a < 1 < b là các số cho trước. Ký hiệu

M = {am bn ; m, n ∈ N} .

log b
1) Nếu log a ∈
/ Q, thì tập M trù mật trong (0, ∞) .
log b 
2) Nếu log a ∈ Q, thì tồn tại d > 0 sao cho M = dk ; k ∈ Z .
Bây giờ, ta chứng minh các định lý sau đây
Định lý 2.1. (Dạng “cộng”)
Giả sử a, b, α, β là các số thực cho trước thỏa mãn
α β
a < 0 < b, = ,
a b

192
và giả sử rằng hàm f : R → R liên tục tại ít nhất một điểm.
1) Nếu ab ∈
/ Q, thì f thỏa mãn hệ bất đẳng thức hàm

f (a + x) ≤ α + f (x) , f (b + x) ≤ β + f (x) , x ∈ R. (6)

khi và chỉ khi


f (x) = px + f (0) , x ∈ R,

trong đó p := αa .
2) Nếu ab ∈ Q, thì tồn tại duy nhất một nghiệm hàm liên tục f : R → R
của hệ phương trình hàm tương ứng

f (a + x) ≤ α + f (x) , f (b + x) ≤ β + f (x) , x ∈ R, (7)



sao cho f [0,d] = f0 , trong đó d := min {ma + nb > 0; m, n ∈ N} tồn tại, là số
dương, và f0 : [0, d] → R là hàm liên tục cho trước thỏa mãn điều kiện
α
f0 (d) = d + f0 (0) .
a

Hơn nữa, nếu f0 là đơn điệu nghiêm ngặt thì nó trùng với hàm f trên đoạn
[0, d].
Chứng minh.
1) Từ (6), dễ dàng suy ra

f (ma + x) ≤ mα + f (x) , f (nb + x) ≤ nβ + f (x) , m, n ∈ N, x ∈ R.

Trong bất đẳng thức đầu tiên ở trên, thay x bởi nb + x, ta có

f (ma + nb + x) ≤ mα + f (nb + x) ≤ mα + nβ + f (x) .

Do đó

f (ma + nb + x) ≤ mα + nβ + f (x) , m, n ∈ N, x ∈ R.

193
Đặt
α β
p := = ,
a b
ta có thể viết bất đẳng thức này dưới dạng

f (t + x) ≤ pt + f (x) , t ∈ A, x ∈ R, (8)

trong đó, theo Bổ đề 2.1, tập

A = {ma + nb; m, n ∈ N}

trù mật trong R.


Giả sử rằng x0 là điểm mà tại đó hàm f liên tục và x là một giá trị thực
tùy ý. Bởi tính chất trù mật của A trong tập R, tồn tại một dãy (tn ) sao cho

tn ∈ A (n ∈ N) , lim tn = x0 − x.
n→+∞

Từ bất đẳng thức (8), ta có

f (tn + x) ≤ ptn + f (x) , n ∈ N.

Cho n → ∞, bởi tính liên tục của hàm f tại x0 , ta thu được

f (x0 ) ≤ p (x0 − x) + f (x) , x ∈ R.

Bây giờ, để chứng minh phần đảo, thay x bởi x − t trong (8), ta được

f (x) ≤ pt + f (x − t) , t ∈ A, x ∈ R.

Chọn một điểm x ∈ R cố định tùy ý và, bởi tính trù mật của A trong R,
một dãy (tn ) sao cho

tn ∈ A (n ∈ N) , lim tn = x − x0 .
n→+∞

194
Thế thì, ta có
f (x) ≤ ptn + f (x − tn ) , n ∈ N.

Cho n → ∞, bởi tính liên tục của hàm f tại x0 , ta thu được bất đẳng thức

f (x) ≤ p (x − x0 ) + f (x0 ) , x ∈ R.

Do đó, ta có
f (x) = px + (f (x0 ) − px0 ) , x ∈ R,

trong đó f (0) = f (x0 ) − px0 .


Ta có điều phải chứng minh.
2) Từ (7), dễ dàng suy ra
α
f (ma + nb + x) = (ma + nb) + f (x) , m, n ∈ N, x ∈ R. (9)
a
Theo Bổ đề 2.1, phần 2), số d := min {ma + nb > 0; m, n ∈ N} là xác định
và là số dương. Hơn nữa

{ma + nb > 0; m, n ∈ N} = {kd; k ∈ Z} .

Do đó, (9) có dạng


α
f (kd + x) = kd + f (x) , k ∈ Z, x ∈ R.
a
Dễ dàng thấy rằng hệ phương trình hàm này tương đương với phương
trình
α
f (d + x) = d + f (x) , x ∈ R.
a
Bây giờ, ta xác định f1 : (d, 2d] → R bởi công thức
α
f1 (x) := d + f0 (x − d) , x ∈ (d, 2d] .
a
Giả sử rằng
fn : (nd, (n + 1) d] → R, (n ∈ N)

195
đã được xác định. Thế thì, ta xác định

fn+1 : ((n + 1) d, (n + 2) d] → R

bởi hệ thức truy hồi


α
fn+1 (x) := d + fn (x − d) , x ∈ ((n + 1) d, (n + 2) d] , n ∈ N.
a

Tương tự, giả sử


α
f−1 (x) := − d + f0 (x + d) , x ∈ [−d, 0) .
a

Giả sử rằng ta có định nghĩa

f−n (x) := [−nd, (−n + 1) d) → R (n ∈ N) .

Thế thì, ta định nghĩa


α
f−(n+1) (x) := − d + f−n (x + d) , x ∈ [− (n + 1) d, −nd) , n ∈ N.
a

Dễ dàng kiểm tra được rằng f : R → R, xác định bởi



 f−n (x), khi x ∈ [−nd, (−n + 1)d),

f (x) = f0 (x), khi x ∈ [0, d], , n ∈ N,

fn (x), khi x ∈ (nd, (n + 1)d],


thỏa mãn hệ (7), là hàm liên tục và f [0,d] = f0 .
Định lý được chứng minh hoàn toàn.
Định lý 2.2. (Dạng “cộng-nhân”)
Giả sử a, b ∈ R và α, β > 0 là các số cho trước thỏa mãn
log α log β
a < 0 < b, = ,
a b

và giả sử rằng hàm f : R → R liên tục tại ít nhất một điểm.

196
1) Nếu ab ∈
/ Q, thì f thỏa mãn hệ bất đẳng thức hàm

f (a + x) ≤ αf (x) , f (b + x) ≤ βf (x) , x ∈ R, (10)

khi và chỉ khi


f (x) = f (0) .epx , x ∈ R,
log α
trong đó p := a .
2) Nếu ab ∈ Q, thì tồn tại duy nhất một nghiệm hàm liên tục f : R → R
của hệ phương trình hàm tương ứng

f (a + x) = αf (x) , f (b + x) = βf (x) , x ∈ R, (11)



sao cho f [0,d] = f0 , trong đó d := min {ma + nb > 0; m, n ∈ N} tồn tại, là số
dương, và f0 : [0, d] → R là hàm liên tục cho trước thỏa mãn điều kiện
log α
d
f0 (d) = f0 (0) .e a ,

Hơn nữa, nếu f0 là đơn điệu nghiêm ngặt thì nó trùng với hàm f trên đoạn
[0, d].
Chứng minh.
Tương tự như chứng minh Định lý 2.1.
1) Từ (10), dễ dàng suy ra

f (ma + x) ≤ αm .f (x) , f (nb + x) ≤ β n .f (x) , m, n ∈ N, x ∈ R.

Trong bất đẳng thức đầu tiên ở trên, thay x bởi nb + x, ta có

f (ma + nb + x) ≤ αm .f (nb + x) ≤ αm β n .f (x) .

Do đó
f (ma + nb + x) ≤ αm β n .f (x) , m, n ∈ N, x ∈ R.

197
Đặt
log α log β
p := = ,
a b
ta có thể viết bất đẳng thức này dưới dạng

f (t + x) ≤ ept .f (x) , t ∈ A, x ∈ R, (12)

trong đó, theo Bổ đề 2.1, tập

A = {ma + nb; m, n ∈ N}

trù mật trong R.


Giả sử rằng x0 là điểm mà tại đó hàm f liên tục và x là một giá trị thực
tùy ý. Bởi tính chất trù mật của A trong tập R, tồn tại một dãy (tn ) sao cho

tn ∈ A (n ∈ N) , lim tn = x0 − x.
n→+∞

Từ bất đẳng thức (12), ta có

f (tn + x) ≤ eptn .f (x) , n ∈ N.

Cho n → ∞, bởi tính liên tục của hàm f tại x0 , ta thu được

f (x0 ) ≤ ep(x0 −x) .f (x) , x ∈ R.

Bây giờ, để chứng minh phần đảo, thay x bởi x − t trong (12), ta được

f (x) ≤ ept .f (x − t) , t ∈ A, x ∈ R.

Chọn một điểm x ∈ R cố định tùy ý và, bởi tính trù mật của A trong R,
một dãy (tn ) sao cho

tn ∈ A (n ∈ N) , lim tn = x − x0 .
n→+∞

198
Thế thì, ta có
f (x) ≤ eptn .f (x − tn ) , n ∈ N.

Cho n → ∞, bởi tính liên tục của hàm f tại x0 , ta thu được bất đẳng thức

f (x) ≤ ep(x−x0 ) .f (x0 ) , x ∈ R.

Do đó, ta có
f (x) = ep(x) . (f (x0 ) − px0 ) , x ∈ R.

Ta có điều phải chứng minh.


2) Chứng minh tương tự chứng minh phần 2) của Định lý 2.1.
Định lý được chứng minh hoàn toàn.
Định lý 2.3. (Dạng “nhân-cộng”)
Giả sử a, b, α, β là các số thực cho trước thỏa mãn
α β
0 < a < 1 < b, = ,
log a log b

và giả sử rằng hàm f : I → R liên tục tại ít nhất một điểm.


log b
1) Nếu log a ∈
/ Q, thì f thỏa mãn hệ bất đẳng thức hàm

f (ax) ≤ α + f (x) , f (bx) ≤ β + f (x) , x ∈ I, (13)

thì
i) Trường hợp I = (0, ∞) :

f (x) = p log x + f (1) , x > 0,

ii) Trường hợp I = (−∞, 0) :

f (x) = p log (−x) + f (−1) , x < 0,

α
trong đó p := log a .

199
log b
2) Nếu log a ∈ Q, thì tồn tại duy nhất một nghiệm hàm liên tục f : I → R
(I = (0, ∞) hoặc I = (−∞, 0)) của hệ phương trình hàm tương ứng

f (ax) = α + f (x) , f (bx) = β + f (x) , x ∈ I, (14)



sao cho f [1,d] = f0 , trong đó d := min {am bn > 1; m, n ∈ N} tồn tại, lớn hơn 1,
và f0 : [1, d] → R là hàm liên tục cho trước thỏa mãn điều kiện
α
f0 (d) = . log d + f0 (1) .
log a

Hơn nữa, nếu f0 là đơn điệu nghiêm ngặt thì nó trùng với hàm f trên đoạn
[1, d].
Chứng minh.
1) i) Giả sử rằng I = (0, ∞).
Từ (13), chứng minh tương tự như các phần trên, ta có

f (am bn x) ≤ mα + nβ + f (x) , m, n ∈ N, x > 0.

Đặt
α β
p := = ,
log a log b
ta có thể viết bất đẳng thức này dưới dạng

f (am bn x) ≤ p log (am bn ) + f (x) , m, n ∈ N, x > 0,

hay
f (tx) ≤ p log t + f (x) , t ∈ M, x > 0, (15)

trong đó, theo Bổ đề 2.2, tập

M = {am bn ; m, n ∈ N}

trù mật trong I.

200
Giả sử rằng x0 > 0 là điểm mà tại đó hàm f liên tục và x > 0 là một giá
trị tùy ý. Bởi tính chất trù mật của M trong tập I , tồn tại một dãy (tn ) sao
cho
x0
tn ∈ M (n ∈ N) , lim tn = .
n→+∞ x
Từ bất đẳng thức (15), ta có

f (tn x) ≤ p log tn + f (x) , n ∈ N.

Cho n → ∞, bởi tính liên tục của hàm f tại x0 , ta thu được
x0
f (x0 ) ≤ p log + f (x) , x > 0.
x
x
Bây giờ, để chứng minh phần đảo, thay x bởi t trong (15), và chọn một
dãy (tn ) sao cho
x
tn ∈ M (n ∈ N) , lim tn = .
n→+∞ x0
Thế thì, ta có
x
f (x) ≤ p log + f (x0 ) , x > 0.
x0
Do đó, ta có

f (x) = f (x0 ) − p log x0 + p log x, x > 0.

Phần i) được chứng minh.


ii) Giả sử rằng I = (−∞, 0) .
Ta xét hàm g : (0, ∞) → R xác định bởi công thức g (x) = f (−x) , x < 0,
thỏa mãn hệ (13) và chứng minh tương tự như chứng minh phần i).
2) Phần này chứng minh tương tự như chứng minh Định lý 2.1, phần 2.
Hệ quả 2.1. Giả sử a, b, α, β ∈ R thỏa mãn các giả thiết của Định lý 2.3,
phần 1).
Nếu hàm f : [(−∞, 0) ∪ (0, ∞)] → R thỏa mãn hệ bất đẳng thức (13) và trong

201
mỗi khoảng (−∞, 0) và (0, ∞) tồn tại ít nhất một điểm mà tại đó hàm f liên
tục, thì (
p log x + f (1) , khi x ∈ (0, ∞) ,
f (x) =
p log (−x) + f (−1) , khi x ∈ (−∞, 0) ,
trong đó p := logα a .
Chú ý 2.1. Giả sử a, b, α, β là các số thực cho trước thỏa mãn 0 < a < 1 < b
α β
và log a = log b . Nếu 0 ∈ I , thì không tồn tại hàm nào thỏa mãn hệ (13).
Thật vậy, trong bất đẳng thức (13) nếu đặt x = 0, thì 0 ≤ α, 0 ≤ β , mâu
thuẫn với giả thiết αβ < 0.
Định lý 2.4.(Dạng “nhân”)
Giả sử a, b, α, β là các số thực cho trước thỏa mãn
log α log β
a < 1 < b, = ,
log a log b
và giả sử rằng hàm f : I → R liên tục tại ít nhất một điểm.
log b
1) Nếu log a ∈
/ Q, thì f thỏa mãn hệ bất đẳng thức hàm

f (ax) ≤ αf (x) , f (bx) ≤ βf (x) , x ∈ I, (16)

thì
i) Trường hợp I = (0, ∞) :

f (x) = f (1) .xp , x > 0,

ii) Trường hợp I = (−∞, 0) :

f (x) = f (−1) . (−x)p , x < 0,

trong đó p := logα a .
log b
2) Nếu log a ∈ Q, thì tồn tại duy nhất một nghiệm hàm liên tục f : I → R
(I = (0, ∞) hoặc I = (−∞, 0)) của hệ phương trình hàm tương ứng

f (ax) = αf (x) , f (bx) = βf (x) , x ∈ I, (17)

202

sao cho f [1,d] = f0 , trong đó d := min {am bn > 1; m, n ∈ N} tồn tại, lớn hơn 1,
và f0 : [1, d] → R là hàm liên tục cho trước thỏa mãn điều kiện
log α
f0 (d) = f0 (1) .d log a .

Hơn nữa, nếu f0 là đơn điệu nghiêm ngặt thì nó trùng với hàm f trên đoạn
[1, d].
Chứng minh.
1) i) Giả sử rằng I = (0, ∞) .
Từ (16), chứng minh tương tự như các phần trên, ta có

f (am bn x) ≤ αm β n .f (x) , m, n ∈ N, x > 0.

Đặt
log α log β
p := = ,
log a log b
ta có thể viết bất đẳng thức này dưới dạng

f (am bn x) ≤ (am bn )p .f (x) , m, n ∈ N, x > 0,

hay
f (tx) ≤ tp .f (x) , t ∈ M, x > 0, (18)

trong đó, theo Bổ đề 2.2, tập

M = {am bn ; m, n ∈ N}

trù mật trong I.


Giả sử rằng x0 > 0 là điểm mà tại đó hàm f liên tục và x > 0 là một giá
trị tùy ý. Bởi tính chất trù mật của M trong tập I , tồn tại một dãy (tn ) sao
cho
x0
tn ∈ M (n ∈ N) , lim tn = .
n→+∞ x

203
Từ bất đẳng thức (18), ta có

f (tn x) ≤ tpn .f (x) , n ∈ N.

Cho n → ∞, bởi tính liên tục của hàm f tại x0 , ta thu được
 x p
0
f (x0 ) ≤ .f (x) , x > 0.
x
x
Bây giờ, để chứng minh phần đảo, thay x bởi t trong (18), và chọn một
dãy (tn ) sao cho
x
tn ∈ M (n ∈ N) , lim tn = .
n→+∞ x0
Thế thì, ta có  p
x
f (x) ≤ .f (x0 ) , x > 0.
x0
Phần i) được chứng minh.
ii) Giả sử rằng I = (−∞, 0) .
Ta xét hàm g : (0, ∞) → R xác định bởi công thức g (x) = f (−x), x < 0,
thỏa mãn hệ (16) và chứng minh tương tự như chứng minh phần i).
2) Phần này chứng minh tương tự như chứng minh Định lý 2.1, phần 2.
Chú ý 2.2. Giả sử a, b, α, β là các số thực cho trước thỏa mãn 0 < a < 1 < b
log α log β
và log a = log b . Nếu I = R hoặc I = [0, ∞) hoặc I = (−∞, 0] và f : I → R thỏa
mãn hệ (16), thì f (0) = 0.
Thật vậy, bởi một trong hai giả thiết α < 1 < β hoặc β < 1 < α và, hơn
nữa, f (0) . (1 − α) ≤ 0 và f (0) . (1 − β) ≤ 0, ta suy ra f (0) = 0.
Từ Chú ý này, ta có

Chú ý 2.3. i) Giả sử f : [0, ∞) → R thỏa mãn hệ (16). Nếu f (0,∞) và
a, b, α, β thỏa mãn tất cả các giả thiết của Định lý 2.4, phần 1), thì
(
f (1) .xp , khi x ∈ (0, ∞) ,
f (x) =
0, khi x = 0,

204
log α
trong đó p := log a .

ii) Giả sử f : (−∞, 0] → R thỏa mãn hệ (16). Nếu f (−∞,0) và a, b, α, β thỏa
mãn tất cả các giả thiết của Định lý 2.4, phần 1), thì
(
f (−1) . (−x)p , khi x ∈ (−∞, 0) ,
f (x) =
0, khi x = 0,

log α
trong đó p := log a .
Hệ quả 2.2. Giả sử a, b, α, β ∈ R thỏa mãn các giả thiết của Định lý 2.4,
phần 1).
i) Nếu hàm f : [(−∞, 0) ∪ (0, ∞)] → R thỏa mãn hệ bất đẳng thức (16) và
trong mỗi khoảng (−∞, 0) và (0, ∞) tồn tại ít nhất một điểm mà tại đó hàm
f liên tục, thì
(
f (1) .xp , khi x ∈ (0, ∞) ,
f (x) = p
f (−1) . (−x) , khi x ∈ (−∞, 0) ,

log α
trong đó p := log a .
ii) Nếu hàm f : R → R thỏa mãn hệ bất đẳng thức (16) và trong mỗi
khoảng (−∞, 0) và (0, ∞) tồn tại ít nhất một điểm mà tại đó hàm f liên tục,
thì 
p khi x ∈ (0, ∞) ,
 f (1) .x ,

f (x) = 0, khi x = 0,
 p
f (−1) . (−x) , khi x ∈ (−∞, 0) ,

log α
trong đó p := log a .
Chú ý 2.4. Ta luôn có các định lý tương tự như các Định lý 2.1 - 2.4,
với hàm f thỏa mãn các bất đẳng thức có dấu ngược lại.
T.Đ.C

205
Tài liệu tham khảo
[1]. Nguyễn văn Mậu, “Bất đẳng thức, định lí và áp dụng”, Nhà xuất bản
Giáo dục, 2006.
[2]. Th. M. Rassias, “Functional equations, inequalities and applications”,
73 - 89, Kluwer Academic Publishers, 2003.
[3]. PI. Kannappan, “Functional equations and with applications”, 617 -
636, Springer Monographs in Mathematics, 2009.

206
Phương trình Pell

Phạm Văn Quốc


Trường Đại học KHTN, ĐHQGHN

1. Phương trình Pell


Định nghĩa. Phương trình Pell là phương trình Diophant có dạng

x2 − dy 2 = 1, x, y ∈ Z, (1)

trong đó d là số nguyên dương không chính phương cố định.


Chú ý : i) Nếu d là số chính phương thì phương trình (1) không có nghiệm
nguyên dương,
ii) Phương trình (1) luôn có nghiệm (1, 0) .
Giải phương trình (1):
Ký hiệu
h√ i n √ o
Z d = x + y d|x, y ∈ Z .

Với mỗi số z = x + y d, ta ký hiệu

z̄ = x − y d

N (z) = z z̄ = x2 − dy 2 ∈ Z.

Ta dễ dàng kiểm tra lại đẳng thức sau:

N (z1 z2 ) = N (z1 ) N (z2 ) và z1 z2 = z̄1 z̄2 .

207

Khi đó phương trình (1) có thể viết dưới dạng N (z) = 1 với z = x + y d ∈
√ 
Z d . Rõ ràng (1) luôn có nghiệm x = 1, y = 0 tương ứng với z = 1, hơn nữa
nếu z là một nghiệm của (1) thì −z cũng là nghiệm. Do đó ta chỉ quan tâm
đến các nghiệm z > 0 và z 6= 1.
Định lý 1. Nếu z0 > 1 là nghiệm nhỏ nhất của N (z) = 1 thì tất cả các nghiệm
nguyên dương của phương trình này được cho bởi z = z0n , n ∈ Z.
Chứng minh. Giả sử số z thỏa mãn N (z) = 1, z > 0, khi đó tồn tại duy nhất số
nguyên k sao cho z0k ≤ z < z0k+1 . Xét số z1 = zz0−k = z z̄0k thỏa mãn 1 ≤ z1 < z0
và N (z1 ) = N (z) N (z0 )−k = N (z) = 1. Mà z0 là nghiệm lớn hơn 1 nhỏ nhất
nên z1 = 1, tức là z = z0k . Suy ra điều phải chứng minh. 
Hệ quả. Nếu (x0 , y0 ) là nghiệm nhỏ nhất (hay còn gọi là nghiệm cơ bản) của

(1) thì tất cả các nghiệm nguyên dương (x, y) của nó được cho bởi x + y d =
√ n
x0 + y0 d , n ∈ N. Tức là

1 n 1
x= (z0 + z̄0n ) , y = √ (z0n − z̄0n ) .
2 2 d

Hay viết cách khác ở dạng dãy truy hồi, n ≥ 0

xn+2 = 2x0 xn+1 − xn , x−1 = 1

yn+2 = 2y0 yn+1 − yn , y−1 = 0.

Từ hệ quả trên, để tìm tất cả các nghiệm của (1) ta sẽ chứng minh nó
luôn có nghiệm (x, y) khác (1, 0). để làm được điều này ta cần một số kết quả
sau.
Bổ đề (định lý Dirichlet). Giả sử α là số vô tỷ và n là số nguyên dương, khi
đó luôn tồn tại số nguyên p và số q ∈ {1, 2, ..., n} sao cho

p
α − < 1
.
q (n + 1) q

208
1
Chứng minh. Bất đẳng thức tương đương với |qα − p| < . Xét n + 2 số
n+1
0, {α} , {2α} , ..., {nα} , 1

nằm trong đoạn đoạn [0, 1] . Khi đ ó nếu có số {kα} = 0 thì chọn q = k, p = kα,
1
trái lại sẽ tồn tại hai số, giả sử {kα} và {hα} mà |{kα} − {hα}| < lúc đó
n+1
ta chọn q = |k − h| , p là số nguyên gần qα nhất. 
Hệ quả. Nếu α là số thực tùy ý, khi đó tồn tại vô hạn các cặp số nguyên
dương (p, q) thỏa mãn
α − < 1 .
p
q 2 q
Định lý 2. Phương trình Pell (1) có vô số nghiệm nguyên dương.

Chứng minh. Từ hệ quả Bổ đề Dirichlet với α = d tồn tại vô số cặp số
nguyên dương (x, y) sao cho

x2 − dy 2 < 2 d + 1.

Suy ra tồn tại số nguyên n, |n| < 2 d + 1 sao cho phương trình x2 − dy 2 = n
có vô số nghiệm nguyên dương (x, y). Nên tồn tại hai cặp nghiệm phân biệt

(x1 , y1 ) , (x2 , y2 ) sao cho x1 ≡ y1 (modn) , x2 ≡ y2 (modn). đặt z1 = x1 +y1 d, z2 =
√ √ 
x2 + y2 d. Giả sử z1 > z2 thì z0 = z1 /z2 > 1, z0 ∈ Z d , N (z0 ) = 1 là nghiệm
của phương trình Pell. 
Ví dụ: Tìm tất cả các nghiệm nguyên dương của phương trình x2 − 2y 2 = 1.
Dễ kiểm tra thấy nghiệm nhỏ nhất của phương trình là (3, 2), nên tất cả
các nghiệm (xn , yn ) của nó là
√ n √ n
3+ 2 + 3+ 2
xn = ,
√ n 2 √ n
3+ 2 − 3+ 2
yn = √ , n ∈ N.
2 2

209
2. Phương trình "kiểu Pell"
Phương trình có dạng
x2 − dy 2 = a (2)

với a là số nguyên còn gọi là phương trình "kiểu Pell".


√ √ 
Phương trình này có thể viết dưới dạng N (z) = a với z = x+y d ∈ Z d .

Nhận xét: i) Nếu d, a < 0 thì (2) vô nghiệm,


ii) Nếu d < 0, a > 0 thì (2) chỉ có thể có hữu hạn nghiệm.
a) Trường hợp a = −1
x2 − dy 2 = −1 (3)

Định lý 3. Phương trình x2 − dy 2 = −1 có nghiệm nguyên khi và chỉ khi tồn


√ 
tại z1 ∈ Z d sao cho z12 = z0 .
Chứng minh. Giả sử phương trình có nghiệm, ta xét nghiệm nhỏ nhất z =
√ 
z1 ∈ Z d của phương trình N (z) = −1 với z > 1. Suy ra 1 ≤ z1 < z0 . Mà
z = z12 là nghiệm của phương trình N (z) = 1 nên z12 = z0 . 

Định lý 4. Giả sử (3) có nghiệm nguyên dương (x0 , y0 ) nhỏ nhất. Nghiệm
tổng quát (xn , yn ) của nó có thể viết dưới dạng

xn = x0 un + dy0 vn ,

yn = y0 un + x0 vn ,

với (un , vn ) là nghiệm tổng quát của phương trình Pell u2 − dv 2 = 1.


 √ 
Chú ý. Ta có xn = yn d . Thật vậy:
 √  √ 
x2n − dyn2 = −1 ⇒ yn d + xn yn d − xn = 1,

√ √
mà yn d + xn > 1 nên 0 < yn d − xn < 1.

210
Định lý 5. Giả sử d = p là số nguyên tố. Phương trình (3) có nghiệm khi và
chỉ khi p = 2 hoặc p ≡ 1 (mod4) .
Chứng minh. Nếu phương trình có nghiệm suy ra p|x2 + 1 ⇒ p = 2 hoặc
p ≡ 1 (mod4) .
Ngược lại với p = 2 thì x = y = 1 là nghiệm.
Xét trường hợp p = 4t + 1. Giả sử (x0 , y0 ) là nghiệm phương trình Pell
x20 −py02 = 1. Ta có x0 lẻ (trái lại y02 ≡ py02 ≡ 3 (mod4)). Khi đó (x0 − 1) (x0 + 1) =
py02 do gcd (x0 + 1, x0 − 1) = 2 nên một trong chúng có dạng 2x2 và số còn lại
có dạng 2py 2 .
Nếu x0 + 1 = 2x2 , x0 − 1 = 2py 2 ta có nghiệm nhỏ hơn của phương trình Pell
x2 − py 2 = 1 nên không xảy ra. Do đó x0 − 1 = 2x2 , x0 + 1 = 2py 2 ⇒ x2 − py 2 =
−1. 

b) Trường hợp a bất kỳ


Bây giờ ta xét phương trình N (z) = a. Tương tự như trên ta có thể tìm
được tất cả các nghiệm của nó từ nghiệm z mà 1 ≤ z ≤ z0 , với z0 là nghiệm
nhỏ nhất không tầm thường của N (z) = 1. Tức là ta chỉ cần thử một số hữu
hạn các giá trị của x. Hơn nữa ta có nhận xét sau.
Nhận xét: Nếu phương trình N (z) = a có nghiệm nguyên z1 thì nó có nghiệm
z với
z0 + 1 p
|x| ≤
√ |a|.
2 z0
√ √
Thật vậy, tồn tại m ∈ Z sao cho a/ z0 ≤ z0m z1 < a z0 . Khi đó z2 = z0m z1 =

x + y d là nghiệm của N (z) = 1 và

a z0 + 1 p
2 |x| = z2 + ≤ √ |a|.
z2 z0

Ví dụ: Tìm tất cả các nghiệm nguyên của phương trình x2 − 7y 2 = 2.

211
√ √
Dễ thấy z0 = 8 + 3 7 là nghiệm củaqN (z) = 1. Giả sử z = x + y 7 thỏa

|a| = 3 và y = x 7−2 ≤ 1. Khi đó dễ thấy z = 3 + 7.
2
p
mãn N (z) = 2, x ≤ 2z0√+1
z0
Vậy tất cả các cặp nghiệm (x, y) xác định bởi
√ √  √ n
x+y 7=± 3+ 7 8+3 7 , n ∈ N+ .

c) Trường hợp tổng quát hơn (1)


Bây giờ ta xét phương trình dạng

ax2 − by 2 = 1, (4)

trong đó a, b ∈ N+ .
Mệnh đề. Nếu ab = k 2 với k là số nguyên lớn hơn 1, thì phương trình (4)
không có nghiệm nguyên.
Chứng minh. Giả sử (4) có nghiệm (x, y), dễ thấy gcd (a, b) = 1 nên theo giả
thiết ta có a = k12 , b = k22 . Khi đó k12 x2 − k22 y 2 = 1 nhưng điều này không xảy
ra vì k1 x + k2 y > 1. 
Xét phương trình
u2 − abv 2 = 1. (5)

Định lý 6. Giả sử (4) có nghiệm nguyên dương và (x0 , y0 ) là nghiệm nhỏ


nhất. Khi đó nghiệm tổng quát (xn , yn ) của (5) là

xn = x0 un + by0 vn ,

yn = x0 un + ay0 vn

trong đó (un , vn )n≥0 là nghiệm của (5).


Chứng minh. Dễ thử thấy (xn , yn ) thỏa mãn (4). Ngược lại nếu (x, y) là nghiệm
của (4) thì (u, v) với u = ax0 x − by0 y, v = y0 x − x0 y là nghiệm của (5). Giải
phương trình với biến x, y ta có ngay công thức trong mệnh đề. 

212
Cụ thể hơn ta có thể viết

1 y0 √ √ n 1 y0 √ √ n
     
xn = x0 + ab u1 + v1 ab + x0 − ab u1 − v1 ab
2 a 2 a
1 x0 √ √ n 1 x0 √ √ n
   
yn = y0 + ab u1 + v1 ab + y0 − ab u1 − v1 ab .
2 b 2 b

Ví dụ: Tìm tất cả các nghiệm nguyên của phương trình 6x2 − 5y 2 = 1.

Giải. Dễ thấy nghiệm nhỏ nhất là (x0 , y0 ) = (1, 1). Xét phương trình u2 −
30v 2 = 1, nghiệm nhỏ nhất là (11, 2) và nghiệm tổng quát của nó là un+1 =
11un + 60vn , vn+1 = 2un + 11vn . Khi đó nghiệm tổng quát của phương trình đã
cho là xn = un + 5vn , yn = un + 6vn .

Ví dụ: Tìm tất cả các số nguyên dương n sao cho 2n + 1, 3n + 1 là số chính


phương.

Giải. Giả sử 2n + 1 = x2 , 3n + 1 = y 2 ⇒ 3x2 − 2y 2 = 1. Nghiệm nhỏ nhất


của nó là (1, 1). Phương trình Pell tương ứng u2 − 6v 2 = 1 có nghiệm nhỏ nhất
(1, 1) nên

1
h √ m √ m i
um = 5+2 6 + 5−2 6
2
1
h √ m √ m i
vm = √ 5+2 6 − 5−2 6 .
2 6

Suy ra xm = um + 2vm , ym = um + 3vm , m ≥ 0. Tức là

2
n = ym − x2m = vm (2um + 5vm ) .

3. Phương trình Pell và phân số liên tục

213
Giả sử a0 , a1 , ..., am là các số thực, a1 , ..., am > 0. Khi đó số
1
a0 +
1
a1 +
1
a2 +
...
1
+
1
am−1 +
am
được gọi là phân số liên tục hữu hạn và ký hiệu là [a0 , a1 , ..., am ]. Nếu ai là số
nguyên thì ta gọi là phân số liên tục đ ơn. Tương tự ta có khái niệm phân số
liên tục vô hạn.
Định lý 7. Giả sử phân số liên tục [a0 , a1 , ..., am ] và 0 ≤ n ≤ m, xét dãy

p−1 = 1, p0 = a0 , pn = an pn−1 + pn−2

q−1 = 0, q0 = 1, qn = an qn−1 + qn−2 .

Khi đó
i) pn qn−1 − pn−1 qn = (−1)n−1 ,
ii) pn qn−2 − pn−2 qn = (−1)n an ,
pn
iii) hội tụ thứ k thỏa mãn [a0 , a1 , ..., an ] = .
qn
định lý này có thể chứng minh dễ dàng bằng quy nạp.
Nhận xét. Mỗi phân số liên tục đơn hữu hạn biểu diễn một số hữu tỷ và ngược
lại mỗi số hữu tỷ có thể biểu diễn dưới dạng một phân số liên tục đơn hữu
hạn.
Định nghĩa. Cho α là số vô tỷ, ta nói α là số vô tỷ bậc hai nếu nó là nghiệm
của đa thức bậc hai hệ số nguyên. Nghiệm còn lại của phương trình được gọi
là liên hợp của α.
Định lý 8. (định lý Lagrange) Một số vô tỷ là vô tỷ bậc hai khi và chỉ khi
phân số liên tục đơn vô hạn của nó là tuần hoàn.

214
Định lý 9. Giả sử d là số nguyên dương không chính phương. Ký hiệu pk /qk

là hội tụ thứ k của phân số liên tục đơn của d và giả sử m là độ dài của chu
kỳ phân số liên tục này. Khi đó
i) Nếu m chẵn thì các nghiệm nguyên dương của phương trình x2 − dy 2 = 1
là x = pjm−1 , y = qjm−1 (j = 1, 2, 3, ...) ; còn phương trình x2 − dy 2 = −1 vô
nghiệm.
ii) Nếu m lẻ thì các nghiệm nguyên dương của phương trình x2 − dy 2 = 1
là x = p2jm−1 , y = q2jm−1 (j = 1, 2, 3, ...) ; còn phương trình x2 − dy 2 = −1 có
nghiệm x = p(2j−1)m−1 , y = p(2j−1)m−1 .
√ 2
Ví dụ: 3 = [1, 1, 2, 1, 2, ...] , nên m = 2 và [1, 1] = . Rõ ràng 22 − 3.12 = 1 nên
1
(2, 1) là nghiệm dương nhỏ nhất của phương trình x2 − 3y 2 = 1.
√ 3
2 = [1, 2, 2, ...] , nên m = 1 và [1, 2] = . Rõ ràng 32 − 3.32 = 1 nên (3, 2) là
1
nghiệm dương nhỏ nhất của phương trình x2 − 2y 2 = 1.

4. Bài tập áp dụng

Bài 1. Chứng minh rằng n2 + (n + 1)2 là số chính phương với vô hạn giá trị
tự nhiên của n.
Giải. Giả sử n2 + (n + 1)2 = m2 suy ra (2n + 1)2 − 2m2 = −1. Xét phương
trình Pell x2 − 2y 2 = −1. Nghiệm nhỏ nhất là (1, 1), khi đó tất cả các nghiệm
khác thỏa mãn
√ √ 2k+1
xk + y k 2 = 1+ 2 ,
1
nk = (xk − 1) ,
2

(chú ý là xk luôn là số lẻ).

Bài 2. Tìm tất cả các số nguyên dương n sao cho n (n + 1) chia hết cho 3 và
thương là số chính phương.

215
n (n + 1)
Giải. đặt = y 2 ⇒ (2n + 1)2 − 12y 2 = 1. Phương trình Pell x2 −
3
12y 2 = 1 có nghiệm nhỏ nhất (7, 2) nên

1
h √ m √ m i
xm = 7 + 2 12 + 7 − 2 12
2
1
h √ m √ m i
ym = √ 7 + 2 12 − 7 − 2 12 .
2 12

đáp số nm = 12 (xm − 1) .
Bài 3. Chứng minh rằng có vô hạn các bộ ba số nguyên dương (a, b, c) có ước
chung lớn nhất là 1 và a2 b2 + b2 c2 + c2 a2 là số chính phương.

Giải. Giả sử a2 b2 + b2 c2 + c2 a2 = d2 suy ra d2 − a2 + b2 c2 = a2 b2 . Chọn
a = b = 1 ta có phương trình d2 − 2c2 = 1. Nghiệm nhỏ nhất của phương trình
Pell này là (3, 2) .
Bài 4. Giả sử a0 = 0, a1 = 4 và an+1 = 18an − an−1 , n ≥ 1. Chứng minh rằng
5a2n + 1 là số chính phương với mọi n.
Giải. Xét phương trình Pell x2 − 5y 2 = 1. Nghiệm nhỏ nhất (9, 4) và nghiệm
√ √ n
tổng quát thỏa mãn xn + yn 5 = 9 + 4 5 , n ≥ 0. Khi đó

xn+1 = 9xn + 20yn ,

yn+1 = 4xn + 9yn .

Suy ra yn+2 = 18yn+1 − yn ⇒ an = yn . Do đó 5a2n + 1 = 5yn2 + 1 = x2n .


Bài 5. Giả sử hai số lập phương đúng liên tiếp có hiệu là n2 , chứng minh
rằng 2n − 1 là số chính phương.
Giải. Giả sử (m + 1)3 − m3 = n2 ⇒ (2n)2 − 3 (2m + 1)2 = 1. Do đó
√ 2k+1 √ 2k+1
4n = 2 + 3 + 2− 3 .

Từ đó suy ra điều phải chứng minh.

216
Bài 6. Chứng minh rằng có vô hạn số nguyên dương n sao cho n + 1 và 3n + 1
đều là các số chính phương. Giả sử tất cả các số này là n1 < n2 < · · · < nk < · · ·
thì nk nk+1 + 1 cũng là số chính phương.
Giải. Nếu n + 1 = x2 , 3n + 1 = y 2 ⇒ 3x2 − y 2 = 2. Hay u2 − 3v 2 = 1 với
1 1
u= 2 (3x − y) , v = 2 (y − x) . Suy ra

2 1
h √ 2k+1 √ 2k+1 i
nk = x2k − 1 = (uk + vk ) − 1 = 2+ 3 + 2− 3 −4 .
6
Bài 7. Chứng minh rằng có vô số số nguyên dương n sao cho n2 + 1 là ước
của n!.
Giải. Xét phương trình x2 − 5y 2 = −1 có nghiệm dương nhỏ nhất (2, 1).
Xét nghiệm dương (x, y) với y > 5. Ta có 5 < y < 2y ≤ x ⇒ 4y 2 < 5y 2 − 1 = x2 .

Suy ra 2 x2 + 1 = 5y.2y là ước của x!.
√ 
Bài 8. Chứng minh rằng có vô hạn các số nguyên dương n sao cho 2n + 1
là số chính phương.
Giải. Xét phương trình x2 − 2y 2 = 1, phương trình này có vô số nghiệm
nguyên dương và nghiệm nhỏ nhất là (3, 2) . Khi đó 2x2 y 2 = x4 − x2 suy ra
2 √
x2 − 1 < 2x2 y 2 < x4 hay x2 − 1 < xy 2 < x2 . Chọn n = xy.
q 
2 2
Bài 9. Đặt an = n + (n + 1) , n ≥ 1. Chứng minh rằng có vô hạn số n

sao cho an − an−1 > 1 và an+1 − an = 1.


Giải. Xét phương trình

n2 + (n + 1)2 = y 2 ⇔ (2n + 1)2 − 2y 2 = −1.

Phương trình này có vô số nghiệm (x, y) nguyên dương với x lẻ. Giả sử x =
2n + 1 ta có an = y và
hp i hp i
an−1 = y2 − 4n , an+1 = y2 + 4n + 4 .

Dễ dàng kiểm tra được tính chất yêu cầu.

217
a2 + 1
Bài 10. Chứng minh rằng nếu các số nguyên dương a, b thỏa mãn +1
b2
là số chính phương thì giá trị của nó là 9.
Giải. để ý rằng
a2 + 1 2 2 2
 2
+ 1 = k ⇒ a − k − 4 b = −1.
b2

Bài 11. Cho a ∈ N+ , d = a2 − 1. Chứng minh rằng nếu các số x, y thỏa mãn
2
x − dy 2 < 2a + 2 thì x2 − dy 2 là số chính phương.
Giải. Nghiệm nhỏ nhất của phương trình x2 − dy 2 = 1 là (a, 1). Do đó nếu
(x, y) là nghiệm của x2 − dy 2 = m thì ta có
r
a+1
x≤ |m| ⇒ x < a + 1
2

Suy ra (x, y) = (a, 1) và m = 1 hoặc m = x2 , y = 0.


Bài 12. Giả sử x, y là hai số nguyên dương sao cho x (y + 1) và y (x + 1) là
các số chính phương. Chứng minh rằng hoặc x hoặc y là số chính phương.
Bài 13. Tìm tất cả các cặp số nguyên (m, n) sao cho mn + m và mn + n là
các số chính phương.
Bài 14. Chứng minh rằng nếu 5x2 + 4 hoặc 5x2 − 4 là số chính phương khi và
chỉ khi x là số hạng của dãy Fibonacci.

Bài 15. Chứng minh rằng nếu m = 2 + 2 28n2 + 1 là số nguyên với n ∈ N,
thì nó là số chính phương.
Bài 16. Chứng minh rằng nếu số nguyên dương n thỏa mãn 3n + 1 và 4n + 1
là số chính phương thì n là bội của 56.
Bài 17. Cho k là số nguyên lớn hơn 2. Chứng minh rằng phương trình

x2 − k 2 − 4 y 2 = −1 có nghiệm khi và chỉ khi k = 3.

218
Bài 18. Chứng minh rằng nếu p là số nguyên tố dạng 4k + 3 thì có đ úng một
trong hai phương trình x2 − py 2 = ±2 có nghiệm nguyên.
Bài 19. Chứng minh rằng 3n −2 là số chính phương khi và chỉ khi n = 1, n = 3.
Bài 20. Chứng minh rằng nghiệm nguyên dương của phương trình 5a − 3b = 2
là a = b = 1.
Bài 21. Cho p là số nguyên tố, chứng minh rằng phương trình x2 − 2y 2 = p
có nghiệm nguyên dương khi và chỉ khi p = 2 hoặc p = ±1 (mod8) .
Bài 22. (HSGQG) Cho hai dãy số {xn } , {yn } , n ∈ N xác định như sau:

x0 = 1, x1 = 4, xn+2 = 3xn+1 − xn

y0 = 1, y1 = 4, yn+2 = 3yn+1 − yn .

a) Chứng minh rằng x2n − 5yn2 + 4 = 0.


b) Giả sử các số nguyên dương a, b thỏa mãn a2 − 5b2 + 4 = 0, chứng minh
rằng tồn tại số tự nhiên k sao cho xk = a, yk = b.
Bài 23. (Canada) Cho x0 = 0, x1 = 1, xn+1 = 4xn − xn−1 , và y0 = 1, y1 =
2, yn+1 = 4yn − yn−1 . Chứng minh rằng với mọi n ≥ 0 ta có yn2 = 3x2n + 1.
Bài 24. (IMO Shortlist) Chứng minh rằng tồn tại hai dãy số nguyên dương
tăng (an ) , (bn ) sao cho an (an + 1) là ước của b2n + 1 với mọi n ≥ 1.
Bài 25. (IMO Shortlist) Xét hệ phương trình
(
x+y =z+u
2xy = zu.
x
Tìm giá trị lớn nhất của hằng số thực m sao cho m ≤ y với mọi nghiệm nguyên
dương (x, y, z, u) của hệ mà x ≥ y.
Bài 26. (IMO Shortlist) Chứng minh rằng tồn tại vô số các số nguyên dương
n sao cho p = nr, trong đó p, r lần lượt là nửa chu vi và bán kính đường tròn
nội tiếp một tam giác với độ dài các cạnh là các số nguyên.

219
Bất biến và nửa bất biến

Lê Anh Vinh
ĐH Giáo dục, ĐHQGHN

1 Khởi động
Chúng ta sẽ bắt đầu bằng một ví dụ đơn giản. Giáo viên yêu cầu học sinh
làm một thí nghiệm nhỏ – viết lên bảng a + b số gồm a số 0 và b số 1. Sau đó
thực hiện a + b − 1 lần phép biến đổi sau: xoá hai số bất kỳ trên bảng. Nếu
chúng bằng nhau thì viết số 0 lên bảng và nếu khác nhau thì viết số 1 lên
bảng. Sau khi học sinh làm thử trên vở, giáo viên có thể nói ngay số còn lại
trên bảng là số 1 hay số 0.
Học sinh sẽ thắc mắc một cách tự nhiên: làm thế nào giáo viên biết được
số còn lại trên bảng? Rõ ràng các phép biến đổi có thể thực hiện theo nhiều
cách khác nhau, nhưng sau các phép biến đổi, tổng các số trên bảng là không
đổi theo modulo 2. Do đó, số còn lại trên bảng sẽ là 1 nếu b lẻ và 0 trong
trường hợp ngược lại.
Chúng ta tiếp tục với ví dụ sau.

Bài toán 1.1. Khối A0 có một ngôn ngữ riêng chỉ gồm hai chữ cái A và 0,
đồng thời thỏa mãn hai điều kiện sau:

• Nếu xóa hai chữ cái kề nhau A0 trong bất kì một từ nào, ta không làm
thay đổi nghĩa của từ đó.

220
• Nếu thêm tổ hợp 0A hoặc AA00 vào vị trí bất kì nào trong một từ, ta
cũng không làm thay đổi nghĩa của từ đó.
Liệu hai từ A00 và 0AA có cùng nghĩa không?
Dễ thấy sau mỗi phép biến đổi, số lượng A và 0 thêm vào hay bớt đi là
như nhau. Vì vậy, xuất phát từ A00 với nhiều chữ cái 0 hơn, ta không thể
thu được 0AA với nhiều chữ cái A hơn. Đại lượng bất biến ở đây có thể chọn
là sai khác giữa số chữ cái A và chữ cái 0 trong một từ.
Lời giải trên đã chỉ ra ý tưởng cơ bản nhất của bất biến. Cho trước một
số cấu hình, chúng ta có thể thực hiện một số phép biến đổi trên chúng. Câu
hỏi đặt ra là có thể thu được một cấu hình này từ một cấu hình khác không?
Tính bất biến thường được dùng để chỉ ra rằng từ một cấu hình không thể
đạt tới một cấu hình khác. Để làm được điều đó, chúng ta xây dựng một đại
lượng không đổi (hoặc thay đổi đơn điệu - khi đó ta có khái niệm nửa bất
biến) dưới các phép bién đổi sao cho giá trị của đại lượng này là khác nhau
ở hai cấu hình trong câu hỏi. Tuy nhiên, đối với các bài toán bất biến, phần
khó nhất thường là chỉ ra đại lượng bất biến. Trong bài giảng này, chúng ta
sẽ hệ thống một số dạng bất biến thường gặp qua một loạt các ví dụ từ đơn
giản đến nâng cao.
Bài toán 1.2. Một hình tròn được chia làm 6 ô dẻ quạt bằng nhau và đặt
một quân tốt vào mỗi ô. Trong mỗi bước, cho phép chuyển hai quân tốt bất
kỳ vào ô kề với nó. Hỏi có thể chuyển tất cả quân cờ vào một ô hay không?

Chứng minh. Đánh số các ô từ 1 đến 6 theo chiều kim đồng hồ. Với mỗi cách
sắp xếp, xét S là tổng các ô có chứa quân cờ (tính cả bội). Khi đó, tính chẵn
lẽ của S không thay đổi.

Trong một số trường hợp, bất biến không chỉ được dùng để chứng minh
ta không thể thu được cấu hình này từ một cấu hình khác mà còn có thể sử

221
dụng để tìm hiểu cấu hình nào có thể thu được từ một cấu hình cho trước.
Ta có ví dụ sau.

Bài toán 1.3. Các số 1, 2, 3, . . . , 20 được viết lên bảng. Mỗi phép biến đổi, ta
xóa hai số a, b và thêm vào số a + b − 1. Số nào sẽ còn lại trên bảng sau 19
bước?

Chứng minh. Với bộ n số trên bảng ta xét đại lượng X bằng tổng các số trên
bảng trừ đi n. Khi đó X không thay đổi trong các phép biến đổi. Lúc đầu
X = (1 + 2 + . . . + 20) − 20 = 190. Sau 19 bước, X = 190 hay số còn lại sẽ là
191.

Sẽ không ngạc nhiên nếu như một số học sinh đưa ra lập luận như sau:
tại mỗi bước, tổng các số giảm đi 1. Lúc đầu tổng là 210 và sau 19 bước, số
còn lại sẽ là 210 - 19 = 191. Cách giải này hiển nhiên đúng nhưng không làm
rõ được ý tưởng của “bất biến”. Chúng ta sẽ đưa cho học sinh một bài toán
tương tự, mà ở đây, những lập luận “rút gọn” như vậy là khó có thể thực hiện
được.

Bài toán 1.4. Các số 1, 2, 3, . . . , 20 được viết lên bảng. Mỗi phép biến đổi, ta
xóa hai số a, b và thêm vào số ab + a + b. Số nào sẽ còn lại trên bảng sau 19
bước?
Sau đây là một số bài toán khá thú vị sử dụng ý tưởng của bất biến.

Bài toán 1.5. Trong bàn cờ 8 × 8, một ô bị tô màu đen và các ô còn lại được
tô màu trắng. Liệu có thể làm cho cả bảng màu trắng bằng cách tô lại các
hàng và cột không? Ở đây, tô lại một hàng hay cột được hiểu như là một
phép đổi màu tất cả các ô trên hàng hoặc cột đó.

Bài toán 1.6. Giải Bài 5 cho bảng 3 × 3.

222
Bài toán 1.7. Giải Bài 5 cho bảng 8 × 8 với bốn ô ở góc được tô màu đen và
các ô khác được tô màu trắng.
Lưu ý rằng Bài 5, khác với Bài 6 và Bài 7, có thể giải chỉ sử dụng tính
chẵn lẻ của số ô đen trên bảng. Để giải Bài 6, ta có thể xét tính chẵn lẻ của
số ô đen trong bốn ô ở góc. Để giải Bài 7, ta phải xét tính chẵn lẻ của số ô
đen trong bốn ô cụ thể, ví dụ bốn ô ở góc phải trên.

Bài toán 1.8. Các số 1, 2, . . . , 2013 được viết lên bảng. Cho phép xóa đi hai
số và thay bởi hiệu của chúng. Liệu có thể thu được một bảng gồm toàn số 0
không?
Có nhiều cách để giải bài toán trên, một trong những bất biến có thể sử
dụng là tính chẵn lẻ của tổng các số viết trên bảng. Lưu ý rằng tổng và hiệu
của hai số bất kỳ là cùng tính chẵn lẻ.

Bài toán 1.9. Có 13 con tắc kè xanh, 15 con tắc kề đỏ và 17 con tắc kè vàng
trên một hòn đảo. Khi hai con tắc kè khác màu gặp nhau, chúng đổi sang
màu còn lại. Liệu có thể đến một lúc nào đó tất cả các con tắc kè có cùng
màu hay không?

Chứng minh. Mỗi “trạng thái” trên đảo gồm a con tắc kè xanh, b con tắc kè
đỏ và c con tắc kè vàng với a+b+c = 45. Phép biến đổi màu sẽ chuyển từ trạng
thái (a, b, c) sang một trong ba trạng thái (a−1, b−1, c+2), (a−1, b+2, c−1) hoặc
(a+2, b−1, c−1). Dễ thấy (a−1)−(b−1) ≡ (a−1)−(b+2) ≡ (a+2)−(b−1) ≡ a−b
mod 3. Bất biến X = sai khác giữa số tắc kè xanh và số tắc kề đỏ theo modulo
3. Lúc đầu X ≡ 2 mod 3 và khi tất cả các tắc kè cùng màu thì X ≡ 0 mod 3.
Vì vậy, trường hợp tất cả các con tắc kè có cùng màu không thể xảy ra.

Bài toán 1.10. Viết 11 số +1 và 01 số −1 lên đỉnh của 12 giác đều. Cho
phép đổi dấu của các số trên k đỉnh bất kỳ của đa giác. Có thể hay không
luôn chuyển số −1 sang đỉnh kề của nó nếu

223
a) k = 3;
b) k = 4;
c) k = 6?

Chứng minh. Câu trả lời là phủ định trong cả ba trường hợp. Chứng minh
cho cả ba trường hợp có thể thực hiện như sau: chúng ta chọn các đỉnh cách
đều nhau đúng k − 1 đỉnh (ví dụ khi k = 3 ta chọn được 4 đỉnh, khi k = 4 ta
chọn được 3 điểm và k = 6 ta chọn được 2 điểm). Bất biến của chúng ta là
tích các số trên các đỉnh được chọn. Chúng ta xếp số −1 vào một trong các
điểm được chọn. Dễ kiểm tra rằng nếu số −1 được chuyển sang đỉnh kề thì
tích các số trên các điểm được chọn lúc đó sẽ là 1.

Lưu ý rằng khái niệm bất biến là khá trừu tượng và phức tạp đối với phần
lớn học sinh trong lần tiếp cận đầu tiên. Chúng ta nên lưu ý phân tích các lập
luận logic của việc sử dụng các đại lượng bất biến trong giải các bài toán cụ
thể. Ở đây, chúng tôi chỉ đưa ra những gợi ý tóm tắt cho các bài toán nhưng
khi hướng dẫn cho học sinh, có thể sử dụng các ví dụ minh họa khiến lời giải
trở nên trực quan và dễ hiểu hơn. Ngoài ra, chúng ta chỉ nên giới thiệu khái
niệm và phương pháp sử dụng bất biến sau khi mỗi học sinh đã tự tìm tòi
và giải quyết độc lập một vài ví dụ minh họa đơn giản nhất, thậm chí đã sử
dụng bất biến mà không hề ý thức được điều đó. Rõ ràng, bước khó nhất khi
giải các bài toán sử dụng bất biến là phát hiện ra được đại lượng bất biến phù
hợp. Đây là một nghệ thuật mà chúng ta chỉ có thể thành thạo được thông
qua việc giải một loạt các bài toán trong cùng một chủ đề.

224
2 Cơ bản
Chúng ta đã gặp một số bất biến trong phần Khởi động. Tiếp theo chúng
ta sẽ xem xét một số dạng bất biến cơ bản khác, ví dụ như tính chẵn lẻ, tô
màu, công thức đại số, cặp nghịch đối của hoán vị,... Đối với mỗi bài toán,
giáo viên có thể bắt đầu bằng việc thảo luận với học sinh dạng bất biến có
thể sử dụng là gì.
Bài toán 2.1. Trên bảng viết các số 1, 2, . . . , 1000. Ở mỗi bước cho phép thay
một số bằng tổng các chữ số của nó. Quá trình dừng lại khi có toàn các số có
một chữ số. Hỏi số số 1 còn lại trên bảng nhiều hơn hay số số 2 còn lại trên
bảng nhiều hơn?

Chứng minh. Nếu chúng ta viết tất cả các số trên bảng theo modulo 9 thì các
số này sẽ là bất biến trong các phép biến đổi. Do số các số đồng dư 1 mod 9
nhiều hơn số các số đồng dư 2 mod 9 trong tập {1, . . . , 1000}, số các số 1 còn
lại trên bảng sẽ nhiều hơn số số 2 còn lại trên bảng.
Bài toán 2.2. Vào năm 3000, ở Việt Nam, một nhân dân tệ (RMB) đổi được
10 đồng Việt Nam (VNĐ). Trong khi đó, ở Trung Quốc, một VNĐ đổi được
10 RMB. Một du khách người Nhật lúc đầu có 01 VNĐ. Ông này có thể đi lại
tùy ý giữa hai nước VN và TQ. Hỏi ông ta có thể làm cho số VNĐ và RMB
ông ta có là bằng nhau hay không?

Chứng minh. Xét X = số VNĐ − số RMB của du khách. Khi đó X mod 11


sẽ là bất biến trong các bước đổi tiền. Nếu số VNĐ và RMB bằng nhau thì
X ≡ 0 mod 11. Lúc đầu X ≡ 1 mod 11, do đó không thể thu được X ≡ 0
mod 11. Ta có câu trả lời phủ định.

Bài toán 2.3. Hình vuông 8 × 8 bỏ đi hai ô ở góc đối nhau. Có thể phủ phần
còn lại bởi 31 quân đômino 1 × 2 không? Nếu bỏ hai ô bất kì thì sao?

225
Chứng minh. Chúng ta tô màu hình vuông đen trắng như bàn cờ vua. Hai ổ
ở góc đối nhau luôn cùng mau nên sau khi bỏ chúng đi, số ô đen khác số ô
trắng. Mỗi quân đômino phủ đúng một ô đen, một ô trắng nên phần còn lại
của hình vuông không thể phủ kín được bởi các quân đômino. Bất biến ở đây
chính là hiệu số giữa số ô trắng và số ô đen trên bảng.

Bài toán 2.4. Cho đa thức P (x) = ax2 + bx + c, có thể thực hiện một trong
hai phép biến đổi:
a) Đổi chỗ a và c.
b) Đổi biến x bởi x + t với t ∈ R.
Hỏi từ x2 − 31x − 3 có thu được x2 − 20x − 12 không? Tìm mối liên hệ của
hai đa thức bậc hai P (x) và Q(x) sao cho từ đa thức này có thể thu được đa
thức kia bởi hai phép biến đổi nói trên.

Chứng minh. Bất biến của chúng ta là định thức ∆ = b2 − 4ac của đa thức
P (x) = ax2 + bx + c. Dễ kiểm tra rằng hai phép biến đổi a) và b) không làm
thay đổi định thức của đa thức. Định thức ∆1 của x2 − 31x − 3 và định thức
∆2 của x2 − 20x − 12 là khác nhau. Ta có câu trả lời phủ định! Chúng tôi để
lại câu hỏi tìm mối liên hệ giữa hai đa thức bậc hai nhận được từ nhau qua
hai phép biến đổi trong đề bài cho bạn đọc.

Bài toán 2.5. Tô đen 09 ô của hình vuông 10 × 10. Mỗi lần tô màu đen một
ô chưa tô nếu nó kề với ít nhất hai ô đen (kề được hiểu là chung cạnh). Có
thể tô màu hết bàn cờ hay không? Nếu là 10 ô thì sao? Nếu là hình vuông
n × n thì lúc đầu cần tô đen ít nhất bao nhiêu ô để có thể tô đen cả bàn cờ?

Chứng minh. Nếu tô 10 ô thì câu trả lời là khẳng định. Ví dụ ta có thể bắt
đầu với 10 ô đen trên đường chéo chính của hình vuông.

226
Nếu tô 9 ô thì câu trả lời là phủ đinh. Xét X là tổng chu vi của phần tô
đen trên hình thì lúc đầu X ≤ 36. Dễ kiểm tra X là nửa bất biến, cụ thể, X
là không tăng. Nếu cả bàn cờ được tô màu thì lúc này X = 40 - mâu thuẫn.
Vậy, không thể tô đen được cả bàn cờ nếu xuất phát với 9 ô màu đen.

Bài toán 2.6. Cho một hoán vị của các số 1, 2, . . . , 2012. Mỗi lần cho đổi chỗ
hai số bất kì. Sau 2011 bước có thể quay về hoán vị ban đầu không?

Chứng minh. Bài toán này liên quan đến số cặp nghịch đối của một hoán vị.
Cặp nghịch đối của hoán vị π của {1, . . . , n} là số cặp 1 ≤ i < j ≤ n sao cho
π(i) > π(j). Bạn đọc hãy tự kiểm tra rằng tính chẵn lẻ của số cặp nghịch
đối thay đổi khi chúng ta hoán vị một cặp trong dãy. Sau 2011 bước, số cặp
nghịch đối sẽ bị thay đổi tính chẵn lẻ và chúng ta không thể quay trở về hoán
vị ban đầu.

Bài toán 2.7. Trên bảng viết các số 1, 2, 3, 4, 5. Mỗi bước cho phép chọn hai
số a, b và thay bởi a + b, ab. Hỏi có thu được 21, 27, 64, 180, 540 hay không?

Chứng minh. Bài toán này thoạt nhìn khá đơn giản nhưng để tìm được bất
biến không phải là điều dễ dàng. Trước hết ta kiểm tra rằng số các số chia
hết cho 3 không giảm và số lượng này tăng khi và chỉ khi từ hai số chia 3 dư
1 và chia 3 dư 2 chúng ta thu được một số chia hết cho 3 và một số chia hết
cho 2. Vì vậy, khi chúng ta lần đầu tiên chuyển sang trạng thái có 4 số chia
hết cho 3 thì số còn lại chia 3 dư 2, nhưng 64 chia 3 dư 1 nên câu trả lời sẽ
là phủ định.

Bài toán 2.8. Trên bảng viết số 99 . . . 99 (2012 lần). Mỗi bước cho phép chọn
một số a, phân tích a thành tích hai số m, n và viết lên bảng m ± 2, n ± 2 tùy
ý. Ví dụ: a = 15, a = 3.5 có thể viết lên bảng 1 = 3 − 2 và 7 = 5 + 2. Hỏi sau
một số bước như vậy, có thể thu được trên bảng toàn các số 9 không?

227
Chứng minh. Đây cũng không phải là một bài toán “dễ” như cách phát biểu
cũng như lời giải của nó. Bất biến là trên bảng luôn có ít nhất một số chia 4
dư 3.

Bài toán 2.9. Một túi gồm 1001 viên đá. Mỗi bước chọn một túi có nhiều
hơn 01 viên. Bỏ đi một viên và chia các viên còn lại thành 02 túi. Hỏi có thể
làm như vậy để thu được tất cả các túi đều có 03 viên?

Chứng minh. Xét X là tổng số đá và số túi tại mỗi bước. Dễ thấy X mod 4
không đổi. Lúc đầu X = 1002 không chia hết cho 4. Nếu tất cả các túi có 3
viên thì X lúc đó chia hết cho 4, mẫu thuẫn. Vậy câu trả lời là phủ định.

Bài toán 2.10. Chúng ta xét một quân cờ đặc biệt, được gọi là quân “lạc
đà”, di chuyển trên bàn cờ 10 × 10 như là một quân mã (1, 3). Có nghĩa là di
chuyển sang ô kề và sau đó di chuyển ba ô theo hướng vuông góc với hướng
vừa di chuyển. Quân mã thông thường di chuyển theo hướng (1, 2). Liệu quân
lạc đà có thể di chuyển từ một ô sang ô kề nó không?

Chứng minh. Câu trả lời là không. Xét các tô màu đen trắng của bàn cờ
thông thường. Dễ dàng kiểm tra được rằng quân lạc đà luôn di chuyển trong
các ô cùng màu và hai ô kề nhau lại là khác màu.

Bài toán 2.11. Một bảng hình chữ nhật có thể phủ kín không đè lên nhau
bởi các hình 1 × 4 và 2 × 2. Khi bỏ các hình này ra ngoài, chúng ta làm mất
một hình 2 × 2 và thay vào đó một hình 1 × 4. Liệu có có thể dùng các hình
lúc này phủ kín hình chữ nhật được không?

Chứng minh. Câu trả lời là phủ định. Hãy tìm một cách tô màu các ô của
hình chữ nhật bởi các màu 1, 2, 3, 4 sao cho mỗi hình 2 × 2 có một ô mỗi màu
và hình 1 × 4 hoặc không có ô màu i hoặc có 2 ô màu i với mỗi i = 1, 2, 3, 4.

228
Bài toán 2.12. Có thể hay không một quân mã đi qua tất cả các ô của bàn
cờ 4 × N , mỗi ô đúng một lần và quay lại ô xuất phát ban đầu?

Chứng minh. Tô màu hàng thứ nhất 1, 2, 1, 2, . . .. Tô màu hàng thứ hai 3, 4, 3, 4, . . ..
Tô màu hàng thứ ba 4, 3, 4, 3, . . ., và tô màu hàng thứ tư 2, 1, 2, 1, . . .. Giả sử
tồn tại một chu trình bởi quân mã trên bàn cờ. Dễ kiểm tra rằng với cách tô
của chúng ta, nếu quân mã đứng ở ô màu 1 hoặc 2 thì bước tiếp theo sẽ là
ô màu 3 hoặc 4, tương ứng. Do số ô mỗi màu là như nhau, các cập màu sẽ
thay đổi luân phiên trong chu trình. Bạn đọc hãy tự chỉ ra rằng lúc này ta
phải luân phiên giữa các ô màu 1 và màu 3 hoặc luân phiên giữa các ô màu
2 và màu 4. Hay nói một cách khác, chúng ta không thể đi hết được cả bàn
cờ.

Bài toán 2.13. Có ba máy in thẻ trong đó các thẻ là một cặp các số tự nhiên
(không sắp thứ tự). Máy thứ nhất nhận một thẻ với hai số a, b và cho ra thẻ
với hai số a + 1, b + 1. Máy thứ hai chỉ nhận các thẻ với hai số chẵn a, b và cho
ra thẻ với hai số a/2, b/2. Máy thứ ba nhận thẻ gồm hai a, b và thẻ gồm hai
b, c và cho ra thẻ gồm hai số a, c. Các máy cũng sẽ trả lại các thẻ được đưa
vào. Có thể nhận được thẻ gồm hai số 1 và 2012 chỉ từ một tấm thẻ gồm hai
số 5 và 19 được không? Tổng quát, từ thẻ gồm hai số 5 và 19 có thể nhận
được những thẻ như thế nào?

Chứng minh. Ba phép toán có thể viết lại dưới dạng (a, b) → (a + 1, b + 1),
(a, b) → (a/2, b/2) và (a, b), (b, c) → (a, c). Trong phép biến đổi thứ nhất, hiệu
giữa hai số trên thẻ không đổi. Trong phép biến đổi thứ hai, hiệu giữa hai
số trên thẻ giảm một nửa. Và trong phép biến đổi thứ ba, hiệu hai số trên
thẻ mới bằng tổng hiệu các số trên hai thẻ cũ. Như vậy, hiệu các số trên thẻ
không phải là bất biến! Tuy nhiên, nếu nhìn kĩ, có thể tính chia hết cho một
số bất kì sẽ được bảo toàn. Kiểm tra được rằng 19 − 5 = 14 chia hết cho 7

229
nhưng 2012 − 1 thì không. Câu trả lời là phủ định. Phần còn lại của bài toán
được dành cho độc giả với gợi ý là tất cả các thẻ gồm hai số a, b thỏa mãn
a − b chia hết cho 7 đều nhận được.

Bài toán 2.14. Một quân cờ di chuyển trên bàn cờ n × n theo một trong ba
cách: đi lên một ô, sang bên phải một ô, đi xuống về bên trái một ô. Hỏi quân
cờ có thể đi qua tất cả các ô, mỗi ô đúng một lần và quay lại ô kề bên phải
ô xuất phát được không?

Chứng minh. Sau mỗi bước, tổng thứ tự của hàng và cột chứa quân cờ hoặc
giảm đi 2 hoặc tăng lên 1. Như vậy, khi xét theo modulo 3 thì tổng này tăng
1 mỗi bước. Do có n2 − 1 bước, nếu kết thúc ở ô kề bên phải ô xuất phát thì
tổng này tăng 1 đơn vị. Do đó n2 − 2 chia hết cho 3, mẫu thuẫn. Vậy câu trả
lời lại là phủ định.

Bài toán 2.15. Có bảy số 0 và một số 1 được điền vào các đỉnh của khối lập
phương. Mỗi bước cho phép cộng thêm 1 vào các số ở một cạnh nào đó. Có
thể thu được khối lập phương với tất cả các số bằng nhau không? Có thể thu
được khối lập phương với tất cả các số chia hết cho 3 không?

Chứng minh. Chúng ta đánh dấu 4 đỉnh của khối lập phương sao cho các
đỉnh này không kề nhau. Xét hiệu giữa tổng các số được đánh dấu và các số
không được đánh dấu thì tổng này không đổi. Sử dụng bất biến này, chúng
ta dễ dàng chứng minh được rằng câu trả lời trong cả hai trường hợp là phủ
định.

Bài toán 2.16. Hình tròn được chia thành 06 hình dẻ quạt, trong đó điền
các số 1, 0, 1, 0, 0, 0 theo thứ tự chiều kim đồng hồ. Cho phép thêm 1 vào các
số trong hai ô kề nhau. Có thể làm cho tất cả các số bằng nhau được không?

230
Chứng minh. Tương tự như Bài 25, đánh số các dẻ quạt 1, 2, . . . , 6 và tô màu
đỏ các dẻ quạt 1, 3, 5. Xét hiệu giữa tổng các số trên các dẻ quạt được tô màu
và tổng các số trên các dẻ quạt còn lại thì tổng này không đổi.

Bài toán 2.17. Chúng ta thực hiện phép biến đổi trên các bộ ba số như sau:
√ √
thay hai số trong chúng, ví dụ a và b, bởi (a + b)/ 2 và (a − b)/ 2. Hỏi có thể
√ √ √ √
nhận được 1, 2, 1 + 2 từ 2, 2, 1/ 2 không?

Chứng minh. Tổng bình phương của các số trong mọi cấu hình là không đổi.
Sử dụng bất biến này, dễ dàng đưa ra câu trả lời phủ định cho bài toán.

Bài toán 2.18. Các số thực được viết lên vòng tròn. Nếu bốn số liên tiếp
a, b, c, d thỏa mãn (a − d)(b − c) > 0 thì có thể đổi chỗ b và c. Chứng minh rằng
quá trình sẽ phải kết thúc.

Chứng minh. Giả sử các số trên vòng tròn theo thứ tự là a1 , a2 , . . . , an . Xét
Pn
X = i=1 ai ai+1 với an+1 = a1 . Khi đó trong mỗi phép biến đổi X tăng thực
sự. Nếu quá trình kéo dài vô hạn thì tổng này có vô hạn giá trị, nhưng tổng
chỉ có tối đa n! giá trị. Suy ra quá trình buộc phải kết thúc.

Bài toán 2.19. Cho một đồ thị n đỉnh, bậc của mỗi đỉnh không quá 5. Chứng
minh rằng các đỉnh có thể tô bởi ba màu sao cho không quá n/2 cạnh có các
đỉnh mút cùng màu.
Lời giải của Bài 29 được dành cho bạn đọc!

3 Nâng cao
Trong phần này chúng ta sẽ thảo luận một số bài toán nâng cao có sử
dụng phương pháp bất biến. Trong 29 bài toán chúng ta đã đề cập từ đầu
đến giờ, các bài toán gần như được giải quyết ngay lập tực khi đã chỉ ra được

231
bất biến phù hợp. Các bài toán trong phần này, ngoài ý tưởng chính là bất
biến, sẽ yêu cầu thêm một số bước biến đổi khác làm tăng độ khó và thú vị
của chúng.

Bài toán 3.1. Các ô vuông được xếp kề nhau tạo thành một dải hình chữ
nhật vô hạn về cả hai phía. Ta xếp vào các ô vuông một số hữu hạn các viên
đá. Mỗi bước, chọn hai viên đá ở cùng ô và chuyển chúng sang hai ô bên cạnh
khác hướng nhau.
a) Có thể sau một số hữu hạn bước quay lại ví trí ban đầu không?
b) Có thể thực hiện vô hạn bước như vậy không?
c) Nếu quá trình dừng lại thì trạng thái sắp xếp cuối cùng có phụ thuộc
vào quá trình thực hiện các bước không?

Chứng minh. Gán cho viên đá ở ô thứ n số n2 . Xét tổng tất cả các số thu
được. Rõ ràng mỗi phép biến đổi ta thay hai số n2 bởi số (n − 1)2 và (n + 1)2 .
Do đó tổng này tăng 2 đơn vị trong mỗi phép biến đổi. Suy ra sau một số
hữu hạn bước không thể quay lại vị trí ban đầu.
Tiếp theo, chúng ta đi chứng minh rằng tổng không thể tăng vô hạn bằng
phương pháp quy nạp. Lưu ý rằng nếu tổng tăng vô hạn, có nghĩa là một số
viên đá sẽ phải tiến ra xa vô hạn. Viên đá cuối cùng bên phải nhất luôn tăng
chỉ số và viên đá cuối cùng bên trái luôn giảm chỉ số. Do đó khoảng cách giữa
hai viên đá này tăng vô hạn. Và đến một lúc nào đó, sẽ có một viên không
chịu tác động của các viên còn lại! Lập luận hoàn chỉnh của phần b và lời giải
của phần c được dành cho bạn đọc.

Bài toán 3.2. Hình tròn được chia thành 2011 hình dẻ quạt. Xếp 2012 viên
kẹo vào các phần dẻ quạt. Mỗi bước, cho phép chuyển hai viên ở cùng một
phần sang hai phần kề khác hướng. Chứng minh rằng đến một lúc nào đó có
ít nhất 1006 phần có chứa kẹo.

232
Chứng minh. Trước hết, chúng ta có 03 nhận xét quan trọng:
a) Do số kẹo lớn hơn một nửa số ô, quá trình ở đây thực hiện được vô hạn
lần;
b) Bài toán sẽ được giải quyết xong nếu ta chứng minh được một lúc nào
đó hai ô kề nhau bất kì có kẹo. Thật vậy, lúc đó số ô có chứa kẹo sẽ ≥ 2011/2
và do là số nguyên nên số ô có chứa kẹo ít nhất là 1006.
c) Đến một lúc nào đó, nếu hai ô kề nhau có ít nhất một viên kẹo thì kể
từ đó, hai ô này luôn luôn có kẹo. Điều này là hiển nhiên từ phép chuyển.
Theo Nhận xét c) nếu tại mọi thời điểm đều tồn tại hai ô liền nhau không
có kẹo thì sẽ tồn tại hai ô liền nhau không bao giờ có kẹo trong tất cả các
phép biến đổi. Ta đánh số các dẻ quạt bởi 1, 2, . . . , 2011 sao cho hai ô đó là 1
và 2011. Gán cho mỗi chiếc kẹo một số tương ứng với số của ô chứa nó và xét
X là tổng bình phương các số đó.
Tương tự như bài trên, X tăng trong mỗi phép biến đổi. Theo Nhận xét
a), X tăng vô hạn. Nhưng lại có X ≤ 1006 × 20102 , dẫn đến mâu thuẫn. Vậy,
đến một lúc nào đó hai ô kề nhau bất kì luôn có kẹo. Bài toán được suy ra
từ Nhận xét b).

Bài toán 3.3. Giả thiết và câu hỏi như ở Bài 30, chỉ khác cách chuyển viên
đá được thực hiện như sau:
a) Bỏ một viên ở ô thứ n − 1 và một viên ở ô thứ n, thêm vào một viên ở
ô thứ n + 1.
b) Bỏ hai viên ở ô thứ n và thêm một viên vào ô thứ n − 2, một viên vào
ô thứ n + 1.

Chứng minh. Nhận xét rằng viên đá ở ô phải nhất sẽ luôn di chuyển về bên
phải và viên đá ở bên trái nhất sẽ luôn đi về bên trái. Nếu như quay lại trạng
thái ban đầu sau hữu hạn bước thì chúng ta sẽ không được tác động đến hai

233
viên này. Khi đó, có thể bỏ đi hai viên này và lặp lại lập luận trên để suy ra
mâu thuẫn. Do đó, không thể quay lại trạng thái ban đầu sau hữu hạn bước.
Chọn α > 1 là nghiệm của α2 − α − 1 = 0. Gán cho viên đá ở ô thứ n số αn
và xét X là tổng các số này. Khi đó tổng X không đổi. Giả sử có thể chuyển
viên đá vô hạn lần thì theo nhận xét trên, các viên đá sẽ tiến ra vô cùng và
khi đó tổng X cũng vậy (do α > 1). Điều này mâu thuẫn với tính bất biến
của X .
Để chứng minh trạng thái sắp xếp cuối cùng không phụ thuộc vào quá
trình các bước chuyển, ta chỉ cần chứng minh nếu từ một trạng thái thu được
hai trạng thái khác nhau thì tổng X sẽ khác nhau. Chi tiết của lập luận này
(thật ra là một bài toán bất đẳng thức đơn giản) được dành cho bạn đọc.

Bài toán 3.4. Có 119 người ở trong 120 căn hộ. Một căn hộ được gọi là quá
tải nếu có nhiều hơn 14 thành viên. Mỗi ngày, các thành viên của một căn
hộ quá tải xảy ra mẫu thuẫn và chuyển sang các căn hộ khác nhau. Hỏi quá
trình có buộc phải kết thúc không?

Chứng minh. Trước khi chuyển nhà cho các thành viên ở căn hộ quá tải bắt
tay nhau. Có thể chứng minh được rằng tổng số cái bắt tay sẽ giảm thực sự.
Và do đó quá trình sẽ buộc phải kết thúc sau hữu hạn bước.

Bài toán 3.5. Trên vòng tròn có 20 số. Cho phép chọn 3 số liên tiếp X, Y, Z
và thay bởi X + Y, −Y, Z + Y . Có thể từ

[1, 2, . . . , 10, −1, −2, . . . , −10]

thu được [10, 9, . . . , 1, −10, . . . , −1] hay không?

Chứng minh. Chọn x1 , . . . , x20 sao cho x1 − x2 , ..., x20 − x1 là bộ 20 số ban đầu.
Khi đó dễ dàng kiểm tra được rằng, phép biến đổi đã cho trên bộ 20 số x1 −

234
x2 , ..., x20 −x1 sẽ tương ứng với việc đổi chỗ hai số cạnh nhau trên bộ x1 , . . . , x20 .
Từ x1 , . . . , x20 tương ứng với [1, 2, . . . , 10, −1, −2, . . . , −10], đổi chỗ liên tiếp các
số cạnh nhau ta thu được x20 , . . . , x1 ương ứng với bộ [10, 9, . . . , 1, −10, . . . , −1].
Do đó, câu trả lời là khẳng định.

Bài toán 3.6. Giả sử tổng của 20 số là dương. Cho phép biến đổi như ở bài
trên, liệu có thể thu được một bộ gồm 20 số không âm hay không?

Chứng minh. Ý tưởng chứng minh tương tự như trên. Chỉ có điều chúng
ta không điền các số trên vòng tròn mà điền trên đường thẳng vô hạn
. . . , x−n , . . . , xn , . . . sao cho chọn 21 số liên tiếp trên đường thẳng thì hiệu
các cặp giữa chúng sẽ là 20 số tương ứng. Ta chỉ cần chỉ ra rằng với các đổi
chỗ như trong giả thuyết của đề bài, ta có thể sắp xếp lại dãy theo thứ tự
tăng. Khi đó, ta sẽ có câu trả lời khẳng định cho bài toán.

Bài toán 3.7. Trên vòng tròn có một số điểm Xanh, Đỏ. Cho phép thêm vào
một điểm Đ và đổi màu hai điểm kề nó, hoặc bớt đi một điểm Đ và đổi màu
hai điểm kề nó. Lúc đầu có hai điểm Đ và quá trình ko được phép làm cho có
ít hơn hai điểm. Hỏi có thể thu được:
a) 2 điểm X, Đ.
b) 8 điểm Đ.
c) 1 điểm Đ, 6 điểm X.
d) 2 điểm X.

Chứng minh. a) Không thể thu được 2 điểm X, Đ do tính chẵn lẻ của số điểm
X không thay đổi.
b) c) Xây dựng được cụ thể.
d) Không được thể thu được 2 điểm X. Do lúc đầu không có điểm X nào
nên số điểm X luôn là chẵn. Đánh số các điểm xanh bởi x1 , . . . , x2n và gọi

235
d1 , . . . , d2n là số điểm đỏ giữa các điểm X. Bất biến là tính chia hết cho 3 của

S = |d1 − d2 + . . . + d2n−1 − d2n |.

Nếu không có điểm X nào thì đặt S là số điểm đỏ. Đây là một bài toán rất
khó và chúng tôi khuyến khích bạn đọc tìm hiểu xem với các cấu hình nào
thì có thể nhận được từ một cặp điểm Đ? Gợi ý rằng đại lượng S sẽ giúp xác
định chính xác các cấu hình như vậy.

236

You might also like